You are on page 1of 249

Abdominal Wall Hernia

1. The diagnosis of an inguinal hernia:


A. In infants often depends on the history given by its mother.
B. In the adult is most easily made with the patient in the sitting position.
C. Depends on the hernial sac or cough impulse being felt below the inguinal ligament.
D. Is supported by the presence of a transilluminable scrotal swelling.
E. B&C only.

2. Inguinal herniae:?????
A. In children are usually of the direct type.
B. Of the indirect type are congenital in origin.
C. Will regress spontaneously in children.
D. In young adults are most commonly of the direct type.
E. C&D only.

3. Strangulated contents of hernial sacs:


A. Are always accompanied by intestinal obstruction.
B. Are more common in direct than indirect inguinal herniae.
C. Are usually reducible.
D. Produce local pain and tenderness # General pain.
E. All of the above

4. Incisional herniae are related to: )All correct except one)


A. Wound infections.
B. Anaemia and malnutrition.
C. Obesity.
D. The use of absorbable suture materials.
E. To the technique of wound closure.

5. herniae in the umbilical region:


A. Are always congenital in origin.
B. Usually occur in males.
C. Usually require surgical repair in infants.
D. Rarely strangulate.
E. All of the above.

6. A discharge from the umbilicus: (all correct except one)


A. May indicate a patent vitello-intestinal duct.
B. May indicate an anomaly of the urachus.
C. At the time of menstration may indicate endometriosis.
D. In the neonate is of no immediate clinical significance.
E. May indicate infection of the umbilicus (omphalitis)

7. An exomphalos:
A. Is a congenital defect of the urethra.
B. Is acquired defect of the anterior abdominal wall.
C. Is otherwise known as gastroschisis.
D. Needs urgent surgical treatment.
E. A&B only.

8. The cremaster muscle is derived from:


A. External oblique aponeurosis.
B. Internal oblique muscle.
C. Transversus abdominis muscle
D. Transversalis fascia
E. Parietal peritoneum.

9. structures lying within the spermatic cord include: ( all correct except one)
A. Direct hernia sac
B. Testicular artery.
C. Indirect hernia sac.
D. Proportional fat.
E. Vas deferens.

10. True statements concerning a femoral hernia include which of the following?
A. It is common in male.
B. It is usually results from a defect in lateral part of transversalis fascia.
C. It is common in children.
D. It is less common in female.
E. It may be confused with inguinal lymphadenopathy.

Acute Abdomen

1. Acute abdominal pain which is (all correct except on)


A. Colicky in nature indicates obstruction of hollow viscus.
B. In right upper quadrant increased by inspiration is typical of cholecystitis
C. Continuous is typical of inflammation
D. Maximal in the right loin is typical with duodenal ulcer.# LEFT
E. Maximal in epigastrium and related to meal is typical of gastric problem.
2. Vomiting in acute abdomen
A. Occurring soon after the onset of colicky pain indicates pathology outside the
gastrointestinal tract.
B. Of fluid containing no bile is characteristic of small bowl obstruction
C. Of faeculent fluid usually indicates gastrointestinal fistula.
D. Of bile stain fluid usually indicates stomach outlet obstruction
E. B&C only

3. Faeculent vomiting:?????????
A. Is commonly seen after upper gastrointestinal tract bleeding.
B. Indicates large bowel obstruction.
C. Indicates bacterial proliferation in the upper intestinal.
D. Suggests a gastro-colic fistula.
E. Suggests small bowel fistula

4. A patient with generalized peritonitis:(all correct except one)


A. Usually has an elevated temperature and pulse rate.
B. Characteristically complains of spasmodic severe pain which causes him to be restless.
C. Is characteristically vomites.
D. Will usually have a rapid and deep respiratory pattern.
E. Is Usually has an elevated WBCs.

5. A perforated duodenal ulcer:??????


A. Usually lies on the anterior or superior surface of the duodenum.
B. Usually presents with the acute onset of severe back pain.
C. Produces radiological evidence of free gas in the peritoneum in over 90 percent of the
patients.
D. Is usually treated by vagotomy and pyloroplasty
E. Is usually treated conservatively.
‫ بدون "اكسبت" ونفس االجابة‬51 ‫!!! السؤال صفحة‬

6. Congential pyloric stenosis:(all correct except one)


A. Occurs more commonly in male children.
B. Usually presents in the first few days of life.
C. Presents with non bile stained vomiting.
D. Is usually diagnosed on clinical examination.
E. May cause right upper quadrant tenderness.
7. Appendicitis is: ????
A. More common in females
B. Distributed evenly thoughout the world's population
C. More likely to occur if the appendix is in the retrocaecal position.
D. Commonly the result of appendicular obstruction.
E. B&C only.

8. Likely differential diagnoses in a young woman with appendicitis include:


A. Ovarian carcinoma
B. Ruptured ectopic pregnancy.
C. Colonic diverticulitis.
D. Caecal carcinoma.
E. C&D only.

9. Obstruction of the lumen of the appendix may lead to:(all correct except one)
A. Mucosal ulceration
B. Gangrenous appendicitis.
C. A perforated appendix.
D. Intussusception of the appendix.
E. Acute appendicitis.

10. Acute non-specific mesenteric lymphadenitis: (all correct except one)


A. Is commonest between 5 and 12 years of age.
B. Is usually associated with an upper respiratory tract infection.
C. Is usually associated with cervical lymphadenopathy
D. Is characterized by enlarged mesenteric lymph nodes which are infected by gram-negative
organisms.
E. Is one of the differential diagnosis of acute appendicitis.

11. The level of intestinal obstruction can be determined by:(all correct except one)
A. Questioning the patient.
B. Examining the patient.
C. Radiological examination of the patient.
D. Repeated measurements of the patient's girth.
E. Upper & lower contrast studies.

12. Acute small bowel obstruction:(all correct except one)


A. Is commonly caused by postoperative adhesions.
B. Accompanied by the signs of peritonitis, suggests bowel strangulation.
C. Is often associated with a raised serum amylase.
D. Generally produces abdominal distension within 2 to 3 hours of onset.
E. Can be diagnosed by history and clinical examination.

13. In the treatment of intestinal obstruction:


A. Nasogastric suction should be instituted preoperatively.
B. Intravenous fluid replacement might be required.
C. Immediate surgery is essential.
D. Surgery should be restricted to those cases where strangulation is diagnosed.
E. C&D only.

14. Strangulation of the bowel:(all correct except one)


A. commonly complicates closed loop obstruction.
B. Is difficult to distinguish from simple intestinal obstruction.
C. Is accompanied by bleeding into the affected bowel.
D. Frequently causes peritonitis.
E. Is characterized by severe abdominal pain.

15. Large bowel obstruction:


A. Is most commonly caused by colonic cancer
B. Has its maximum incidence before the age of 50.
C. Frequently presents with nausea and vomiting.
D. Usually heralds its onset with constant suprapubic pain.
E. Frequently treated conservatively.

16. Patients with acute colonic deveticulitis:??????


A. Often give a history of recent lower abdominal colic.
B. Often present with pyrexia.
C. Can be frequently diagnosed on sigmoidoscopic appearances.
D. Frequently develop faecal peritonitis.
E. All the above.

17. Acute pancreatitis typically:(all correct except one)


A. Is accompanied by hypercalcaemia.
B. Produces paralytic ileus.
C. Is associated with a pleural effusion.
D. Produces pyloric stenosis.
E. Upper abdominal pain and vomiting.

18. Acute pancreatitis:(all correct except one)


A. Often simulates a perforated peptic ulcer in its presentation.
B. Often presents with the signs of hypovolaemia.
C. Can readily be distinguished from other causes of acute abdominal pain by the presence of
a raised serum amylase.
D. Frequently has a raised concentration of urinary amylase.
E. Most commonly caused as a complication of GB stones.
1:25 PM

19. The treatment of acute pancreatitis:


A. Is largely nonspecific and supportive.
B. Should include a laparotomy in the majority of cases.
C. Should routinely include the administration of calcium.
D. Should routinely include the administration of antibiotics.
E. All the above.

20. Childhood intussusception:(all correct except one)


A. Usually presents during the first year of life.
B. Is frequently ileocolic.
C. Can usually be diagnosed without x-ray examination of the abdomen.
D. Rarely requires surgical treatment.
E. Can be diagnosed by abdominal US.

21. Meconium ileus:


A. Is the presenting feature in the majority of patients with cystic fibrosis.
B. Is associated with achlorhydria.
C. Presents with a distended abdomen and bilious vomiting.
D. Often can be effectively treated with acetyl cysteine.
E. All the above.

22. Neonatal duodenal obstruction:


A. May be associated with down's syndrome.
B. Is more frequently found in premature infants.
C. Typically presents with gross abdominal distension.
D. Usually presents with vomiting of non-bile stained fluid
E. B&C only.

23. acute superior mesenteric artery occlusion: (all correct except one)
A. Characteristically presents with sudden pain and tenderness of increasing intensity.
B. Is frequently accompanied by overt or occult blood loss in the stools.
C. Frequently produces peritonitis.
D. Can usually be diagnosed on plain abdominal x-rays. ( ischemia not visible on xray )
E. Can be diagnosed by mesenteric artery ongiography.

24. A ruptured ectopic pregnancy:(all correct except one)


A. Usually occurs in the first month of pregnancy.#after 1.5 month at least .
B. Usually presents with severe lower abdominal pain.
C. Frequently presents with hypovolaemic shock.
D. Can usually be diagnosed by pelvic examination.
E. Can usually be diagnosed by transvaginal US.

25. Biliary colic typically:


A. Occurs 3 to 4 hours after meals.
B. Lasts 5 to 20 minutes.
C. Radiates from the upper abdomen to the right subscapular region.
D. Is made better by deep inspiration.
E. B&C only.

Stomach , Duodenum and Small intestine

1. Bengin gastric ulcers:


A. Occur in the same age group as duodenal ulcers.
B. Are more common in females than males.
C. Are more common in the upper social classes.
D. Produce epigastric pain after eating food.
E. All of the above is correct.

2. Uncomplicated benign gastric ulcers: (All correct except one)


A. Occure most commonly on the greater curve of the stomach.
B. Should initially be treated medically.
C. Commonly recur after medical treatment.
D. Should receive surgical treatment if healing has not occurred after 4 to 6 weeks of
medical.
E. The surgical treatment of gastric ulcer is billoroth one partial gastrectomy.

3. There is an association of gastric cancer with: (All correct except one)


A. Achlorhydria of the stomach.
B. Atrophic gastritis.
C. Adenomatous gastric polyps.
D. Duodenal ulceration.
E. Chronic non healing gastric ulcer.

4. Surgical treatment of gastric cancer:????


A. Should preferably be by a total gastrectomy.
B. Gives overall 5 years survival figures over 40 per cent.
C. Includes excision of the tumour as a palliative measure.
D. Should be advised for all diagnosed cases.
E. The resectability rate is over 70per cent.

5. The signs and symptoms of potentially curable gastric cancer: (All correct except one)
A. May simulate those of a benign gastric ulcer.
B. Include a hard palpable left supraclavicular node.
C. Require investigation by a barium meal examination.
D. Require investigation by a gastroscopic examination even when the barium meal is
normal.
E. Include a non palpable abdominal mass.

6. Gastric cancer:
A. Is most common in the fundus of the stomach.
B. Is most commonly a squamous cell carcinoma.
C. Frequently metastasises via the blood stram.
D. Is most frequently an ulcerating lesion.
E. Non of the above is correct.

7. Treatment of symptomatic duodenal ulcers:


A. Should include twice daily antacids.
B. Shjould include a strict dietary regime .
C. Should include anti-cholinergic drugs.
D. Should be by early surgery.
E. Non of the above is correct.

8. Duodenal ulcers:
A. Have an equal incidence in both sexes.
B. Have a clicical course characterised by long periods of remission.
C. Are characterised by postprandial pain.
D. Occur most commonly ion the duodenal cap.
E. B&D only.

9. Acute haemorrhage from a duodenal ulcer:


A. Requires no urgent diagnosis of the source of haemorrhage.
B. Requires an urgent gastroscopy.
C. Should be treated surgically as soon as the diagnosis is made.
D. Indicates the presence of a chronic duodenal ulcer.
E. C&D only.

10. Post-gastrectomy nutritional disturbances may result in:


A. Megaloblastic anaemia.
B. Steatorrhoea.
C. Iron deficiency anaeimia.
D. Osteoporosis.
E. All of the above.

11. The incidence of crohn’s disease:


A. Is not distributed evenly throughout the world’s population.
B. Is similar in the two sexes.
C. Is highest amongst young adults.
D. Is increased in the close relatives of patients with the disease.
E. All the above are correct.

12. Crohn’s disease:


A. Has an infective aetiology.
B. Is limited to the bowel mucosa.
C. Does not produce mucosal ulceration.
D. Is characterised by the absence of fibrous tissue in the affected inflamed bowel.
E. Spreads through the hole thickness of the bowel wall.

13. Patients with crohn’s disease characteristically present with:


A. Colicky abdominal pain.
B. Constipation.
C. Nutritional deficiencies.
D. Rectal bleeding.
E. A&C only.

14. In the treatment of patients with crohn’s desease:


A. Medical methods have no parts to play.
B. Surgery should be the primary method of treatment.
C. Steroids may provide a remission in the progress of the disease.
D. A high bulk diet should be considered.
E. All of the above are correct.

15. Tumours of the small bowel:


A. Are common.
B. May be an inherited disorder.
C. Most commonly present with overt or occult rectal bleeding.
D. Most commonly present in childhood.
E. C&D only.
16. Carcinoid tumours of the gastrointestinal tract:
A. Are most commonly located in the appendix .
B. Are usually malignant.
C. Arise in the mucosa.
D. Are frequently the cause of gastrointestinal haemorrhage.
E. Mucosal ulceration is common.

17. The carcinoid syndrome:(all correct except one)???


A. Characteristically includes abdominal cramps.
B. Is produced by the release of vasodilating substances from the tumour.
C. Occurs most commonly with metastatic carcinoid tumours.
D. Is usually cured by excision of the primary tumour.
E. May produce diarrhoea and dyspnoea.

18. Chronic radiation injury to the intestinal tract: (all correct except one)
A. Typically presents with mucosal atrophy.
B. Often presents with perforation of the bowel.
C. Frequently presents with intestinal obstruction.
D. May present with malabsorption.
E. Often produce progressive fibrosis.

19. The “blind-loop” syndrome often:


A. Presents with signs of malnutrition.
B. Presents as a complication of crohn’s disease.
C. Presents as a complication of multiple deverticuli.
D. Presents as a complication of intestinal bypass.
E. All of the above are correct.

20. A meckel’s deverticulum of the small intestine: (all correct except one)
A. Is situated at the jejuno-ileal junction.
B. Contains all coats of intestinal wall.
C. May be associated with a fibrous band connecting it to the umbilicus.
D. Most commonly presents as lower GI bleeding diverticulitis.

Fluid And Electrolyte

1. The effective osmotic pressure between the plasma and interstitial fluid compartments is
primarily controlled by:
A. Bicarbonate.
B. Chloride ion.
C. Potassium ion.
D. Protein.
E. Sodium ion.

2. Symptoms and signs of extracellular fluid volume deficit include all of the following
except:
A. Anorexia.
B. Apathy.
C. Decreased body temperature.
D. High pulse pressure
E. Orthostatic hypotension.

3. The osmolarity of the extracellular fluid space is determined primarily by the


concentration of:
A. Bicarbonate
B. Chloride ion
C. Phosphate radicals
D. Sodium ion
E. Sulfate radicals

4. When lactic acid is produced in response to injury, the body minimizes pH change by:
A. Decreasing production of sodium bicarbonate in tissues.
B. Excreting carbon dioxide through the lungs.
C. Excreting lactic acid through the kidneys
D. Lowering renal output of chloride ions
E. Metabolizing the lactic acid in the liver

5. The simplest effective method of estimating the degree of acidosis in a patient in shock is
the measurement of:
A. Arterial pH
B. End tidal CO concentration
C. pH of mixed venous blood
D. serum CO level
E. urinary pH

6. A decrease in intracellular water can be precipitated by:


A. A decrease in sodium in extracellular fluid
B. An increase in sodium in extracellular fluid
C. An increase in sodium in intracellular fluid
D. An isotonic decrease in extracellular fluid
E. An isotonic increase in extracellular fluid

7. The first step in the management of acute hypercalcemia should be:


A. Correction of deficit of extracellular fluid volume
B. Hemodialysis
C. Administration of furosemide
D. Administration of mithramycin
E. Parathyroidectomy.

8. Postoperative third-space accumulation should be managed by intravenous:


A. Albumin
B. Dextrose in water
C. Fluid restriction
D. 1/2 normal saline with potassium supplements
E. Normal saline

9. The normal adult value for:


A. urine output is 1.5 litre/day.
B. Insensible water loss is 200 ml/day.
C. Potassium requirement is 150 mEq (150mmol)/day.
D. Protein requirement is 120 g/day.
E. B&C only

10. Potassium deficiency should be suspected: ( All Correct Except one)


A. In cases of paralytic ileus.
B. When the patient's reflexes are exaggerated.
C. If there is a decrease in height and peaking of the T waves of an ECG.
D. In alkalosis states.
E. In intestinal obstruction.

11. The sodium ion:


A. Is the principal regulator of the intracellular volume.
B. Is the major ionic component of the intracellular fluid volume.
C. Is present in greater concentration in intracellular fluid than extracellular fluid.
D. Is excreted in larger amounts than normal in the early postoperative period.
E. C&D only

12. Acute post traumatic renal failure:(All Correct Except One)


A. May be due to hypovolaemia and poor tissue perfusion.
B. Is particularly associated with crush injuries.
C. May be due to kidney damage following tubular obstruction.
D. Should initially be treated by fluid restriction.
E. Should be treated initially by increase the amount of intravenous fluid.
3:07
PM
13. The anuric patient:
A. Should have a fluid in take of 1 to 1.5 litres per day.
B. Should have no potassium administered.
C. Is at risk from metabolic alkalosis.
D. Should be on continuous urinary catheter drainage.
E. A&D only.

14. Intravenous parenteral feeding: (All Correct Except One)


A. Should deliver at least 2500 calories/day to an adult.
B. Should deliver at least 10g of nitrogen (i.e 66g of protein)/day to an adult.
C. Can be effectively achieved with isotonic solutions.
D. Is with complications with present day solutions and methods of administration.
E. Can be given by central intravenous infusion line (CVL)

Genito-Urinary Tract

1. Haematuria:
A. At the beginning of micturition is usually indicative of urethral pathology.
B. At the end of micturition is usually due to bladder neck pathology.
C. Throughout the urinary stream is typical of renal pathology.
D. In elderly males is usually related to benign prostatic hypertrophy.
E. All of the above.

2. An intravenous pyelogram:
A. Yields most diagnostic information when performed on a slightly hydrated patient.
B. Should be preceded by a plain film of the abdomen.
C. Normally shows incomplete filling of the ureter in any one exposure.
D. Should provide evidence of the presence, if any , of lower urinary tract obstruction.
E. All of the above.

3. Wilms’ tumours:
A. Metastasise readily to the lungs.
B. Metastasise rarely to the bones.
C. Are usually bilateral.
D. Have the worst prognosis of all childhood abdominal tumours.
E. B&C only.
4. In renal transplantation:
A. A donor kidney may be used from a patient with malignancy provided there is no
abdominal involvement.
B. ABO compatibility between donor and recipient does not have to be considered.
C. Satisfactory renal function can be expected with a warm ischaemic time of up to 200
minutes.
D. The characteristic signs of acute rejection include pyrexia, hypertension and leucocytosis.
E. A&B only.

5. An adenocarcinoma of the kidney:


A. Usually occurs in the 35 to 45 age group.
B. Usually presents with a urinary infection.
C. Is often distinguishable from a renal cyst radiologically.
D. Frequently invades and grows along the renal vein.
E. A&C only.

6. Tumours of the renal pelvis:( All correct except one)


A. Rarely present as a mass in the loin.
B. Are possibly due to a urinary carcinogen.
C. Resemble those of the bladder in their histology.
D. Are best treated by a partial or total nephrectomy.
E. Usually cause hematuria and clot colic.

7. Ureteric calculi:
A. Often result from urinary tract infection.
B. Rarely cause haematuria.
C. Are not usually radio-opaque.
D. producing ureteric colic should be surgically removed.
E. B&C only.

8. Cancer of the penis:


A. Is more common in the circumcised.
B. Commonly arises from the corona of glans penis.
C. Is usually an adenocarcinoma.
D. Rarely metastasises.
E. C&D only.

9. Carcinoma of the prostate:


A. Is commonly of squamous cell origin.
B. Usually originates in the periphery of the gland.
C. Usually presents relatively early with lower urinary tract symptoms.
D. Rarely can be diagnosed on rectal examination.
E. All other above.

10. Carcinoma of the prostate:


A. Does not usually metastasise.
B. Usually produces an elevated serum acid phosphatase.
C. Can be effectively treated by hormones.
D. Is most effectively treated by surgery.
E. B&C only.

11. Benign prostatic hypertrophy: ( all correct except one )


A. Is the result of hyperplasia of the fibromuscular capsule of the gland.
B. Results in diminished power of urination.
C. Results in terminal dribbling of urine.
D. Often presents with haematuria.
E. Might cause mild elevation of serum PSA.

12. Benign prostatic hypertrophy:


A. Can readily be assessed on rectal examination.
B. Can be effectively treated with hormones.
C. Is most effectively treated by surgery.
D. Is a premalignant condition.
E. Rarely presented with macroscopic hematuria

13. Acute prostatitis: ( all correct except one )


A. Is most commonly due to coliform organisms.
B. Often presents as an ache in the perineum.
C. May be diagnosed by rectal examination.
D. Requires bladder catherisation as part of the treatment.
E. May be lead to a prostate abscess formation.

14. Bladder cancer: ( all correct except one )


A. May follow exposure to beta-naphthylamine.
B. Is more common in heavy smokers.
C. Is more common in females.
D. Is frequently associated with bladder schistosomiasis.
E. May caused by chronic urinary bladder inflamation.

15. Bladder cancers: ?????


A. Are usually adenocarcinomas.
B. Are usually ulcerating.
C. Usually present with suprapubic pain radiating to the perineum.
D. Are usually diagnosed on cystoscopy.
E. All of the above.

16. Undescended testes:


A. Are often associated with inguinal herniae.
B. Usually descend at puberty.
C. Can usually be made to descend by the examiner with warm hands.
D. Should be treated by orchidopexy at puberty.
E. Can be exist without any complication.

17. The spermatic cord contains:


A. The inferior epigastric vein.
B. The deep circumflex iliac artery.
C. The pudendal nerve.
D. The subcostal nerve.
E. Vasdeference.

18. Torsion of the spermatic cord:


A. Often presents with vomiting and lower abdominal pain.
B. Often produces gangrene of the testis.
C. May be diagnosed clinically.
D. Always requires surgical treatment.
E. All of the above.

19. Seminomas of the testis:


A. Most commonly occur before the age of 40.
B. Are usually sensitive to radiotherapy.
C. Rarely metastasise via the blood stream.
D. Generally carry a good prognosis.
E. All of the above.

20. Hypospadias:
A. Is the result of failure of scrotal development.
B. Results in the abnormal urethra opening on to the dorsum of the penis.
C. Is associated with chordee.
D. Is associated with maldescent of the testis.
E. C&D only.

21. The most ominous sign or symptom of urinary system disease is:
A. Urinary frequency.
B. Pyuria.
C. Pneumaturia.
D. Dysuria.
E. Hematuria.
22. A patient with acute urinary tract infection (UTI) usually presents with:
A. Chills and fever.
B. Flank pain.
C. Nausea and vomiting.
D. 5 to 10 white blood cells
E. Painful urination.

23. Renal adenocarcinomas:


A. Are of transitional cell origin.
B. Usually are associated with anemia.
D. Are extremely radiosensitive.
E. Frequently are signaled by gross hematuria.

24. Ureteral obstruction:???


A. Is associated with hematuria.
B. Is associated with deterioration of renal function and rising blood urea nitrogen (BUN)
and creatinine values.
C. Is commonly caused by a urinary tract calculus.
D. Usually requires open surgical relief of the obstruction.
E. Is usually associated with infection behind the obstruction.

25. Stress urinary incontinence:


A. Is principally a disease of young females.
B. Occurs only in males.
C. Is associated with urinary frequency and urgency.
E. Is a disease of aging produced by shortening of the urethra.

26. The major blood supply to the testes comes through the:
A. Hypogastric arteries.
B. Pudendal arteries.
C. External spermatic arteries.
D. Internal spermatic arteries.

27. Patients who have undergone operations for benign prostatic hypertrophy or
hyperplasia:
A. Require routine rectal examinations to detect the development of carcinoma of the
prostate.
B. Do not need routine prostate examinations.
C. Have a lesser incidence of carcinoma of the prostate.
D. Have a greater incidence of carcinoma of the prostate.

28. To maximize fertility potential, orchidopexy for cryptorchidism should be done before:
A. Age 15 years.
B. Age 12 years.
C. Marriage.
D. Age 2 years.

29. The appropriate surgical treatment for suspected carcinoma of the testis is:
A. Transscrotal percutaneous biopsy.
B. Transscrotal open biopsy.
C. Repeated examinations.
D. Inguinal exploration, control of the spermatic cord, biopsy, and radical orchectomy if
tumor is confirmed.

30. If torsion of the testicle is suspected, surgical exploration:


A. Can be delayed 24 hours and limited to the affected side.
B. Can be delayed but should include the asymptomatic side.
C. Should be immediate and limited to the affected side.
D. Should be immediate and include the asymptomatic side.

31. Epididymitis, either unilateral or bilateral, in a prepubertal male:


A. Is a frequent diagnosis.
B. Can be dealt with on an outpatient basis.
C. Is a major scrotal problem in this age group.
D. Is a rare phenomenon.

32. Patients with prostatitis, especially acute suppurative prostatitis:


A. Should have residual urine measured by intermittent catheterization.
B. Should have bladder decompression by urethral catheter.
C. Should have repeated prostatic massage.
D. Should have no transurethral instrumentation if possible.

33. Benign prostatic hypertrophy with bladder neck obstruction:


A. Is always accompanied by significant symptoms.
B. Is best diagnosed by endoscopy and urodynamic studies.
C. Is easily diagnosed by the symptoms of frequency, hesitancy, and nocturia.
D. Is always accompanied by residual urine volume greater than 100 ml.
34. A 28-year-old white male presents with asymptomatic testicular enlargement. Which of
the following statement(s) is/are
true concerning his diagnosis and management
C. The diagnosis of seminoma should be followed by postoperative radiation therapy

35. Which of the following is/are true of blunt renal trauma?


D. Blunt renal trauma requires exploration only when the patient exhibits hemodynamic
instability.

36. Carcinoma of the bladder:


D. May mimic an acute UTI with irritability and hematuria.

Neurosurgery

1. Intracranial aneurysms:
A. Are the cause of the vast majority of cases of spontaneous subarachnoid haemorrhage.
B. Are multiple in 20 per cent of cases.
C. Rarely rebleed after an initial haemorrhage.
D. Which have ruptured require surgical treatment which involves clipping of the appropriate
middle cerebral artery.
E. C&D only.

2. In head injuries the causes of a rising intracranial pressure: (all correct except one)
A. Intracerebral haemorrhage.
B. Cerebral oedema.
C. Rhinorrhoea.
D. Meningitis.
E. Extradural hemorrhage.

3. In head injuries the signs of an expanding intracranial lesion include:


A. A falling level of consciousness.
B. A rising pulse rate.
C. A falling blood pressure.
D. Small pupils.
E. B&C only.

4. Following head injuries surgical intervention is usually required for:


A. Linear skull fractures.
B. Cerebral oedema.
C. Depressed skull fractures.
D. Extradural haemorrhage.
E. C&D only.
5. Chronic subdural haematomata: (all correct except one)
A. Are common in the young.
B. Have a characteristic angiographic appearance.
C. Should be treated surgically.
D. Have a better postoperative prognosis than acute subdural haematomata.
E. Are venous in orgin.

6. The characteristic signs of chronically raised intracranial pressure include: (all correct
except one)
A. A bitemporal hemianopia.
B. Papilloedema.
C. Epilepsy.
D. Bradycardia.
E. Six nerve pulsy.

7. In a patient with a lumber disc protrusion:???


A. Loss of the knee jerk is characteristic of a second lumbar nerve root lesion.
B. Loss of dorsiflexion of the great toes indicates a third lumbar nerve root lesion.
C. Loss of sensation over the sole indicates a fourth lumbar nerve root lesion.
D. Loss of the ankle jerk indicates a first sacral root lesion.
E. Loss of planter flexion of great toe indicates fifth lumber nerve root lesion.

8. In lesions affecting the common peroneal (lateral popliteal) nerve:


A. Sensory loss is limited to the dorsal aspect of the first interdigital cieft.
B. There is weakness of dorsiflexion of the foot.
C. There is weakness of eversion of the foot.
D. The toes become clawed.
E. B&C only.

9. Birth injuries involving the fifth and sixth cervical nerve roots of the brachial plexus:
A. Are known as klumpke’s palsy.
B. Are rarely followed by full recovery.
C. Are characterised by the arm being held in the pronated and internally rotated position.
D. Show weakness and wasting of the small muscles of the hand.
E. Non of the above correct.

10. Following a peripheral nerve injury:


A. Loss of axon continuity is described as neuropraxia.
B. Due to gunshot wounds primary nerve repair is desirable.
C. Delayed suture is best performed one week after the injury.
D. Delayed suture is best performed three months after the injury.
E. None of the above is correct.

Shock and blood transfusion


4:34
1. 'Shock' can be most comprehensively defined as:
A. A sudden large volume blood loss.
B. A diminished effective circulating fluid volume.
C. A hypotensive state with peripheral vasoconstriction.
D. An unexpected psychological insult.
E. A&C only.

2. In all forms of shock there is:


A. An impairment of cellular oxygenation.
B. A decreased cardiac output.
C. An increased effective circulating fluid volume.
D. A low central venous pressure (CVP).
E. An increased pulse rate.

3. The metabolic acidosis of shock can be effectively treated by:


A. Warming the patient.
B. Administering ammonia chloride.
C. Artificial ventilation.
D. Restoring normal tissue perfusion.
E. Nacl intravenous infusion.

4. In hypovolaemic shock:
A. The central venous pressure is high.
B. The difference in arteriovenous oxygen tension is unaffected.
C. The extremities are pale, cold and sweating.
D. Urine output is unaffected.
E. C&D only.

5. Hypovolaemic shock may result from:(all correct except one)


A. A 25 per cent third degree burn.
B. Generalized peritonitis.
C. Massive pulmonary embolism.
D. Intestinal obstruction.
E. Massive blood loss.
6. Septic shock is particularly associated with: (all correct except one)???
A. Thoracic surgical patients.
B. Hypervolemia.
C. Indwelling urinary or intravenous catheters.
D. Gram-negative bacteremia.
E. Gram-positive.

7. septic shock is associated with a hypodynamic cardiovascular state:( all correct except
one)
A. if preceded by existing hypovolaemia.
B. In generalized peritonitis.
C. When there is a gram-positive bacteraemia.
D. In elderly patients.
E. In late gram negative septicemia.

8. The mortality from septic shock can be effectively reduced by:(all correct except one)
A. Surgical drainage of abscesses.
B. The administration of appropriate antibiotics.
C. The restoration of a normal cardiovascular state.
D. Positive pressure respiration via an indotracheal intubation or tracheostomy.
E. The administration of corticosteroid.

9. In cardiogenic shock:
A. The central venous pressure is high.
B. The difference in the arteriovenous oxygen tension is increased.
C. The haematocrit is raised.
D. The blood pressure is unaffected.
E. C&D only.

10. Prospective blood donors:


A. Should be asked about previous attacks of jaundice.
B. Should have serological tests for syphilis.
C. May transmit glandular fever to a recipient.
D. May transmit malaria to a recipient.
E. All the above.

11. A blood transfusion reaction:(all correct except one)


A. May be due to incompatibility of the recipient serum and donor cells.
B. Is manifest by thrombophlebitis of the infusion site.
C. Occurs within the first 30 minutes of transfusion.
D. May produce renal damage.
E. May produce anaphylactic shock.

12. Pyrexial reactions to blood transfusions:(all correct except one)


A. Have decreased since the introduction of sterile disposable infusion sets.
B. May be caused by allergic reactions.
C. May be caused by contaminated blood.
D. May be a response to a large transfusion of cooled blood.
E. Usually caused by pyrogen reaction.

13. Massive blood transfusions may be complicated by:(all correct except one)
A. Hyperkalemia.
B. Hypocalcaemia.
C. Coagulopathy.
D. Leucopenia.
E. DIC.

Surgical Infection Wound Healing , Postoperative


Complications , Hernia

1. Staphylococcal infections:
A. Do not cause cellutitis.
B. Do not produce septicaemia.
C. Do not produce fever.
D. Produce yellow odourless pus.
E. A&C only

2. Streptococcal infections:
A. Are characterised by abscess formation.
B. Rarely produce lymphadenitis.
C. Do not produce baceraemia.
D. Can produce a gangrenous skin infection.
E. All of the above.

3. Tatanus prophylaxis in a patient with a badly contaminated wound:(all correct except


one)
A. Depends, even in an actively immunised patient, on meticulous immediate debridement of
the wound.
B. Should include the administration of tetanus toxoid.
C. Is more safely achieved with human antitoxin.
D. Is unnecessary in patients who have been recently actively immunised.
E. Is necessary for the actively immunised patient to have a booster dose of toxiod.

4. Tetanus:
A. May have an incubation period of over 20 days.
B. Can be prevented by the immediate administration of tetanus toxoid.
C. Is more common after scalp lacerations than wounds of the extremities.
D. Is usually associated with stupor or coma.
E. B&C only.

5. In clostridial infections:???
A. A spreading cellutitis may be present.
B. Gram-positive cocci can be isolated from the discharge.
C. Surgical treatment has a minor part to play.
D. Gas production is often present.
E. C&D only.

6. Actinomycosis is characterised by:


A. Chronic abscesses of the foot.
B. A granulomatous abcess.
C. Red coloured pus.
D. Its resistance to antibiotics.
E. Blake coloured pus

7. A subphrenic abscess:
A. Is usually accompanied by consederable systemic effects.
B. Is associated with local rib tenderness.
C. Rarely produces abnormal signs in the chest.
D. Usually can be diagnosed by a barium meal examinations.
E. A&C only.

8. A pelvic abscess:
A. Lies extrapeitoneally.
B. May be a complication of abdominal surgery.
C. Rarely presents with diarrhoea.
D. Should be treated with antibiotics alone.
E. A&C only

9. Paralytic ileus:
A. Is associated with electrolyte imbalance.
B. May be associated with mechanical intestinal obstruction.
C. Requires treatment with nasogastric suction and intravenous fluids.
D. Is associated with retroperitoneal haematoma
E. All of the above.
4:49
6:9
10. Acute postoperative gastric dilatation:???
A. Usually cause postoperative vascular collapse.
B. Can be prevented by regular nasogastric aspiration.
C. Characteristically occurs on the first postoperative day.
D. Is a relatively common problem after surgery on the gastrointetinal tract.
E. A&D only.

11. The appearance of jaundice in the postoperative period: (all correct except one)
A. May indicate an intraperitoneal haemorrhage.
B. Is usually due to the toxic effects of anaesthetic agents.
C. May be due to septicaemia.
D. May indicate chronic liver disease.
E. May be due acute liver insalt

12. A clean incised skin wound:


A. Undergoes an inflammatory phase during the processes of repair.
B. Commences epithelialization after 7 to 10 days.
C. Regains the full strength of normal skin within 10 days.
D. Regains its strength as the result of fibroblast activity.
E. A&D only.

13. Wound healing:


A. Is impaired in anaemic patients.
B. Is impaired by haematoma formation.
C. Is not impaired by hypoproteinaemia.
D. Is stimulated by steroids.
E. A&D only.

14. The principles of wound care include: (All except one)


A. Early skin cover.
B. Removal of foreign material.
C. Routine administration of antibiotics.
D. Close apposition of uninfected wounded tissues.
E. Wound toilet if possible.

15. Heavily contaminated and dirty wounds:


A. Require surgical toilet and delayed closure.
B. Require the administration of systemic antibiotics.
C. Can usually be treated by wound toilet and primary closure.
D. Should be totally excised.
E. A&B only.
16. surgical drainage of abscesses:???????
A. Should be via a small incision with minimal disturbance of the adjacent tissues.
B. Should be dependent wherever possible.
C. Has been outmoded by antibiotic therapy.
D. Should be undertaken before the signs of fluctuation appear.
E. Usually performed under local anaethesia.

17. In localised surgical infections:


A. An elevated leucocyte count is usually present.
B. Fever and tenderness are usually present.
C. The presence of glycosuria usually indicates metastatic pancreatic abscesses.
D. Pus is frequently absent.
E. A&B only.

18. A distal small bowel fistula:


A. Loses intestinal fluid rich in potassium and sodium.
B. May give rise to a metabolic acidosis.
C. Rapidly results in dehydration.
D. May be managed conservatively by the abandonment of oral feeding.
E. All of the above.

19. A pancreatic fistula:


A. Loses fluids with high potassium and lower sodium levels than plasma.
B. May give rise to a metabolic acidosis.
C. Requires long term treatment with intravenous normal saline.
D. Will usually close if oral feeding is temporarily suspended.
E. All of the above.

20. Bed sores:


A. Can be prevented by sheepskin blankets.
B. Can be prevented by changing the patient’s position four times each 24 hours.
C. Are the consequence of local ischemia.
D. Only occur over the sacrum.
E. A&B only.

Thyroid, parathyroid And Adrenal Glands

1. Signs and symptoms of hyperthytoidism include: (All correct except one)


A. Decreased sweating.
B. An irregular pulse rate.
C. Cardiac failure.
D. Diplopia.
E. Loss of weight.
2. Symptoms of hyperthyroidism include:
A. Intolerance to heat.
B. Increased appetite.
C. Emotional instability.
D. Diarhoea.
E. All of the above.

3. Medical therapy of hyperthyroidism:


A. Is particularly indicated in the pregnant patient.
B. Will usually produce a long-lasting remission rate of 70 to 80 per cent after a 6 month
course.
C. Is occasionally complicated by bone marrow depression.
D. Depends on the efficacy of the drugs in inhibiting the hypothalamic centres which govern
thyroid stimulating hormone (TSH) release by the pituitary.
E. B&D only.

4. Treatment of hyperthyroidism with radioactive iodine:


A. Is complicated by the late occurrence of thyroid cancer.
B. Should not be undertaken in patients below the age of 40.
C. Is frequently complicated by the development of myxoedema.
D. Is often complicated by a short-lived exacerbation of the hypethyroid state.
E. A&B only.

5. The surgical treatment of hyperthyroidism:????


A. Is commonly followed by hypothyroidism.
B. Has a high rate of recurrence.
C. Can be safely effected without preoperative drug therapy.
D. May be complicated by postoperative high parathormone levels.
E. Non of the above.

6. a multinodular (adenomatous) goitre:


A. Is more common in those patients having a deficient iodine intake.
B. Is usually preceded by a diffuse goitre in early adult life
C. Is rarely a precancerous condition.
D. Is effectively treated in early stage with thyroid hormones to prevent further adenomatous
changes.
E. All of the above.

7. auto-immune thyroiditis (Hashimoto’s disease):


A. Characteristically occurs in young females.
B. Often presents with hyperthyroidism.
C. Produces a soft diffuse enlargement of the thyroid gland.
D. Is associated with regional lymph node enlargement.
E. C&D only.

8. Hashimoto’s disease:
A. Is often associated with increased levels of circulating thyroid antibodies.
B. Is characterized by lymphocytic infiltration and fibrosis of the thyroid gland.
C. Is usually treated by sub-total thyroidectomy.
D. Should be treated by anti-thyroid drugs.
E. A & B only.

9. Thyroid cancer:???
A. Frequently produces multinodular enlargement of the gland.
B. Usually diagnosed by preoperative needle biopsy of the gland.
C. Can be excluded if there is a localised but soft enlargement of the gland.
D. Often produces a bruit over the gland.
E. Non of the above.

10. Papillary carcinoma of the thyroid:


A. May follow radiation exposure in childhood.
B. Is a slow growing tumor.
C. Usually metastasises to lymph nodes.
D. Is usually a multifocal tumor.
E. All of the above.

11. Follicular carcinoma of the thyroid:(all correct except one)


A. Is most common in females.
B. Is most common above the age of 30 years.
C. Usually metastasises to lymph nodes.
D. Is associated with a relatively good prognosis in childhood.
E. Usually treated by total thyroidectomy.

12. Undifferentiated carcinoma of the thyroid:??


A. Is most common in females.
B. Is most common above the age of 60.
C. Is often thyroid stimulating hormone (TSH) dependent.
D. Often exhibits independent thyroid hormone production.
E. Is usually treated by thyroidectomy.
13. The surgical treatment of thyroid cancer:
A. Should be by total thyroidectomy in the potentially curable patient.
B. Should include block dissection of the neighboring L.N whether or not they appear to
contain tumor.
C. Is most successful in the papillary type of cancer.
D. Should be preceded by a therapeutic dose of iodin 131.
E. A& C only.

14. Recurrent thyroid cancer can effectively be treated by:


A. High doses of iodin 131.
B. High doses of thyroid hormone.
C. External irradiation.
D. Chemotherapy
E. All correct except d.

15. The earliest symptoms of hyperparathyroidism include:


A. Diarrhoea.
B. Polydipsia and polyuria.
C. Unexplained weight gain.
D. Mucle spasm
E. C&D only.

16. Primary hyperparathyroidism: (All correct except one)


A. Is most common in postmenopausal females.
B. Is occasionally deagnosed by routine biochemical screening of patients.
C. Should be considered when a patient presents with tatany.
D. Can be caused by adenomas or hyperplasia of the glands.
E. Usually presents by vague abdominal pain, bone pain and uretric calculi.

17. Renal symptoms of hyperparathyroidism:


A. May be due to renal stones.
B. May be due to nephrocalcinosis.
C. Are frequently those of chronic renal failure.
D. Are associated with hypertension.
E. All of the above correct.

18. Acute adrenal insufficiency:


A. In the newborn is due to haemorrhage into one or both adrenal glands.
B. In the newborn typically occurs in the second week of life.
C. In the adult usually follows a pneumococcal septicaemia.
D. In the adult is most frequently encountere following the routine administration of steroid.
E. All of the above correct.

19. Phaeochromocytomata:???
A. Are tumors of the spinal nerve roots.
B. Are frequently multiple.
C. Characteristically present with a yellow skin discolouration.
D. Can be effectively managed by long term medical therapy.
E. C&D only.

20. In cushing’s syndrome:???


A. There is usually an increased deposition of fat over the face and trunk.
B. Hypotension ans hypokalaemia are frequently present.
C. There is an increased production of adrenal hormones.
D. The cause is most frequently an adrenal tumour.
E. A&C only.

21. Cushing’s syndrome:(all correct except one)


A. May be due to adrenal cortical hyperplasia, adrenal adenomata or adrenal carcinomata.
B. May be associated with bronchogenic cancer.
C. Is effectively treated by total bilateral adrenalectomy even when shown to be pituitary
dependent.
D. Should be managed medically unless complications develop.
E. Usually treated by surgery.

22. A thyroglossal cyst:


A. Usually presents in early adult life.
B. Is a remnant of fifth pharyngeal arch mesoderm.
C. Should usually be excised with the thyroid cartilage.
D. Does not move with swallowing.
E. Non of the above is correct.

23. Neuroblastomata:
A. Usually arise in the renal substance.
B. Are one of the commonest solid tumours of childhood.
C. Usually metastasise via the blood stream.
D. Are difficult to distinguish from wilms’ tumours on clinical examination.
E. B&C only.

Trauma & Burns


9:30
1. When determining the depth of a burn:
A. A knowledge of the type of injury is important.
B. The presence of blisters is of no clinical significance.
C. Impairment of sensibility of the burned area denotes full thickness burn.
D. The presence of severe pain denotes a full thickness skin loss.
E. A&C only.

2. Estimation of the area of a burn: ( all correct except one)


A. Is of very little clinical significance.
B. Provides important prognostic information.
C. Is an important factor in the estimation of the fluid required.
D. Can be based on a formula which states that the adult trunk is 36 per cent of the whole
body surface area.
E. Should be recorded in each chart of burned patient.

3. Patients with major burns:9:34


9:44
A. Are in a negative nitrogen balance.
B. Have normal calorie requirements.
C. Do not generally become anaemic.
D. Are resistant to septicaemia.
E. All of the above.

4. The catabolic response to trauma and infection is characterized by:


A. An increase in lean body mass.
B. A positive nitrogen balance.
C. Gluconeogenesis.
D. A falling haemoglobin level.
E. A&B only.

5. The catabolic response to trauma:


A. Is related to the severity of the trauma.
B. Is accompanied by decreased urinary losses of potassium and nitrogen.
C. Can be prevented by parenteral nutrition.
D. Does not occur in the adrenalectomised patient.
E. C&D only.

6. Scalds:
A. Are more frequent in children.
B. Commonly cause full thickness skin loss.
C. Should be skin grafted within 48 hours of the injury.
D. Need routine antibiotic treatment.
E. All of the above.

7. The dressing of a small burn should be:??`


A. Occlusive.
B. Non-absorbtive.
C. Non-compressive.
D. Changed daily as a routine.
E. All of the above

8. A partial thickness burn:


A. Usually requires grafting.
B. May deteriorate into full thickness skin loss.
C. Rarely causes severe physiological derangement of the patient.
D. Heals within 7 days in the absence of infection.
E. C&D only.

9. Fluid losses in a major burn:


A. Are maximal between 12 and 24 hours after the injury.
B. Are related to the age of the patient.
C. Are not related to the weight of the patient.
D. Are related to the area burnt.
E. Are not related to the burnt duration.

10. The increased fluid requirements of a patient with a full thickness burn are due to: (all
correct except one)
A. Increased evaporative water loss.
B. Sequestration of fluid in the injured tissues.
C. Serum exuding from the burned area.
D. Destruction of blood in the skin vessels.
E. Vomiting & paralytic ileus occasionally

11. 48 hours after a major burn and with satisfactory fluid therapy a patient:????
A. Has very few abnormal fluid losses.
B. May need a blood transfusion.
C. Is often hypornatraemic.
D. Usually needs skin grafting.
E. C&D only.

12. Major burns are sometimes complicated by: (all correct except one)
A. Acute gastric and duodenal ulcers.
B. Paralytic ileus.
C. Cerebral oedema.
D. Mesenteric thrombosis.
E. Septicemia.

13. If burned patient, associated pulmonary injury: (all correct except one)
A. Should be suspected in head and neck burns.
B. Should be suspected when the nasal hairs are burnt.
C. Does not appear clinically in the first 24 hours.
D. Should be suspected if burns occurred in closed space.
E. May require ICU care.

14. Secondary infection of burns:???


A. Is less common in partial than in full thickness skin loss.
B. Is relatively more common in burns of more than 20 per cent body area.
C. Is avoided by leaving the burn eschar intact.
D. Is avoided by the immediate application of a sterile occlusive dressing.
E. All of the above.

15. The early management of a burn wound may include:


A. Early excision.
B. Occlusive dressings.
C. Exposure treatment.
D. Dressings with local antibiotics.
E. All of the above.

16. Skin grafting of a burn wound:


A. Should usually be with full thickness skin grafts.
B. Is more likely to be successful if undertaken in the first week after injury.
C. Will be unsuccessful unless the wound surface is sterile.
D. Minimizes scar contracture.
E. A&C only.

17. The prognosis of a burned patient is:


A. Not related to the patient's age.
B. Related to the area burnt.
C. Generally better below the age of 10 years.
D. Very poor in the patient with burns of over 40 per cent surface area.
E. C&D only.
18. Nasotracheal intubation:
A. Is preferred for the unconscious patient without cervical spine injury.
B. Is preferred for patients with suspected cervical spine injury.
C. Maximizes neck manipulation.
D. Is contraindicated in the patient who is breathing spontaneously.

19. Which of the following statements about head injuries is/are false?
A. The majority of deaths from auto accidents are due to head injuries.
B. Head injury alone often produces shock.
C. A rapid and complete neurologic examination is part of the initial evaluation of the trauma
patient.
D. Optimizing arterial oxygenation is part of initial therapy.

20. Which of the following statements about maxillofacial trauma is/are false?
A. Asphyxia due to upper airway obstruction is the major cause of death from facial injuries.
B. The mandible is the most common site of facial fracture.
C. The Le Fort II fracture includes a horizontal fracture of the maxilla along with nasal bone
fracture.
D. Loss of upward gaze may indicate either an orbital floor or orbital roof fracture.

21. What percentage of patients with thoracic trauma require thoracotomy?


A. 10%–15%.
B. 20%–25%.
C. 30%–40%.
D. 45%–50%.

22. Hemorrhage initiates a series of compensatory responses. Which of the following


statement(s) is/are true concerning the physiologic responses to hemorrhagic shock?
A. An immediate response is an increased sympathetic discharge with resultant reflex
tachycardia and vasoconstriction
B. Trans capillary refill is a response serving to restore circulating volume
C. Extracellular fluid becomes increasingly hyperosmolar
D. Adrenergic ally mediated vasoconstriction is well maintained at the arteriolar and
precapillary sphincters

23. Which of the following steps is/are part of the primary survey in a trauma patient?
A. Insuring adequate ventilator support
B. Measurement of blood pressure and pulse
C. Neurologic evaluation with the Glasgow Coma Scale
D. Examination of the cervical spine
1:10
24. Immediate life-threatening injuries that preclude air exchange which can be treated in
the field include which of the following?
A. Tension pneumothorax
B. Massive open chest wounds
C. Sucking chest wounds
D. Tracheal disruption

25. Which of the following statement(s) is/are true concerning the diagnosis of a peripheral
vascular injury?
A. The presence of a Doppler signal over an artery in an extremity essentially rules out an
arterial injury
B. Doppler examination is a valuable tool in the diagnosis of venous injuries
C. A gunshot wound in the proximity of a major vessel is an absolute indication for
arteriography
D. Both the sensitivity and specificity of arteriography of the injured extremity approaches
100%

26. Which of the following statement(s) is/are true concerning the management of chest
trauma?
A. The majority of injuries to the chest require surgical intervention
B. The posterior lateral thoracotomy is the optimal approach for emergency thoracotomy
C. Either computed tomography or angiography are suitable methods for detecting aortic
disruption in a patient with an abnormal chest x-ray
D. Persistent bleeding associated with a penetrating injury to the chest is often due to injury
to an artery of the systemic circulation

27. A 22-year-old male driving a car at a high speed and not wearing a seatbelt, leaves a
road and crashes with a full frontal impact into a tree. Which of the following injury
patterns may be predictable from this type of motor vehicle accident?
A. Orthopedic injuries involving the knees, femurs, or hips
B. Laceration to the aorta
C. Hyperextension of the neck with cervical spine injury
D. Diaphragmatic rupture due to marked increase in intraabdominal pressure

28. Indications for Cesarean section during laparotomy for trauma include:except?
A. Maternal shock after 28 weeks gestation
B. Unstable thoracolumbar spinal injury
C. Mechanical limitation for maternal repair
D. Maternal death if estimated gestational age is at least 28 weeks
29. Important physiologic alterations of pregnancy which may alter the injury response
include: except????????????
A. Increased cardiac output
B. Expanded plasma volume
C. Decreased fibrinogen and clotting factors
D. Partial obstruction of the inferior vena cava

30. The treatment of patients with high-voltage electric injury differs from that of patients
with conventional thermal injury with respect to the need for: except????????
A. Fasciotomy.
B. Hemodialysis.
C. Amputation.
D. Pulse oximetry.
E. Prehospital cardiopulmonary resuscitation.

31. Characteristics of the hyper metabolic response to burn injury include:


A. Elevation of core temperature, skin temperature, and core-to-skin heat transfer.
B. Ambient temperature dependency of metabolic rate.
C. A marked increase of blood flow to the burn wound.
D. Oxidation of stored lipd as the major source of metabolic energy.

32. Which of the following statement(s) is/are true concerning the initial fluid resuscitation
of a burn patient?
A. Rigid adherence to the Modified Brooke formula is advised
B. In general, children require less fluid than that predicted by standard formulae
C. Patients with inhalation injuries require less fluid than predicted by standard formulae
D. Dextrose should not be given as the primary resuscitative fluid for any age group
E. Most resuscitative formulae withhold colloid solutions until 24 hours post-injury

33. Which of the following are accepted adjuncts in the management of hypertrophic scar?
Except?
A. Local steroid injection
B. Compression garments
C. Topical silicone
D. Release or excision with auto grafting
E. Topical platelet-derived growth factor

34. Which of the following statement(s) is/are true concerning carbon monoxide and
cyanide exposure?
A. A normal oxygen saturation by standard transmission pulse oximetry precludes the
possibility of significant carboxyhemoglobinemia
B. Most patients with cyanide exposure require administration of sodium thiosulfate
C. The half-life of carbon monoxide is reduced by a factor of 5 with ventilation with 100%
oxygen
D. Even if fire victims are well ventilated with high concentrations of oxygen by emergency
response personnel from the time of extrication, carboxyhemoglobin values are frequently
greater than 10% on initial evaluation

35. Regarding tension pneumothorax, the first step in the management is:
A. Obtaining a stat chest x-ray.
B. Cricothyroidotomy
C. Pass in an endotracheal tube
D. Starting oxygen by a valve-mask device
E. Chest decompression needle.

36. Regarding central facial bone injury in polytrauma victim Who has labored breathing
the first step in management is:
A. Perform tracheostomy
B. Perform chricothyroidotomy
C. Pass endotracheal tube by oral root.
D. Pass endotracheal tube by nasal root
E. Obtain a cross table x-ray of cervical spine.

37. A young man is admitted to Emergency department with a stab wound to upper third
of his right thigh with severe bleeding,the first thing to do is:
A. Apply compression to the bleeding vessel with gloved finger.
B. Apply bandage and splint to the right thigh.
C. Apply tourniquet on right side above the wound.
D. Use clamps to control the bleeding.
E. Start blood transfusion.

38. Regarding the management of polytrauma:?????


A. Death follow a trimodal distribution.
B. X-ray after primary survey should be AP cervical spine, chest and pelvis.
C. Cardiac tamponade is characterized by raised B.p, a low JUP.
D. Assessment of uncomplicated limb fractures should occur during the primary survey.
E. A and B only.

39. Blunt injuries to the abdomen (all are correct except)


A. May cause shock
B. May cause peritonitis
C. May cause acute gastro duodenal ulceration
D. May treated conservativly
E. Rarely need urgent laparatomy

40. A serious intra-abdominal injury in a comatose patient may be diagnosed by: (all
correct except one)
A. Abdominal paracentesis.
B. The observation of pattern bruising on the abdominal wall.
C. Falling of heamoglobin values.
D. The presence of marked abdominal distetion
E. The presence of diarrhea.

41. Injuries to the urethra (all are correct except one)


A. Are more common in male.
B. Are often caused by road traffic accidents.
C. Are easily diagnosed on intra venous pyelography.
D. Require urgent surgical treatment.
E. Diagnosed by retrograde urethragraphy.

42. Regarding injury to the spleen(all are correct except one )


A. Is the commonest organ injured in blunt abdominal trauma.
B. Usually continues to bleed once its capsule and its pulp lacerated.
C. Usually managed conservatively in polytrauma patient.
D. Should be removed if there is severe laceration of its capsule.
E. The risk of infection is increase after its removal.

43. Regarding urinary tract injuries(all are correct except one)


A. Are usually accompanied by hematuria.
B. Require urgent intravenous pyelogram.
C. Involving the kidney require urgent surgery.
D. May cause by RTA.
E. Which demonstrate extravasations of urine from urinary bladder require surgery.

44. Patient with renal trauma ??????


A. Usually present with hematuria
B. Usually require surgical management.
C. May require an intravenous pyelogram.
D. Often present with acute renal failure.
E. B & C only.

45. Regarding traumatic rupture of the spleen (all are correct except one )
A. Frequently presents more than 7 days after the causative injury.
B. Frequently presents with shoulder tip pain
C. May be diagnosed by paracentesis.
D. May be accompanied by other abdominal organ injuries.
E. Usually manage conservatively

46. Car seat belts when properly adjusted???


A. Prevent injuries to abdominal organs.
B. May cause small bowel injuries.
C. Do not reduce the incidence of head injuries of passengers involving in RTA.
D. Protect the cervical spine during sudden acceleration
E. A & D only.

47. Penetrating wounds of the abdomen.


A. Can be adequately explored by a probe.
B. Frequently result in acquired abdominal wall hernia.
C. May be managed by careful observation, laparatomy being indicated if signs of peritonitis
occur.
D. Can be treated conservatively if cause by gun shot.
E. A&D only.

48. In head trauma patient after control of air way , the first diagnostic study
A. X-rays of the skull.
B. CT scan of the head.
C. X-rays of cervical spine
D. Carotid angiography.
E. Lumber puncture.

49. In abdominal trauma with signs of hypovolamic shock the first thing to do after
resuscitation of the patient is
A. Obtain lateral x-rays of cervical spine.
B. Obtain supine x-rays of the abdomen.
C. Obtain abdominal CT scan.
D. Obtain an abdominal aortogram.
E. Explore the abdomen.

50. An early signs of anterior compartment syndrome in the calf is:


A. Absence of pulse in the foot.
B. Firm calf muscle
C. Foot drop
D. Paraethesia between the big and second toes.
E. Painful calf muscle.
Vascular Systems
7:12

1. Acute arterial occlusion:


A. Should be treated conservatively if the site of the occlusion is above the inguinal ligament.
B. Demands the urgent use of vasodilator drugs.
C. Of a limb is usually painless due to the anoxic damage produced in the peripheral nerves.
D. May produce irreversible muscle necrosis after 6 hours.
E. B&C only.

2. In chronic arterial occlusion of the lower limbs:??


A. Buttock claudication is suggestive of arterial occlusion above the inguinal ligament.
B. Skin ulceration most commonly occurs along the medial border of the foot.
C. Which is severe, there may be venous’ guttering on elevation of the legs.
D. There is usually an associated peripheral neuropathy.
E. Usually the peripheral pulses are present.

3. Abdominal aortic aneurysms:


A. Arise from an atheromatous vessel in approximately 50 per cent of cases.
B. Characteristically produce epigastric pain.
C. Are associated with duodenal ulceration.
D. Which are asymptomatic are relatively benign conditions and should not usually be
resected.
E. Which are less than 7cm in diameter should be treated surgically.

4. Raynaud’s disease:??
A. Is caused by an abnormal sensitivity of skin vessels to cold.
B. Is marked by a characteristic pallor of the hands after cold stimulation followed by blue
and then re colour changes.
C. May be associated with scleroderma.
D. Is permanently relieved by sympathectomy in the vast majority of cases.
E. A & B only.

5. Common sites for atheromatous arterial aneurysms are:?


A. The femoral artery.
B. The middle cerebral artery.
C. The abdominal aorta.
D. Intrarenal.
E. A&C only
6. A dissecting aneurysm of the aorta:( all correct except one)
A. Usually starts around the aortic arch.
B. Is so-called because of the extensive mediastinal destruction it produces when it ruptures.
C. Is associated with pregnancy.
D. Is a feature of marfan’s syndrome.
E. Might reach its biforcation.

7. The long saphenous vein:??


A. Arises on the medial aspect of the sole of the foot.
B. Passes 1cm in front of the medial malleolus.
C. Passes in front the knee joint.
D. Enters the femoral sheath by piercing the fascia lata.
E. Related to the sural nerve

8. In deep venous thrombosis of the lower limb:????


A. One of the most common sites of origin is the short saphenous vein.
B. One of the common sites of origin is in the iliofemoral segment.
C. The diagnosis can usually be made by clinical examination.
D. Tender swollen thrombosed veins are usually palpable.
E. Usually associated with varicose vein.

9. The incidence of postoperative venous thrombosis can be reduced:(all correct except


one)
A. By raisin the foot of the operating table during surgery.
B. By passive calf contractions during an operation.
C. By administration of low molecular weight heparin during operation and on the first two
postoperative days.
D. By the prophylactic use of subcutaneous heparin.
E. By early ambutation.

10. Venous ulcers of the lower limb:


A. Are usually the result of longstanding varicose veins.
B. Commonly follow deep venous thrombosis.
C. Most commonly occur below the medial malleolus.
D. Will usually heal when firm bandaging is applied.
E. B&D only.

THYROID GLAND & NECK SWELLINGS


1. The best test to monitor adequacy of L-Thyroxine therapy is :
A. radioactive isotope thyroid scan
B. thyroglobulin blood level
C. total T4 blood level
D. free T3 blood level
E. blood level of TSH

2. Hashimoto s thyroiditis :
A. is of viral origin
B. presents usually by nodular goiter
C. can be diagnosed by thyroid antibodies and fine needle aspiration of thyroid
D. is often premalignant
E. must be treated always by surgery

3. Thyrotoxicosis usually manifests by all of the following EXCEPT:


A. loss of weight inspite of good appetite
B. a recent preference to cold
C. palpitation
D. more common in male than in female
E. excitability of the patient

4. Exophthalmos :??
A. is always present in thyrotoxicosis
B. is always bilateral
C. can cause diplopia in severe cases
D. decreases by hypothyroidism
E. is cured by surgery to the thyrotoxic goiter

5. Complications after thyroidectomy include all the following EXCEPT:


A. parathyroid insufficiency
B. recurrent laryngeal nerve paralysis
C. tracheomalacia
D. thyrotoxic crisis(storm) on operating on inadequitly prepared thyrotoxic patient
E. hypercalcemia

6. Regardig Carcinoma of the thyroid gland:


A. common diffrenciated types are the papillary and the follicular carcinoma
B. medullary carcinoma of the thyroid is treated only by resection of the involved lobe
C. papillary carcinoma never metastasize to cervical lymph nodes
D. anaplastic carcinoma is more common in the young age group
7. Tuberculous cervical lymphadenitis:
A. is caused by bovine tuberculous bacillus rather than the human bacillus
B. usually occurs through the tonsil of the corresponding side
C. collar-stud abscess is never a feature of T. B adenitis
D. is treated always by surgery
E. biopsy is not necessary for diagnosis

8. Branchial cyst:
A. develops from the third branchial cleft
B. presence of cholesterol crystals in aspirated fluid from the cyst is pathognomonic to it
C. should be treated with repeated aspiration
D. is not related to carotid vessles,hypoglossal and spinal accessory nerves
E. is never inflamed

9. Cystic hygroma:
A. develops as a sequestration of a portion of a jugular lymph sac
B. typically occupies the upper one third of the neck
C. occasionally may occur in cheek ,axilla and groin but exceptionally in tongue and
mediastinum
D. repeated aspiration is the only treatment
E. A and C are correct

10. Carotid body tumour:


A. is chromffin paragranuloma
B. is usually bilateral
C. is called chemodactoma or potato tumour
D. is situated ot the bifurcation of the carotid artery
E. C and D are correct

11. Differential diagnosis of lateral sided neck swelling:


A. cannot be diagnosed by fine needle asoiration
B. can be thyroglssal cyst
C. can be a lymphoma when having other characteristic features
D. cannot be diffrenciated into solid or cystic by ultrasound
E. is never a reactive lymphadenitis or secondary malignant disease

THE BREAST
12. The breast :
A. is a modified apocrine sweat gland
B. overlies the third to the sixth rib
C. is having usually more than 30 lacteferous ducts
D. is drained only by six groups of axillary lymph nodes/3
E. is having no attachment to skin

13. The commonest cause of bloody nipple discharge is :


A. mammary ductectasia
B. carcinoma of the breast
C. lactational mastitis
D. duct papilloma
E. fibrocystic disease of the breast

14. All of the following may be manifestations of breast carcinoma EXCEPT


A. peau d orange of covering skin
B. no mass may be felt clinically
C. enlargement of the ipsilateral axillary lymph nodes
D. cervical spine metastasis
E. none of the above

15. Bacterial mastitis:


A. never occurs in lactating mothers
B. is usually caused by streptococcus hemolyticus
C. is mostly caused by staphylococcus aureus
D. is treated only by incision and drainage
E. is the same as mastitis of puberty

16. Signs of malignancy on mammogram may include all of the following EXCEPT :
A. mass lesions with poorly defined irregular margins
B. fine stipped soft tissue with periductal and not vascular microcalcifications
C. thickening and retraction of the overlying skin
D. dysplastic ductal pattern
E. well circumscribed , homogenous, and often surrounded by a zone of fatty tissue

17. Early detection of carcinoma of the breast includes all of the following EXCEPT:
A. self-examination just after menstruation
B. screening mammography
C. frequent consultations among the high risk group patients
D. presence of fixed breast mass to chest wall with skin changes on clinical examination
E. A, B, C, are correct
18. Regarding Paget s disease of the nipple :
A. It is a benign condition
B. It is simply an eczematous lesion of the nipple
C. It is treated by excision of the nipple
D. It is usually diagnosed by biopsy of the suspected lesion
E. the areola and the surrounding skin are never involved

19. in a patient with fibroadenosis of the breast ;


A. cyst formation, adenosis, fibrosis,epitheliosis and papillomatosis are invariably present
B. is premalignant
C. pregnancy usually produces relief
D. presentation may include pain, nipple discharge, and/ or breast lump
E. all of the above are usually characteristics

20. Regarding gynaecomastia :


A. it is hypertrophy of female breast
B. it is hypertrophy of male breast
C. it may associate leprosy and liver failure
D. it may associate cimetidine, spironolactone, INH, or digitalis thrapy
E. B, C, and D are correct

21. Regarding modified radical mastectomy all are correct EXCEPT :


A. it is indicated recently for T1N0M0
B. pectoralis major muscle is excised
C. axillary clearance is mandatory
D. adjuvant radiotherapy must be given for all patients post-operatively
E. the long thoracic nerve of Bell (nerve to serratous anterior) must be preserved but nerve to
latismus dorsi might be sacrified

22. Mammary duct ectasia is characterized by the following EXCEPT :


A. is defiened as primary dilatation of major ducts of breast in middle aged women
B. may present with nipple retraction and Peau d orange picture
C. is treated usually by simple mastectomy
D. anaerobic superinfection commonly occurs in this recurrent periductal plasma cell
mastitis
E. is commonly pre-malignant

23. Regarding carcinoma of the breast :


A. invasive intraductal carcinoma is the commonest form
B. lobular carcinoma may present bilateral
C. clinical staging is always correct and definit
D. the medullary (anaplastic) type feels hard
E. A, and B, are correct

_____________________________________________________________________________

1. The radial nerve.


A. Is a branch of the anterior cord of the brachial plexus.
B. Is derived from the posterior primary rami of C5 to C7.
C. Supplies the flexors of the arm.
D. Gives rise to the anterior interosseous nerve.
E. Injury above elbow produces a classical wrist drop.

2. Regarding femoral canal all are true except:


A. Lies lateral to the femoral vein.
B. Has the inguinal ligament as its anterior border.
C. Has the lacunar ligament as its medial border.
D. Has the pectineal ligament as its posterior border.
E. Contains the lymph node of Cloquet.

3. The following causes hypercalcaemia except:


A. Sarcoidosis.
B. Primary hyperparathyroidism.
C. Acute pancreatitis.
D. Metastatic bronchial carcinoma.
E. Milk-Alkali syndrome.

4. Number of human chromosomes in human female are:


A. 23 pairs +XX.
B. 21 pairs +XY.
C. 22 pairs +XY.
D. 22 pairs +XX.
E. 23 pairs +XY.

5. Potassium deficiency should be suspected in all the following except:


A. In cases of paralytic ileus.
B. When the patient's reflexes are exaggerated.
C. If there is a decrease in height and peaking of the T waves of an ECG.
D. In alkalotic states.
6. Active immunization in case of tetanus:
A. Antitetanus human serum.
B. Gives short period of protection.
C. Given in case proved tetanus.
D. Use of immunoglobulin.
E. None of the above.

7. All of the following are signs of rised intracranial pressure except:


A. Headache.
B. Vomiting.
C. Papilledema.
D. Aphasia.
E. Bradycardia.

8. All of the following are extra cranial hematomas except:


A. Subcutaneos haematoma.
B. Extra dural haematoma.
C. Cephalohaematoma.
D. Subgaleal hematoma.
E. Subperiostial haematoma.

9. Glasgow coma scale all the following are true except:


A. Used for evaluation of comatose patient.
B. It ranges from ( 3 to 15).
C. Useful for neurological follow up.
D. Useful for pupils evaluation.
E. Best motor response given 6 point.

10. All the followings are Indications for central line insertion EXCPET:
A. Massive fluid replacement
B. Massive blood replacement
C. Measurement of central venous pressure
D. Prolonged Intervenes fluid therapy

11. Most common early complication of central venous line is:


A. Sepsis
B. Pneumothorax
C. Thoracic duct injury
D. Thrombosis

12. The following are Complications of shock:


A. Acute Respiratory failure
B. Acute myocardial infarction
C. Acute renal failure
D. A&C only
E. All the above

8:13
8:25
13. Causes of delayed union of fractures includes all the following EXCPET:
A. Compound fracture
B. Infection
C. Adequate immobilization
D. Poor blood supply

14. In Head injury C.T. scan is indicated in the following


A. Aphasia after the injury
B. Deterioration of level of consciousness
C. Skull fracture with persistent headache
D. A&B only
E. All the above

15. Most important steps in management of head injury include:


A. Prevent hypoxia
B. Prevent Dehydration
C. Assure Brain Metabolism
D. Prevent secondary brain injury
E. All the above

16. Tension pneumothorax


A. is the commonest type of chest injuries
B. Needs urgent X-Ray chest
C. Is a clinical Diagnosis
D. Causes flat neek viens
E. Treated by thoracotomy tube after chest X-ray.

17. Calcitonin hormon is secreted to the blood circulations from:


A. Parathyroid gland.
B. Parafollicular cells of thyroid gland.
C. Supra renal gland.
D. Pituitary gland.
E. Gonads.
18. Regarding tension pneumothorax, the first step in the management is:
A. Obtaining a stat chest x-ray.
B. Cricothyroidectomy.
C. Pass an endotracheal tube.
D. Starting oxygen by a valve-mask device.
E. Chest decompression needle.

19. The following are features of thyrotoxicosis except:


A. Weight gain.
B. Palpitations.
C. Proximal myopathy.
D. Increased skin pigmentation.
E. Pretibial myxoedema.

20. The following is a clinical feature of Horner's syndrome:


A. Miosis
B. Failure of abduction of the orbit
C. Increased sweating on the contralateral side of the forehead
D. Exophthalmos.
E. All are true

21. In role of nine extent of burn if entire trunk is burned it will be equal to:
A. 9% body surface area.
B. 18% body surface area.
C. 36% body surface area.
D. 27% body surface area.
E. 45% body surface area.

22. Trachlea (4th) cranial nerve supply :


A. Lateral rectus muscle of th eye.
B. Medial rectus muscle of the eye.
C. Superior obligue rectus muscle of the eye.
D. Superior oblique muscle of the eye.
E. Muscles of the upper eye lid.

23. Regarding pathological terms :


A. Hypertrophy is an increase in tissue size due to increased cell number.
B. Hyperplasia is an increase in tissue size due to an increase in cell size.
C. Atrophy is an increase in tissue size due to disuse.
D. Metaplasia is a change form one abnormal tissue type to another.
E. A hamartoma is a developmental abnormality.
24. Regarding nephroblastomas:
A. They are otherwise known as a Wilm's tumour.
B. Account for 10% of childhood tumours.
C. The commonest presentation is with an abdominal mass.
D. Most commonly present between 2nd and 4th year of life.
E. All are true.

25. Regarding fluid losses in a major burn all are true except:
A. Are maximal between 12 and 24 hours after the injury.
B. Are related to the age of the patient.
C. Are not related to the weight of the patient.
D. Are related to the area burnt.
E. Are not related to the burn duration.

26. In obstructive jaundice :


A. Urinary conjugated bilirubin is increased.
B. Serum unconjugated bilirubin is increased.
C. Urinary urobilinogen is increased.
D. Serum conjugated bilirubin is reduced.
E. Faecal stercobilinogen is increased.

27. Regarding Hydatid disease:


A. Is due to Ecchinococcus granulosa.
B. Man is an accidental intermediate host.
C. The liver is the commonest site of infection.
D. Can be diagnosed by the Casoni test.
E. All are true.

28. The first aid of treatment in fracture of cervical spine should be:
A. Cervical spine x-ray.
B. Analgesia.
C. Neck immoblization.
D. Cervical traction.
E. Non of teh above.

29. The first aid of treatment in fracture of cervical spine should be:
A. Cervical spine x-ray.
B. Analgesia.
C. Neck immoblization.
D. Cervical traction.
30. Regarding local anaesthesia:
A. Local anaesthetics act on small before large nerve fibres
B. Adrenaline reduces absorption and prolongs the local effects
C. Preparations containing adrenaline are safe to use on digits and appendages
D. Lignocaine has a longer duration of action than bupivicaine.
E. All are false.

31. Small bowel obstruction often results in: (all correct except one)
A. Hyperkalaemia.
B. Metabolic alkalosis.
C. Oliguria.
D. Hypovolaemia.
E. Severe dehydration.

32. A serious intra-abdominal injury in a comatose patient may be diagnosed by (all are
correct except one):
A. Abdominal paracentesis.
B. The observation of bruising pattern on the abdominal wall.
C. Falling of heamoglobin values.
D. The presence of marked abdominal distetion.
E. The presence of diarrhea.

33. A perforated duodenal ulcer, all are true except:???


A. Usually lies on the anterior or superior surface of the duodenum.
B. Usually presents with the acute onset of severe back pain.
C. Produces radiological evidence of free gas in the peritoneum in over 90 percent of the
patients.
D. Is usually treated by vagotomy and pyloroplasty.
E. Is usually treated conservatively.

34. Acute pancreatitis typically: (all correct except one)


A. Is accompanied by hypocalcaemia.
B. Produces paralytic ileus.
C. Is associated with a pleural effusion.
D. Produces pyloric stenosis.
E. Upper abdominal pain and vomiting.

35. Biliary colic typically:


A. Occurs 3 to 4 hours after meals.
B. Lasts 5 to 20 minutes.
C. Radiates from the upper abdomen to the right subscapular region.
D. Is made better by deep inspiration.
E. B&C only.

36. In post operative DVT, the following are true except:


A. Clinical DVT occures in the 4th post operative day.
B. If complicated by pulmonary embolism, it occures usually after the 7th post operative day.
C. The process of DVT starts preoperatively with the induction of anaesthesia .
D. When discovered we should start the patient on coumadin "Warfarin anticoagulation".
E. It may lead to chronic venous in suffering as a complication of DVT.

37. In acute appendicitis all of the following are true except:


A. Anorexia.
B. Abdominal pain usually preceedes vomiting.
C. Pain after begins in the paraumbilical region.
D. Constipation diarrhea may occur.
E. Dysuria excludes the diagnosis.

38. The most common cause of massive haemorrhage in the lower gastroinfestinal tract is :
A. Carcinoma.
B. Diverticulosis
C. Diverticulitis
D. Polyp.
E. Ulcerative colitis.

39. Painless hematuria is the leading presentation of :


A. Renal cell carcinoma.
B. Transitional cell carcinoma of the bladder .
C. Ureteric stone.
D. Pelvi-ureteric obstruction.
E. Ureterocele.

40. All of the following are complications of massive blood transfusion except:
A. Acute congestive heart failure.
B. Transmission of infection.
C. Hypercalcaemia.
D. Hyperkalaemia.
E. Transfusion reactions.

41. Complication of undescended testis include all of the following except :


A. Malignant degeneration.
B. Increased susceptability to trauma.
C. Increased spermatogenesis.
D. More liable to testiculer torsion.
E. Psychological complication.

42. The recurrent laryngoeal nerve is branch of :


A. Facial nerve.
B. Glosso-pharyngeal nerve.
C. Cervical plexus.
D. Vagus nerve.
E. Brachial plexus.

43. The thyroid tumor which is may be associated with pheochromocytoma is :


A. Papillary carcinoma.
B. Medullary carcinoma.
C. Follicular carcinoma.
D. Anaplastic carcinoma.
E. Malignant lymphoma.

44. The most common pancreatic cyst is :


A. Dermoid cyst of the pancreas.
B. Hydatid cyst of the pancreas.
C. Pancreas pseudocyst.
D. Pancreatic cystadenoma.
E. Congenital cystic disease of the pancreas.

45. The anatomical division between the anus and rectum :


A. Lateral haemorrhoidal groove.
B. Inter haemorrhoidal groove.
C. Dentate line.
D. Arcuate line
E. Ano-rectal ring.

46. The comments type of Anorectal abscess is:


A. Ischio rectal
B. Perianal
C. Submucons
D. Pelvirectal

47. Anal Fissure:


A. Usually anterior
B. May be caused by previous anal surgery
C. Can cause dark bleeding PR.
D. Sometimes is painful
E. Treated by steroids

____________________________________________________________________________

1. In the preoperative assessment ,which of the following is not suggestive of a platelet


disorder?
(A) Epistaxis
(B) Gingival bleeding
(C) Hemarthrosis
(D) Easy bruising
(E) Petechia

2. Which of the following is a physical finding specific for vascular structures?


(A) Reducibility
(B) Fluctuation
(C) Transillumination
(D) Compressibility
(E) Crepitancy

3. Which of the following medications must be stopped preoperatively:


(A) Atenolol
(B) Thyroxine
(C) Albuterol
(D) Aspirin
(E) Enalapril

4. A patient has undergone an ileal resection. Which of the following conditions would be the
least likely to develop?
(A) Alopecia
(B) Megaloblastic anemia
(C) Neurological symptoms
(D) Steatorrhea
(E) Cholelithiasis

5. Which of the following is the best method to assess the perioperative pulmonary risk?
(A) Arterial blood gas
(B) Pulmonary function test
(C) CXR
(D) Detailed history & physical exam
(E) Diffusion capacity study

6. Which of the following is the least important data/test to obtain for the preoperative risk
assessment of coagulopathy?
(A) Bleeding time
(B) History& physical examination
(C) Prothrombin time
(D) liver function test
(E) Fibrinogen levels

7. The most consistent physical sign of hyperthyroidism is:


(A) Tachycardia
(B) Exophthalmus
(C) Excessive sweating
(D) Confusion
(E) Hyperperistalsis

8. Which of the following would be the most difficult to manage preoperatively?


(A) Chronic renal failure
(B) Hemophilia
(C) COPD
(D) Heart failure
(E) Advanced cirrhosis

9. Which of the following is the least likely to develop in hospitalized surgical patients?
(A) DVT
(B) Pneumonia
(C) Surgical wound infection
(D) Systemic candidiasis
(E) Urinary tract infection

10. When should parenteral antibiotics be given before mastectomy?


(A) the night before
(B) 6 h prior to surgery
(C) Not indicated in such a case
(D) at the time of incision
(E) 30 min after incision

11. A patient with a postoperative wound cellulitis should be treated with:


(A) Antibiotics and warm soaks to the wound
(B) Incision & drainage
(C) Antibiotics and opening the wound
(D) Antibiotics alone
(E) observation

12. Which of the following is the most likely cause of fever in the first 2 days postoperatively?
(A) Malignant hyperthermia
(B) wound abscess
(C) Urinary tract infection
(D) Pulmonary complications
(E) drug fever

13. Neurogenic shock is characterized by all of the following except:


(A) Warm skin
(B) Bradycardia
(C) Vasoconstriction
(D) Normal blood volume
(E) Good response to vasopressors

14. A 24-year-old man is brought into the emergency department after a fall from 4 meters
height. His breathing is distressed, and he is cyanotic. No breath sounds can be heard in the
right lung field, which is resonant to percussion. Chest tube was inserted. The next step in his
management should be:
(A) Mechanical ventilation
(B) Surgical intervention
(C) IV fluid replacement
(D) Passing an oral endotracheal tube
(E) Obtaining an urgent chest X-ray

15. Which of the following is not a cause of cardiogenic/cardiac compressive shock in a


trauma patient?
(A) Air embolism
(B) Tension pneumothorax
(C) Flail chest
(D) Cardiac tamponade
(E) Myocardial contusion

16. The severity of hypovolemic shock has been found to correlate with the
(A) hematocrit
(B) pulmonary capillary wedge pressure (PCWP)
(C) The percentage of volume lost
(D) PaO2
(E) white blood cell count

17. A patient is transported to the ER following a fall from height . The Glasgow Coma Scale
(GCS) can categorize the patient's neurologic status by assessing all of the following except:
(A) Withdrawal to pain
(B) Inappropriate words
(C) Eye opening response
(D) pupil response to light
(E) Obeying commands

18. Neurogenic shock can be caused by all of the following except:


(A) Spinal anaesthesia
(B) Isolated head injury
(C) Extremely severe pain
(D) Accidental back trauma
(E) Unpleasant scenes

19. Which of the following types of shock is characterized by a higher incidence of delayed
mortality?
(A) hypovolemic
(B) cardiogenic
(C) neurogenic
(D) cardiac compressive
(E) septic

20.What is the first priority in the management of trauma patients ?


(A) continued intravenous hydration
(B) Oxygen mask
(C) Chest tube
(D) Airway & C-spine control
(E) Blood transfusion

21. In physical examination if an inguinal hernia is completely irreducible it is called


(A) Strangulated
(B) obstructed
(C) Recurrent
(D) Fixed
(E) Incarcerated

22. Which of the following is not a characteristic of uncomplicated inguinal hernia


(A) Reducible
(B) Expansile
(C) May be congenital or acquired
(D) Pulsatile
(E) May be caused by chronic cuogh
23. The extension of the existing pain to a site other than the original one is called:
(A) Radiation
(B) Shifting
(C) migration
(D) Exacerbation
(E) All of the above

24. All of the following symptoms and signs are suggestive of acute pancreatitis EXCEPT
(A) Anorexia
(B) Severe epigastric pain
(C) Pain is colicky in nature
(D) Vomiting
(E) Ileus

25. A 42-year-old female comes to the ER complaining of right upper quadrant abdominal
pain for the last 36 h, associated with fever up to 39°C, bilious vomiting, and jaundice.
Direct bilirubin 2.2, alkaline phosphatase 450, WBC 19,000.
What is the best cost- effective imaging study to define the etiology?
(A) HIDA scan
(B) Ultrasound
(C) CT
(D) MRI
(E) Plain abdominal x-ray

26. The diagnosis of appendicitis depends mainly on :


(A) CBC
(B) Ultrasound
(C) Clinical assessment
(D) CT scan
(E) None of the above

27. The risk of which of the following infections is markedly increased after splenectomy?
(A) Candidiasis
(B) Pneumococcal pneumonia
(C) Cytomegalovirus
(D) E.coli
(E) All of them

28. In breast examination dimpling of the overlying skin is most likely due to:
(A) Breast abscess
(B) Mastitis
(C) Cancer
(D) Fibroadenoma
(E) None of the above
29. Melena may be caused by which of the following?
(A) Diverticular disease
(B) Hemorrhiods
(C) Peptic ulcer
(D) Rectal cancer
(E) Anal fissure

30. All of the following increase the risk of breast cancer except?
(A) Estrogen containing oral contraceptives
(B) Early menarche
(C) Prolonged and multiple episodes of lactation
(D) Late menopause
(E) The presence of family history of breast cancer

_____9:13_____________________________________________________________________
__

Choose only the ONE most Appropriate Answer

1:31
1. Which of the following is not true about neurogenic shock?
a. It may follow spinal fractures.
b. There is tachycardia.
c. The extremities are warm.
d. There is bradycardia and hypotension.
e. Vasopressors may be useful.

2. All the following are complications of massive blood transfusion except:


a. Hypothermia.
b. Hypocalcaemia.
c. Hypokalaemia.
d. Acidosis.
e. DIC.

3. In idiopathic thrombocytopenic purpura all the following are present except:


a. Megakaryocytes are present in the bone marrow.
b. Cortisone therapy may improve the condition.
c. Marked enlargement of the spleen is present.
d. Haemarthrosis is not a complication.
e. The INR is normal.
f. The disease is due to the presence of immunoglobulins.

4. Regarding hyperkalemia all are true EXCEPT:


a. Is mostly the result of renal failure.
b. Is common with massive tissue destruction.
c. Muscle rigidity is a manifestation of severe hyperkalemia.
d. Causes a peaked T-wave on the electrocardiogram.
e. Urgent treatment is by Ca gluconate and Na bicarbonate.

5. One of the following is an anaerobic organism:


a. Staphylococcus aureus.
b. E. coli.
c. Klebsiella.
d. Pseudomonas aeroginosa.
e. Bacteriodes.

6. Which of the following tests will be affected after intake of non-steroidal anti-
inflammatory drugs:
a. Coagulation time.
b. PTT.
c. INR.
d. Bleeding time.
e. Thrombin time.

7. All of the following can be treated conservatively in a stable trauma patient except:
a. Lung contusion.
b. Liver laceration.
c. Kidney laceration.
d. Splenic hematoma.
e. Perforation of the small intestine.

8. The initial maneuver to establish an airway in a patient with multiple injuries is:
a. Oropharyngeal airway.
b. Uncuffed endo-tracheal tube.
c. Suctioning foreign debris and lifting up the mandible.
d. Cuffed endo-tracheal tube.
e. Tracheostomy.

9. Which is the most commonly injured intra-abdominal organ in blunt trauma?


a. Pancreas.
b. Kidney.
c. Spleen.
d. Stomach.
e. Colon.
10. In malignant melanoma which of the following correlates best with the prognosis?
a. Age of the patient.
b. Gender.
c. Clark’s level.
d. Site of the lesion.
e. Breslew thickness of the lesion.

11. Risk factors for breast cancer include the following except:
a. Prior breast cancer.
b. Mammary duct ectasia.
c. Breast cancer has affected a sister.
d. Early menarche and late menopause.
e. In situ lobular or duct carcinoma.

12. The commonest histological type of breast cancer is:


a. Duct carcinoma in situ.
b. Lobular carcinoma in situ.
c. Invasive duct carcinoma.
d. Invasive lobular carcinoma.
e. Paget’s disease of nipple.

13. What is the commonest presentation of a nephroblastoma?


a. Abdominal pain.
b. Haematuria.
c. Fever.
d. Abdominal mass.
e. Loss of weight.

14. All the following statements about follicular thyroid cancer are true except:
a. It present at a later age than papillary carcinoma.
b. It disseminates via hematogenous route.
c. It is less common than papillary carcinoma.
d. It is frequently multicentric.
e. Bone is a favorite metastatic site.

15. To prevent foot infection in a diabetic patient the following recommendations are all
correct except:
a. Careful trimming of toe nails.
b. Monthly injection of the long-acting benzathene penicillin.
c. Regular washing and daily inspection of the feet by the patient.
d. Early treatment of taenia pedis infection.
e. Avoidance of walking bare-footed.

16. Which of the following tests is used to monitor the adequacy of anticoagulation with
warfarin?
a. Bleeding time.
b. Clotting time.
c. Activated partial thromboplastin time (APTT).
d. Fibrinogen level.
e. International normalized ratio (INR).
f. Marjolin’s ulcer.

17. Severe limb pain of sudden onset can be caused by all the following conditions except:
a. Acute ischaemia.
b. Deep venous thrombosis.
c. Muscle tear.
d. Sciatica.
e. Bone fracture.

18. The following statements about gall bladder stones are all correct except:
a. Most of these stones are radio-opaque.
b. In many cases these stones produce no symptoms.
c. Gall stones are present in most cases of gall bladder cancer.
d. Ultrasonography is the preferred imaging study.
e. Removal of the gall balder is the standard treatment of symptomatic cases.

19. The following statements about acute cholecystitis are all correct except:
a. The great majority of cases are caused by stones.
b. The condition is more serious in diabetics.
c. Differential diagnosis includes acute pancreatitis.
d. The main presentation is jaundice.
e. Initial treatment may be conservative.

20. Which of the following is a common cause for massive lower gastrointestinal bleeding?
a. Crohn’s colitis.
b. Rectal polyp.
c. Angiodysplasia of the colon.
d. Carcinoma of the caecum.
e. Solitary rectal ulcer.

21. Which of the following is not true about achalasia of the cardia?
a. It usually occurs in middle age.
b. There are hyyperperistlatic waves in the body of the esophagus.
c. It predisposes to squamous cell carcinoma of the esophagus.
d. Manometric study is diagnostic.
e. Pneumatic dilatation is a line of treatment.

22. Organisms found in the large bowel are mostly:


a. Gram-positive cocci and anaerobes.
b. Gram-negative cocci and anaerobes.
c. Gram-negative bacilli and anaerobes.
d. Gram-positive bacilli and anaerobes.
e. Gram-negative cocci only.

23. Which of the following operations is considered as clean contaminated operation?


a. Herniorrhophy.
b. Axillary block dissection.
c. Ureterolithotomy.
d. Urgent colectomy.
e. Appendicular abscess drainage.

24. During conservative treatment of a patient with adhesive intestinal obstruction, which
of the following is an indication to proceed to surgery?
a. Nasogastric suction of more than 2 L/24 hours.
b. Exaggerated bowel sounds.
c. Abdominal rigidity.
d. Abdominal distension.
e. Serum K level below 2.5 mEq/L.

25. The most important prognostic index in breast cancer is:


a. Age of the patient.
b. Site of the lesion.
c. Presence of lymph node metastases.
d. Level of CEA.
e. Presence of micro-calcification.

26. Which of the following are not found in peritonitis?


a. Patient is lying still
b. Guarding
c. Rebound tenderness
d. Hyperactive bowel sounds
e. Rigid abdomen

The following is an indication for thoracotomy in chest .28


,injury

a. Cardiac tamponade
b. Uncontrolled pulmonary air leakage
c. Perforation of thoracic esophagus
d. Blood loss of 200ml/hr for 2-3 hrs via chest tube
e. All of the above

Pancreas
1. In pancreas divisum all the following are true except
A. occur in 5% of the population
B. may lead to recurrent pancreatitis
C. duct of Wirsung is the main duct
D. there is failure of fusion
E. diagnosed by ERCP

2. Cystic fibrosis all the following are true except


A. transmitted as autosomal dominent
B. dysfunction of exocrine glands
C. there is poor appetite & growth,
D. patient has distension, steatorhea, finger clubbing present
E. diagnosis by DNA analysis

3. Sever attack of pancreatitis is??


A. attack followed by pseudocyst
B. has four points on Ranson criteria
C. has 5 points on Balthazar grade
D. all the above
E. none of the above

4. Antibiotics in acute pancreatitis:


A. not indicated
B. second generations cephalosporins are the best
C. indicated in all cases
D. b and c only
E. none of the above

5. CT in acute pancreatitis :
A. should be done on admission
B. should be done 48 hours later
C. should be done one week later
D. not indicated
E. none of the above
6. Pancreatic pseudocyst
A. usually appears after two weeks of the attack
B. has one layer of epithelial lining
C. may be treated by endoscopy
D. all the above
E. none of the above

7. Percutaneous drainage of pancreatic cyst is indicated in all the following except :


A. after three weeks in order to mature the wall
B. rapidly enlarging cyst
C. difficult sites
D. presence of infection
E. unfit patients

8. The following are known complications of acute pancreatitis except


A. respiratory failure
B. renal failure
C. cardiac failure
D. coagulopathy
E. bleeding gastric erosion

9. The following can cause high serum amylase :


A. perf. GB
B. rupture ectopic pregnancy
C. rupture abdominal aortic aneurysm
D. afferent loop obstruction
E. all the above

10. Carcinoma of the pancreas:


A. 50% adeocarcinoma of duct origin
B. slightly more in the head
C. 10% cystadenocarcioma
D. CIS about 40 %
E. none of the above

11. Insolinoma :
A. age usually above 40
B. hyper glycemia in the early morning is classical
C. clinically may simulate DU
D. insulin / glucose ratio should equal one
E. all of the above

Orthopedic

12. Bennet's fracture is :


A. fracture of the base of first meta tarsal bone
B. fracture of the base first meta carpal bone
C. fracture of the head of first meta tarsal bone
D. fracture of the head first meta carpal bone
E. fracture of the head second meta carpal bone

13. Thomas test is positive in :


A. Paget's disease of the hip
B. Perthe's disease of the hip
C. osteo arthritis of the knee
D. Sudeck's atrophy of the shoulder
E. none of the above

14. All the following are complications of cast except


A. compartment syndrome
B. pressure sore
C. blisters with 2ry infection
D. immobilization of joints
E. crush syndrome

15. Volkmann's ischemic contracture :


A. may follows fracture of radius and ulna
B. fracture neck of femur
C. supracondylar fracture of humerus
D. all the above
E. none of the above

16. Compartment syndrome


A. common in fracture of the leg
B. passive flexing of the toes increase the pain
C. compartment pressure should exceed 50 mmHg
D. treated by urgent fasiectomy
E. all the above
17. Indication of operative treatment in fractures :
A. compound fracture
B. intertrochanteric fracture of femur in the elderly
C. associated complications
D. a and c only
E. all the above

18. The following are methods of stabilization of fractures except:


A. External splint e.g. POP
B. Internal fixation
C. Hanging plaster
D. Skin traction
E. a and b only

19. stress fracture :


A. may affect the shaft of tibia
B. may affect the second metatarsal bone
C. occur in marathon runners
D. all the above

20. Mallet finger is due to:


D. avulsion fracture of the base of terminal phalanx

21. operative approach to the hip joint may be :


A. anterior
B. posterior
C. lateral
E. all the above

22. Salter and Harris classification :


A. classification for fracture tibial platue
B. classification for fracture neck femur
C. classification for fracture growth plate

VASCULAR
23. In critical ischemia there is:
A. rest pain
B. oedema
C. color changes
D. hyperesthesia
E. all the above

24. popliteal artery aneurysm


A. accounts for 70% of all peripheral aneurysms
B. 25% are bilateral
C. 30% develop complication within 5years
D. All of the above

25. Beurger' diseases has :


A. occlusion of small and medium arteries
B. thrombophlebitis
C. Reynold's phenomenon
D. all the above

26. abdominal aortic aneurysm


A. usually due to atherosclerosis
B. usually at the level of L1
C. incidence of rupture is 50% for 5 cm aneurysms
D. elective surgery careies 20% mortality rate
E. all the above

27. Berry's aneurysms


A. are congenital
B. affect medium sized arteries
C. fusiform in shape
D. all the above

28. Secondary varicose viens


A. due to valve destruction
B. due to incompetent perforators
C. has lower complication rate than the primary varicose
D. may lead to venous ulcer
E. none of the above

29. Greenfield filter?????????


A. reduce incidence of DVT
B. inserted in the common iliac vien >> jugular or femoral
C. inserted between L1 and L2 levels >> opposite L2 “infrarenal”
D. can be inserted percutaneous

30. lymph edema tarda


A. usually before the age of 35 years
B. affects upper limbs only
C. affect lower limbs only
D. usually pitting in early stage

BASIC SURGICAL SCIENCES

31. Abduction of the vocal cords results from contraction of the:


A. crico-thyroid muscles
B. posterior crico-arytenoid muscles
C. vocalis muscles
D. thyro-ary-epiglottic muscles
E. lateral crico-arytenoid and transverse arytenoids muscles

32. The rectum:


A. is devoid of peritoneum
B. is surrounded by peritoneum
C. has peritoneum on its lateral surfaces for its upper two- thirds, and on its anterior surface
for its upper one-third
D. has pritoneum on its anterior surface for its upper two-thirds, and on its lateral surfaces for
its upper one-third
E. has peritoneum on its anterior surface only

33. The umbilicus:


A. lies near the to the xiphoid than to the pubis
B. derives its cutaneous innervation from the eleventh thoracic nerve
C. transmits, during development, the umbilical cord two arteries and two veins
D. usually lies at about the level between the third and fourth lumbar vertebra
E. emberiologicall, may transmit urine but never bowel contents

34. The superficial perineal pouch:


A. is limited inferiorly by the urogenital diaphragm
B. is not continuous with the space in the scrotum occupied by the testes
C. has a membranous covering which provides a fascial sheath around the penis
D. is traversedby the urethera in the male but not the urethera and vagina in the female
E. in the female, the greater vestibular glands are situated outside this pouch

35. The tongue:


A. has a foramen caecum at the base of the frenulum
B. is separated from the epiglottis by the valleculae on each side of the midline
C. has 7-12 circumvallate papillae situated behind the sulcus terminalis
D. is attached to the hyoid bone by the genioglossus muscle
E. is supplied only by hypoglossal nerve

36. Hypovolaemic shock is characterized by:


A. a low central venous pressure , low cardiac output , low peripheral resistance
B. a high central venous pressure , high cardiac output , low peripheral resistance
C. a low central venoys pressure , low cardiac output , high periphera resistance
D. a low central venous pressure , high cardiac output , high peripheral resistance
E. a high central venous pressure , low cardiac output , low peripheral resistance

37. An oxygen debt is:


A. the amount of oxygen in excess of the resting metabolic needs that must be consumed
after completion of exercise
B. build up because the pulmonary capillaries limit the uptake of Oxygen at high rates of
oxygen consumption
C. related to the fact that skeletal muscle cannot function temporarily in the absence of
oxygen
D. associated with a decrease in blood lactate
E. associated with alkalosis

38. Pulmonary embolism may be a complication of except :


A. prolonged bed rest
B. a surgical operation
C. vitamine K deficiency
D. oral contraceptive therapy
E. Antithrombin III deficiency

39. Which of the following statements regarding potassium metabolism is NOT True?:
A. potassium deficiency commonly results from thiazide diuretic theraoy
B. the normal compensation for potassium deficiency is a metabolic extracellular acidosis
C. aldosterone increases urinary potassium loss
D. hyperkalaemia causes bradycardia and loss of P waves on the ECG
E. hypokalaemia aggrevates the cardiac effects of digitalis toxicity

40. Cutaneous pain:


A. is due to overstimulation of receptors serving other sensory modalities
B. cannot be elicited more readily if the tissue has recently been injured
C. is due to exitation of receptors by pain-producing chemical substances in the injured tissue
D. shows marked adaptation, i. e. decrease in severity in response to a constant stimulus
E. is conducted through the medial spinothalamic tract
41. Which of the following is NOT associated with hyperthyroidism?:
A. increase size of the thyroid gland
B. increased amount of colloid in thyroid follicle
C. increased height of epithelium of the thyroid follicle
D. increased vascularity of the thyroid gland
E. increased uptake of iodine by the thyroid gland

42. Sarcomata may show all of the following EXCEPT:


A. production of myxomatous tissue
B. production of collagen
C. spindle shaped cells
D. signet ring cells
E. blood stream metastasis

43. Anaphylaxis is characterized by all of the following EXCEPT:


A. is a reaction either local or general , frequently occurs within five minutes
B. causes an urticarial eruption
C. is produced by IgA antibody
D. causes eosinophilia
E. causes degranulation of basophils and mast cells

44. Autoimmunity is characterized by the following EXCEPT:


A. occurs because of a breakdown in the ability of the body to distinguish between self and
non self
B. is involved in some forms of orchitis
C. is involved in formation of cryo globulin
D. is important in the pathogenesis of lupus erytheromatosus
E. does not result in immune complex disease

45. Pseudomembranous enterocolitis is caused by the following organisms:


A. Clostridium sporogenes
B. Clostridium defficile
C. Streptococcus faecalis
D. Penicillin sensitive staphylocci
E. Pseudomonas aeruginos

CLINICAL SUGERY

46. The “white clot syndrome”:


A. is usually characterized with antithrombin III deficiency
B. most often present with arterial complications of heparin induced throbocytopenin
C. is best managed by low molecular weight dextran
D. is best managed by halving the therapeutic dose of heparin sodium
E. results from nitric oxide deficiency of endothelial cells

47. A 21-year-old man who was the driver in a head-on collision has a pulse of 140/min ,
respiratory rate of 36 and blood pressure of 75 palpable. His trachea is deviated to the left,
with palpable subcutaneous emphysema and poor breath sounds in the right hemithorax,
The most appropriate initial treatment must be
A. immediate thoracotomy
B. catheter insertion in the subclavian vein for fluid resuscitation
C. intubation and ventilation
D. tube thoracostomy
E. immediate tracheostomy

48. The best test to monitor the adequacy of levothyroxin therapy is:
A. radioactive iodine uptake
B. thyroglobulin
C. free thyroxine index (T4)
D. triiodothyronine resin uptake (T3)
E. thyroid stimulating hormone (TSH)

49. Which of the following statements about fungal infection is NOT true ?:
A. Prior or synchronous culture positive for Candida at another site occurs in few patients
with candidimia
B. For critically ill patients nonhaematogenous sites of candida are appropriately treated with
systemic antifungal therapy
C. Mortality rates are similar regardlss of whether C. albicans fungmia is treated with
amphotericin B or flconazole
D. Intravenous catheters and the gastrointestinal tract are common portals for Candida to gain
blood stream access
E. Septic emboli are more common with fungal endocarditis than with bacterial endocarditis

50. The maximum safe dose of local anaesthetic administered subcutaneously in a 70-kg
man is:
A. 10 to 20 ml of 1% lidocaine
B. 40 to 50ml oh 2% lidocaine with epinephrine
C. 40 to 50 ml of 1% lidcaine with epinephrine
D. 40 to 50 ml of 1% bupivacaine (marcaine)
E. 40 to 50 ml of 1%lidocaine without epinephrine
51. Two days after right hemicolectomy for a Dukes B caecal carcinoma , the Patient
complains of sharp right-sided chest pain and dyspnea. HisPaO2 Is 64mmHg ,his PaCo2 is
32mmHg. CVP is 26 cm water, and the blood pressure is 102/78mmHg. A pulmonary
embolus is suspected, The next step in management should be:
A. A ventilation- perfusion lung scan
B. A pulmonary arteriogram
C. Postrioanterior and lateral chest x-rays
D. Heparin sodium ,100 units/kg intravenously
E. Immediate duplex scanning of both lower extremities

52. The major cause of graft loss in heart and kidney allograft is:
A. acute rejection
B. hyperacute rejection
C. vascular thrombosis
D. chronic rejection
E. graft infection

53. All of the following are indicators of tumor aggressiveness and poor outcome for
papillary carcinoma of the thyroid gland EXCEPT:
A. age over 50 years
B. microscopic lymph node metstasis
C. tumor larger than 4 cm
D. poorly differentiated histological grade
E. invasion through capsule to adjacent tissues

54. A 40-year-old woman has extensive microcalcifications involving the entire upper
aspect of the right breast. Biopsy shows a commedo pattern of intraductal carcinoma.The
most appropriate treatment is :
A. wide local excision
B. radiation therapy
C. wide local excision plus radiation therapy
D. right total mastectomy
E. right modified radical mastectomy

55. In the conventional ventilator management of acute adult respiratory distress syndrome
(ARDS) , arterial O2 saturation is maintained above 90% by all the following EXCEPT :
A. increasing the ventilatory rate
B. the use of positive end-expiratory pressure (PEEP)
C. increasing mean airway pressure
D. increasing tidal volumes
E. increasing FiO2

2:31
3:21
56. Which of the following statements about patients with abdominal compartment
syndrome is NOT true ?
A. Abdominal pressure is usually measured indirectly through inferior vena cava
B. Multiple contributing factors are commonly responsible
C. The chief manifestations are reflected in central venous pressure , ventilatory function,
and oliguria
D. Decopression of the abdomen is required to resverse the syndrome
E. Aggressive hemodynamic monitoring and management is required when the abdomen is
opened

57. The most appropriate treatment for histologically malignant cystadenoma phylloides
is :
A. total mastectomy without axillary node dissection
B. total mastectomy with axillary node dissection
C. wide margin (3) cm excision of the lesion
D. post operative hormonal manipulation
E. postoperative adjuvant chemotherapy

58. Deep venous thrombosis resulting from upper extremity central venous lines:
A. should be treated with catheter removal, heparin therapy, and long term anticoagulants
B. is best with urokinase through the catheter
C. is innocuous and self limiting, and best treated with catheter removal only
D. is best treated with low-dose warfarin (coumadin, 1 mg / day) , without catheter removal
E. is best managed by single systemic dose of low molecular weight heparin daily and
continued catheter use

59. Emergency surgery is indicated for all of the following complications of ulcerative
colitis EXCEPT:
A. colonic dilatation greater than 12 cm (toxic mega colon )
B. free perforation
C. complete intestinal obstruction
D. intractable haemorrhage
E. abscess formation

60. All the following statements concerning carcinoma of the oesophagus are true EXCEPT
that:
A. it has a higher incidence in males than females
B. alcohol has been implicated as a precipitating factor
C. adenocarcinoma is the most common type at the cardio esophageal junction
D. it occurs more commonly in patients with corrosive oesophagitis
E. surgical excision is the only effective treatment

61. Which of the following statements about pathology encountered at esophagoscopy is/are
correct?
A. Reflux esophagitis should be graded as mild, moderate, or severe, to promote consistency
among different observers.
B. An esophageal reflux stricture with a 2-mm lumen is not dilatable and is best treated with
resection.
C. A newly diagnosed radiographic distal esophageal stricture warrants dilation and
antireflux medical therapy.
D. In patients with Barrett's mucosa, the squamocolumnar epithelial junction occurs 3 cm. or
more proximal to the anatomic esophagogastric junction.

Skin And Breast


1- Malignant melanoma:
• Occur more commonly in the black races.
• Occur with equal frequency in all ages.
• Frequently arise from pre-existing benign naevi.
• Occur more frequently in non tropical regions.
• A&B only.

2- A malignant melanoma:
• Frequently arises from hair-bearing naevi.
• Frequently arises from junctional naevi.
• Has a worse prognosis when it arises on the leg.
• Should be suspected in any big pigmented lesion.
• Non of the above is correct.

3- The treatment of a malignant melanoma should include: ( All correct except one)
• A preliminary incision biopsy.
• Wide excision of the tumour.
• ‘En bloc’ removal of adjacent involved lymph nodes.
• Immediate excision of any enlarging lymph node in the postoperative period.
• Excision of 5cm margin of normal tissue.

4- Squamous cancer of the lip:


• Is most common in early adult life.
• Is more common in fair skinned subjects.
• Metastasises readily by the blood stream.
• Is preferably treated by radiotherapy once lymph node deposits are present.
• All of the above are correct.

5 - Basal cell carcinomas:


• Usually metastasise to regional lymph nodes.
• Are less common than squamous cell carcinomas.
• Are characterised histologically by epithelial pearls.
• Are particularly common in oriental races.
• Non of the above is correct.

6 – Capillary angiomas of childhood (strawberry naevi):


• Arise in the dermis.
• Are premalignant.
• Are most satisfactorily treated with superficial radiotherapy.
• Should be surgically excised.
• B&C only.

7- Fibroadenoma of the breast:


• Are commonest in early adult life.
• Are indiscrete and difficult to distinguish.
• Are usually painless.
• Resolve without treatment.
• A&C only.

8 –Fibrocystic disease of the breast:


• Is a variant of the normal cyclical changes that the breast undergoes during menstruation.
• Is normally unilateral.
• Tends to progress in the post-menopausal years.
• Is precancerous.
• C&D only.

9 –The management of fibrocystic breast disease should:


• Usually be by surgical excision.
• Include mammography when available.
• Include therapy with oestrogens.
• Include therapy with progesterone.
• Corticosteroids.

10 – An intradut papilloma of the breast:


• May cause a colourless nipple descharge.
• May be diagnosed with the aid of contrast radiography.
• Should be treated by simple mastectomy.
• Is associated with fibrocystic disease of the breast.
• Is usually a premalignant lesion.

11 – Paget’s disease of the nipple:


• Usually presents as a bilateral eczema of the nipple.
• Is always related to an underlying breast cancer.
• Indicates incurable breast cancer.
• Has non-specific histological characteristics.
• A&C only.

12- x-ray examination of the breast (mammography):???


• Does not improve the clinician’s diagnosis rate of benign and malignant breast disease.
• Is diagnostically most useful in young women.
• Is practical as a nationwide presymptomatic screening procedure.
• Contributes nothing to the management of the patients with clinically obvious breast
cancer.
• Non of the above is correct.

13- Breast cancer:


• Is one of the commonest female neoplasm.
• Has its highest incidence in social class v.
• Has a familial tendency.
• Is less common in nulliparous women.
• A&C only.

14 – Breast cancer: (all correct except one)


• May be presents with a history of breast pain.
• Is most common in the upper outer quadrant of the breast.
• Can be diagnosed preoperatively by the experienced clinician in 95 per cent of cases.
• Must be considered on discovering any discrete mass in the breast.
• Usually presents as a painless lump.

15 –The signs and symptoms of breast cancer include: (all correct except one)
• A bloody nipple discharge.
• Eczematous changes in the nipple and areola.
• Pre-menstrual breast pain.
• Skin tethering.
• A discrete breast lump.

16 – The histological study of breast cancers has shown that:


• The prognosis is not related to histological type.
• The commonest carcinoma is a squamous carcinoma.
• Most breast cancers arise from the epithelium of the breast duct.
• Satellite breast cancers are uncommon.
• B&D only.

17- Signs of incurable breast cancer include:


• Tumour fixity to the chest wall.
• Skin ulceration.
• Palpably enlarged mobile ipsilateral axillary lymph nodes.
• A bloody nipple discharge.
• A&B only.

18– The prognosis of treated breast cancer:


• Is related to the clinical staging of the cancer.
• Is not related to the number of axillary nodes found to be invaded by cancer at operation.
• Is related to the use of postoperative radiotherapy to the regional nodes and operative
field.
• Is better after simple rather than radical mastectomy.
• Non of the above is correct.

19 – The prognosis of treated stage 1 breast cancer:


• Cannot be satisfactorily assessed until 15 years have elapsed.
• Is adversely affected by a subsequent pregnancy.
• Is worse in the male.
• Is worse if the cancer is discovered during late pregnancy or the puerperium.
• A&D only.

Colon, Rectum And Anus


1- Diverticular disease of the colon:
• Is usually asymptomatic.
• Often presents with lower abdominal pain.
• May present with severe rectal haemorrhage.
• May present with peritonitis.
• All of the above are correct.

2- Ulcerative colitis:
• Is more common in female than males.
• Appears most commonly between the ages of 20 and 30.
• Usually presents with abdominal discomfort and diarrhoea.
• Can usually be diagnosed on sigmoidoscopic examination.
• All of the above correct.

3- Crohn’s disease: ( all correct except one)


• Is associated with an anal fissure or fistula in the majority of cases.
• May produce intestinal obstruction
• May involve the colon and the ileum.
• Is characterised by long periods of remission.
• May produce pritonitis.

4-Regarding ischaemic colitis: (all correct except one)


• Often presents with diarrhoea.
• Often presents with rectal bleeding
• Is commonly setuated around the splenic flexure.
• Is often effectively managed by non-surgical means.
• The baruim enema may reveal localised bowel oedema or stricture.

5- A volvulus of the colon: (all correct except one)


• Most frequently occurs in eastern Europe and Africa.
• Most frequently occurs on the right side and includes the caecum.
• Usually produces abdominal distension which is most marked on the left side.
• May be effectively treated without resorting to laparotomy.
• May produce bowel strangulation.

6- Hirschsprung’s disease:
• Is the result of acquired aganglionosis of the large bowel.
• Usually becomes evident in early adult life.
• Can usually be diagnosed on a barium enema.
• Can usually be managed by dietary means.
• A&D only.

7- Colonic polyps: (all correct except one)


• Are associated with colonic cancer.
• May be hereditary.
• Should not be removed if they are asymptomatic.
• May be hyperplastic.
• Are commonly adenomatous.

8- A right sided colonic cancer frequently presents with:


• Anaemia.
• Intestinal obstruction.
• Rectal bleeding
• An abdominal mass.
• A&D only

9- A left sided colonic cancer frequently presents with: ( all correct except one)
• Anaemia.
• Intestinal obstruction.
• Rectal bleeding.
• Abdominal pain.
• Pencil like stools.

Extrahepatic Biliary System


Spleen And Pancreas
1- Acute cholecystitis: (all correct except one)
• Is almost invariably related to the presence of gallstones.
• Usually presents with biliary colic.
• Is often associated with jaundice.
• Is characterised by a pyrexia in the early hours of the disease..
• May be complicated by peritonitis.

2- Acute cholecystitis should usually be treated by: (all correct except one)
• Nasogastric suction and intravenous fluids.
• Antibiotic therapy.
• Urgent cholecystectomy.
• Cholecystostomy.
• Analgesics and antispasmotics.

3- Gallstones: (all correct except one)


• Have an incidence which increases with age.
• Are more frequent in females.
• Usually contain a predominance of cholesterol.
• Are formed in bile which is supersaturated with bile acids.
• Are formed in bile which is supersaturated with cholesterol.

4- The presence of stones in the common bile duct : ( all correct except one)????
• Is commonly associated with a long history of dyspepsia.
• Is usually associated with jaundice.
• Must be considered during every cholecystectomy.
• May requires treatment by choledochoduodenostomy.
• May cause liver abscess.

5- stones in the common bile duct:??


• Are present in nearly 50 per cent of cases of cholecystitis.
• Often give rise to jaundice, fever and biliary colic.
• Are usually accompanied by progressive jaundice.
• Are usually associated with a distended gallbladder.
• A&D only.

6- In severe jaundice diagnostic evidence of an extrahepatic obstrction of the biliary tract


may be gained by: (all correct except one)???
• Intravenous cholangiography.
• A barium meal.
• Endoscopic retrograde cholangiography.
• Percutaneous transhepatic cholangiography.
• CT scan of the abdomen.

7- Internal biliary fistulae: (all correct except one)


• Most commonly arise as a consequence of cholecystitis.
• Most commonly occur between the gallbladder and the duodenum.
• Can result in intestinal obstruction.
• Are in most cases fatal complications.
• May cause cholongitis.

8- Carcinoma of the gallbladder:


• Is usually a squamous cell neoplasm.
• Is more common in men.
• Is rarely associated with gallstones.
• Has a relatively poor prognosis.
• Non of the above is correct.

9- hypersplenism:
• Results in anaemia, leucopenia and thrombocytopenia.
• Only occurs in the presence of a large spleen.
• Frequently follows liver cirrhosis.
• May be diagnosed by bone marrow biopsy.
• All of the above are correct.

10- Acute pancreatitis: (all correct except one)


• Has a higher incidence in alcoholics.
• Is commonly associated with the presence of gallstones.
• Occurs most commonly in diabetics.
• Becomes less severe with each recurring episode.
• May occurs as a complication of abdomenal trauma.

11- Acute pancratitis is characterised by: (all correct except one)


• A history of gallstones
• Diffuse epigastric pain.
• Exaggerated bowel sounds.
• An elevated urinary amylase.
• An elevated serum amylase.

12-chronic pancreatitis:
• Is commonly associated with alcoholism.
• Is associated with diabetes.
• May be diagnosed by the analysis of pancreatic secretions.
• May be treated by surgical procedures which decompress the pancreatic duct.
• All of the above are correct.

13-Pancreatic pseudocysts: (all correct except one)


• Are developmental in origin.
• Usually arise in the lesser peritoneal sac
• Produce a smooth epigastric mass which does not moves on respiration.
• May be effectively treated by internal drainage.
• May be associated by elevation of serum amylase.

Fractures And Dislocations


1- A fracture is said to be:
• Closed if an overlying skin laceration has been sutured.
• Comminuted if there has been associated damage to adjacent nerves or vessels.
• A fatigue fracture if it occurs through a diseased bone.
• Pathological if it occurs through a bony metastasis.
• Non of the above correct.

2- In a healing fracture: (All correct except one)


• The haematoma is initially invaded by osteoblasts.
• The tissue formed by the invading osteoblasts is termed osteoid.
• Calcium salts are laid down in the osteoid tissue.
• The final stage of repair is the remodelling of the callus.
• The callus formation is related to the amount of stress at fracture side.

3- Non-union is often seen in:


• Fractures of the 4th metatarsal.
• Fractures of the neck of the femur.
• Fractures of the condyle of the mandible.
• Colles’ fractures.
• Oblique fracture line of femur.

4- Fractures of the clavicle:


• Are usually of the greenstick variety in children under the age of 10 years.
• Are usually the result of direct violence.
• Can be recognised by the abnormal elevation of the distal fragment.
• Are usually treated by internal fixation.
• C&D only.

5- in fractures of the surgical neck of the humerus: (All correct except one)
• The lesion is usually due to indirect violence.
• The fragments are usually impacted.
• The proximal fragment is usually internally rotated.
• The distal fragment is usually adducted.
• Early mobilisation is encouraged.

6- In a fracture of the distal third of the shaft of the humerus:


• The distal fragment is usually posteriorly angulated by the action of biceps.
• The radial nerve is rarely damaged.
• Delayed radial nerve palsy is usually due to oedema.
• Late onset of radial nerve palsy is usually due the involvement of the nerve with callus.
• Non of the above is correct.

7- A supracondylar fracture of the humerus: ( All correct except one)


• Is a fracture commonly seen in childhood.
• Is particularly subject to the complication of ischaemic muscle contracture.
• Is held in the position of reduction by the tendon of brachioradialis.
• When properly reduced has the index finger pointing approximately to the tip of the
shoulder of the same side.
• When reduced care must be taken to ensure that the radial pulse is present.

8- A transverse fracture of the scaphoid is:


• Prone to infection.
• Usually seen in old men.
• Prone to avascular necrosis.
• Usually seen on an early scaphoid x-ray.
• All of the above is correct.

9- In a colles’ fracture the distal radial fragment:


• Is dorsally angulated on the proximal radius.
• Is usually torn from the intra-articular triangular disc.
• Is deviated to the ulnar side.
• Is rarely impacted.
• Is ventrally displaced.

10- dislocations of the shoulder joint:


• Most commonly occur in middle age.
• Usually occur when the arm is in the abducted position.
• Usually have the head of the humerus situated behind the glenoid fossa.
• Are often recurrent in the young.
• B&D only.

11- In pelvic fractures:


• Avulsion injuries are usually treated by early mobilisation.
• Undisplaced lesions of the ischial or pubic rami are usually treated by early dmbilisation.
• Extraperitoneal urinary extravasation may be due to damage either to the membraneous
urethra or to the base of the bladder.
• Which are unstable are accompanied by extensive hemorrhage.
• All of the above are correct.

12- Intracapsular fractures of the upper end of the femur are usually: (All correct except
one)
• Accompanied by shortening of the leg.
• Accompanied by external rotation of the leg.
• Accompanied by adduction of the leg.
• Treated by internal fixation.
• Accompanied with a vascular necrosis of the head of femur.

13- Extracapsular fractures of the upper end of the femur are usually:
• Subtrochanteric in position.
• Subject t o avascular necrosis of the head of the femur.
• Accompanied by internal rotation of the leg.
• Treated by external fixation.
• Non of the above are correct.

14- In fractures of the mid shaft of the femur: ( all correct except one)
• The proximal fragment is usually flexed.
• The proximal fragment is usually abducted.
• The distal fragment is usually adducted.
• The common femoral vessels are usually damaged.
• Hamstring and quadriceps produce some shortening of the leg.

15- In fractures of the middle third of the tibia and fibula: (All correct except one)
• Delayed union is common.
• Indirect violence usually results in a spiral or oblique fracture line.
• Shortening and anterior angulation of the tibia are common.
• Comminuted fractures are usually treated external fixation.
• All of the above are correct.

16- In injuries of the ankle joint:


• Eversion injuries are the most commonly encoutered.
• Inversion injuries are usually accomapanied by a tear of the deltoid ligament.
• There is frequently associated posterior tibial nerve damage.
• The joint is rendered unstable by rupture of the inferior tibio-fibular ligament.
• A&B only.

17- Dislocation of the hip joint:


• Is most common when the hip is in a neutral position.
• Is usually associated with a fracture of the acetabular rim.
• Usually results in the femoral head coming to lie anteiorly over the pubis or obturator
externus.
• May be associated with injuries of the sciatic nerve.
• B&D only.

18- Acute osteomyelitis in childhood:


• Is usually the result of compound bony injuries.
• Is characterised by a constant bone pain.
• Characteristiclly produces necrosis of the periosteum overlying the infected bone.
• Is not demonstrable radiologically for the first week of the disease.
• All of the above are correct.

19- In osteoarthritis of the hip joint:


• The articular cartilage undergoes initial hypertrophy and then becomes hardened and
eburnated.
• The joint capsule becomes stretched and lax.
• The leg is usually adducted and externally rotated when the patient lies supine.
• A femoral osteotomy usually helps halt the progress of the disease process.
• C&D only.

20- In a case of congenital dislocation of the hip:


• There is a defect of the posterior rim of the acetabulum.
• On bilateral hip abduction with the knees extended there is often limited abduction on the
diseased side.
• Reduction is sometimes hindered by a tight gluteus minimus muscle.
• Splinting of the limbs following reduction should be maintained until the femoral
epiphysis returns to its normal density on x-ray examination.
• Non of the above is correct.

21- Osteognic sarcoma:


• Are most frequent in the 10 to 25 year age group.
• Readily metastasise via the blood stream.
• Are frequently surrounded by non-malignant new bone formation.
• When treated by conventional methods have a 10 percent 5 years survival rate.
• All of the above are correct.
______________________________________________________________________________

1. A 35-year-old male is struck on the lateral aspect of his right knee by the bumper of a
car travelling at low velocity. On examination he is unable to dorsiflex the ankle, evert the
foot and extend the toes. There is loss of sensation of the dorsum of the foot. He is most
likely to have damaged which structure?
A. Common peroneal nerve
B. Deep peroneal nerve
C. Saphenous nerve

2. Which one of the following suggest a diagnosis of Hirschsprung's disease?


A. A contrast-study showing dilatation of the aganglionic bowel segment.
B. Early presentation with vomiting.
C. neonatal large bowel obstruction.
D. Presentation after 1 year of age
E. Red current jelly stools.

3. A 46-year-old man presents after penetrating injuries to his arm and forearm. He is
unable to extend his fingers. There is no sensory disturbance and there is no vascular
injury. Which nerve has been damaged?
A. anterior interosseous nerve
B. median nerve
C. posterior interosseous nerve
D. radial nerve
E. ulna nerve

4. Which of the following regarding the anatomy of the heart is true?


A. The aortic valve is tricuspid
B. The ascending aorta is entirely outside the pericardial sac
C. The left atrial appendage is identified readily by transthoracic echocardiography.

D. The pulmonary trunk lies anterior to the ascending aorta


E. The right atrium is posterior to the left atrium.

5. Which of the following is not within the carpal tunnel?


A. median nerve
B. flexor digitorum profundus
C. flexor digitorum superficialis
D. flexor pollicis longus?
E. abductor pollicis longus

6. A 48-year-old woman with chronic pancreatitis due to gallstones is noted to have a


macrocytic anemia What is the most likely cause of the anemia?
A. Bone marrow dysfunction
B. Folate deficiency
C. Hyposplenism
D. Hypothyroidism
E. Vitamin B12 deficiency

7. Which of the following organs is in direct contact with the anterior surface of the left
kidney, without being separated from it by visceral peritoneum?
A. Duodenum
B. Jejunum
C. Pancreas
D. Spleen
E. Stomach

8. A 5-year-old boy presents to Accident and Emergency complaining of acute pain over his
upper tibia He is febrile and he refuses to move his leg. A diagnosis of osteomyelitis is
suspected The likely infecting organism is?
A. Clostridium difficile
B. Haemophilus influenzae
C. Pseudomonas
D. Salmonella
E. Staphylococcus aureus

9. Which is the principle root inervation for the small muscles of the hand?
A. C5
B. C6
C. C7
D. C8
E. T1

10. Which of the following is true concerning Scaphoid fractures?


A. Rarely occur in young adults
B. when complicated by avascular necrosis the proximal pole is usually affected
C. should be treated by bone grafting and internal fixation even if undisplaced

11. A 17-year-old girl underwent emergency splenectomy after a domestic accident. Which
one of the following organisms is most likely to cause life-threatening infection in the
future?
A. Actinomycosis
B. Haemophilus influenzae
C. Pseudomonas aeruginosa
D. Staphylococcus aureus
E. Streptococcus pneumonia

12. Which vertebral level and corresponding structure is correct?


A. C4 and bifurcation of the carotid artery
B. T2 and manubriosternal joint
C. T10 and opening for vena cava in diaphragm
D. T12 and oesophageal opening in the diaphragm
E. T8 and aortic opening in the diaphragm

13. Which of the following statements is true of Compartment syndrome:


A. Only occurs following fractures.
B. Loss of distal pulse is an early sign.
C. The presence of pain is unhelpful in diagnosis.
D. Passive stretch of affected muscles exacerbates pain.

14. The axillary nerve passes directly inferior to which muscle as it leaves the axilla?
A. Long head of triceps
B. Long head of biceps
C. Pectoralis major
D. Subscapularis
E. Teres minor

15. Which of the following is not a branch of the pudendal nerve?


A. Perineal nerve
B. Dorsal nerve of the penis.
C. Inferior rectal nerve
D. Genitofemoral nerve
E. Posterior scrotal nerve

16. A patient presents with a history of low back pain and sciatica The pain radiates to the
little toe, the ankle reflex is absent and the patient has difficulty in everting the foot. Which
nerve root is likely to be trapped
A. L3
B. L4
C. L5
D. S1
E. S2

17. The commonest clinical manifestation of primary hyperparathyroidism is:


A. Renal stone disease
B. Bone disease
C. Peptic ulceration
D. Constipation
E. Polyuria

18. Which of the following structures accompany the median nerve in the carpal tunnel?

A. Flexor carpi ulnaris.


B. Flexor digitorum profundis.
C. The ulnar artery.
D. All of the above
E. None of the above

19. Which of the following is a branch of the LATERAL CORD of the brachial plexus?

A. Suprascapular nerve
B. Lower subscapular nerve
C. Medial pectoral nerve
D. Musculo-cutaneous nerve
E. Upper subscapular nerve

20. Which of the following vertebrae has the most prominent spinous process?
A. T1.
B. T2.
C. C7.
D. T11.

21. The ulnar nerve all are true except :


A. innervates the first dorsal interosseus muscle
B. originates from the laterall cord of the brachial plexus
C. has no branches above the elbow
D. innervates the medial half of the flexor digitorum profundus
E. innervates the adductor pollicis muscle

22. A 9 month old boy presents with an acute scrotal swelling. The following diagnoses are
likely:
A. Epididymitis
B. Orchitis
C. Torsion of the testicular appendage
D. Irreducible inguinal hernia
E. Acute idiopathic scrotal oedema

23. Congenital pyloric stenosis:


A. Presents in the first few days after birth.
B. Can cause hyperchloraemic alkalosis.
C. Is inherited as an autosomal recessive
D. Typically presents with bile stained vomiting after feeds
E. Is often diagnosed by feeling a mass in the right hypochondrium.

24. If torsion of the testicle is suspected, surgical exploration:


A. Should be immediate and include the asymptomatic side

25. Meckel's diverticulum all are true except :


A. Can present with chronic anaemia
B. Is present in 2% of the population.
C. Can be diagnosed by hydrogen breath test.
D. May contain ectopic pancreatic tissue
E. May be implicated in Littre's hernia

26. Which of the following statements is true of upper limb nerve injuries?
A. Injury to the median nerve results in a wrist drop
B. Injury to the radial nerve results in loss of sensation over the palmar aspect of the index
finger
C. Injury to the median nerve results in loss of sensation in the anatomical snuffbox
D. Injury to the ulnar nerve results in a claw hand
E. Injury to the ulnar nerve results in loss of sensation over the thumb
27. Concerning the inguinal canal:
A. It transmits the ilio-inguinal nerve
B. The deep inguinal ring lies below the mid-point of inguinal ligament
C. The superficial inguinal ring overlies the pubic tubercle
D. Laterally , the anterior wall is made up of the external oblique aponeurosis
E. Laterally, the posterior wall is formed by the conjoint tendon

28. In chest trauma, urgent cardiothoracic surgical referral is necessary for all except:
A. Continuing massive air leak following insertion of chest drain
B. Cardiac tamponade
C. Disruption of the great vessels
D. Severe pulmonary contusion
E. Continuing haemorrhage following insertion of chest drain

29. A raised titre of anti-HBs in the bloodallare true except :


A. signifies previous hepatitis B infection
B. is produced after hepatitis B vaccination
C. indicates immunity to hepatitis infection
D. indicates active hepatitis B infection

30. The following statements regarding intravenous solutions is correct:


A. Normal saline contains 180mmol/l of sodium
B. Ringer's lactate solutions is designed for intracellular fluid replacement
C. Sodium bicarbonate 8.4% is a hyperosmolar solution
D. Normal saline with added potassium is appropriate therapy to correct a respiratory
alkalosis

31. The following is true of the blood supply of the rectum all are true except:
A. The principle blood supply is derived from the inferior rectal artery
B. The median sacral vessels contribute to the blood supply
C. As the inferior mesenteric artery crosses the pelvic brim it becomes known as the superior
rectal artery
D. The internal venous rectal plexus is continuous with the vascular cushions of the anal
canal
E. The superior rectal vein drains to the portal system

32. Which of the following concerning the Femoral sheath is false:


A. Contains the femoral artery
B. Contains lymph nodes
C. Contains the femoral canal
D. Contains femoral nerve

33. The following is true of the spleen:


A. Is the largest lymphoid organ in the body
B. Lies obliquely between the seventh and tenth rib
C. The lower pole extends beyond the mid-axillary line
D. Is usually palpable when normal
E. Usually measures 16cm in maximum length when healthy

34. A knife stabbed horizontally through the fourth intercostals space to the right of the
sternum is likely to damage the:
A. Right brachiocephalic vein-
B. Hemiazygos vein
C. Descending aorta
D. Right pulmonary artery
E. Thoracic duct

35. Breast cancer risk is increased in association with the following factors except:
A. Nulliparity
B. Immediately after pregnancy
C. Early menarche
D. Early age at first pregnancy
E. Late menopause

36. The Appendix all true except:


A. Is typically less than 10 cm in length in the adult.
B. Is located in the retrocaecal recess.
C. Macburneys point, lies 2/3 laterally from a line from umbilicus to the anterior superior
iliac spine
D. The longitudinal coat of the appendix is derived from the three bands of taenia coli.
E. Is supplied by branches of the inferior mesenteric artery

37. In tension pneumothorax the following signs are present except:


A. Hypoxia
B. Hyper resonance to percussion on the affected side
C. Tracheal deviation to the ipsilateral side
D. Distended neck veins
E. Tachycardia

38. The following are encountered during inguinal hernia repair :


A. Scarpas' fascia
B. Colle's fascia
C. Ilioinguinal nerve
D. Cremaster muscle
E. Genitofemoral nerve

39. Which of the following variables best predicts prognosis for patients with a recent
diagnosis of cutaneous melanoma and no clinical evidence of metastatic disease?
A. Breslow thickness

40. The intravenous fluid that a 60 kg., 30-year-old woman with an 80% burn should be
given in the first 24 hours following burn injury is:
A. 19.2 liters of 5% glucose in lactated Ringer's.
B. 14.4 liters of lactated Ringer's.

41. All venous sinuses of the dura mater ultimately drain into :
A. Internal jugular vein

42. The tendon of biceps bronchii muscle inserted into the


A. Radial tuberosity

43. the saphenous nerve is terminal branch of the :


A. Ilioinguinal nerve
B. Femoral nerve
C. Genitofemoral nerve
D. Superior glutial nerve
E. popliteal nerve

44. All the following origenat on the scapula except


A. Trapezius
B. Teres major
C. Deltoid
D. Brachio-radialis
E. Coraco-brachialis

45. Total number of human vertebra is:


A. 33 vertebras.
46. Foot droop is usually associated with paralysis of the:
A. Common peroneal

47. The following pass through the aortic hiatus of the diaphragm
A. Aorta
B. Azygos vein
C. Thoracic duct
D. All of the above
E. Non of the above

48. At birth the umbilical vein become the:


A. Round ligament of the liver

49. Arrangement of structures at hilum of the right kidney as following from anterior to
posterior:
A. Vein ,artery , ureter

50. Shock can best be defined as:


A. Hypotension.
B. Hypo perfusion of tissues.
C. Hypoxemia
D. All of the above

51. All of the following are true about neurogenic shock except:
A. there is a decrease in systemic vascular resistance and an increase in venous capacitance
B. tachycardia or bradycardia may be observed, along with hypotension.
C. the use of an alpha agonist such as phenylephrine is the mainstay of treatment.
D. Severe head injury, spinal cord injury, and high spinal anesthesia may all cause
neurogenic shock.

52. All of the following may be useful in the treatment of cardiogenic shock except:
A. Dobutamine
B. Sodium nitroprusside
C. Pneumatic antishock garment.
D. Intra-aortic balloon pump.

53. All of the following statements about hemorrhagic shock are true except:
A. Following hemorrhagic shock, there is an initial interstitial fluid volume contraction.
B. Dopamine, or a similar inotropic agent, should be given immediately for resuscitation
from hemorrhagic shock, to increase cardiac output and improve oxygen delivery to
hypoperfused tissues.
C. The use of colloid solutions or hypertonic saline solutions is indicated for treatment of
hemorrhagic shock.
D. In hemorrhagic shock, a narrowed pulse pressure is commonly seen before a fall in
systolic blood pressure

54. Which of the following statements about head injury and concomitant hyponatremia is
true?
A. There are no primary alterations in cardiovascular signs.
B. Signs of increased intracranial pressure may be masked by the hyponatremia
C. Oliguric renal failure is an unlikely complication.
D. Rapid correction of the hyponatremia may prevent central pontine injury.
E. This patient is best treated by restriction of water intake

55. Which of the following statements about extracellular fluid is true?


A. The total extracellular fluid volume represents 40% of the body weight.
B. The plasma volume constitutes 20% of the total extracellular fluid volume
C. Potassium is the principal cation in extracellular fluid
D. The protein content of the plasma produces a lower concentration of cations than in the
interstitial fluid
E. The interstitial fluid equilibrates slowly with the other body compartments.

56. Which of the following statements is true of a patient with hyperglycemia and
hyponatremia?
A. Insulin administration will increase the potassium content of cells.
B. The sodium concentration must be corrected by 5 mEq. per 100 mg. per 100 ml. elevation
in blood glucose
C. With normal renal function, this patient is likely to be volume overloaded
D. Proper fluid therapy would be unlikely to include potassium administration.

57. Which of the following is not associated with increased likelihood of infection after
major elective surgery?
A. Age over 70 years.
B. Chronic malnutrition.
C. Controlled diabetes mellitus.
D. Long-term steroid use
E. Infection at a remote body site

58. The transfusion of fresh frozen plasma (FFP) is indicated for which of the following
reasons?
A. For the correction of abnormal PT secondary to warfarin therapy, vitamin K deficiency, or
liver disease
B. Volume replacement.
C. As a nutritional supplement.
D. Treatment of prolonged bleeding time

59. In patients receiving massive blood transfusion for acute blood loss, which of the
following is correct?
A. Packed red blood cells and crystalloid solution should be infused to restore oxygen-
carrying capacity and intravascular volume
B. Two units of FFP should be given with every 5 units of packed red blood cells in most
cases.
C. A “six pack” of platelets should be administered with every 10 units of packed red blood
cells in most cases.
D. One to two ampules of sodium bicarbonate should be administered with every 5 units of
packed red blood cells to avoid acidosis.

60. Hemostasis and the cessation of bleeding require which of the following processes?
A. Adherence of platelets to exposed subendothelial glycoproteins and collagen with
subsequent aggregation of platelets and formation of a hemostatic plug.
B. Interaction of tissue factor with factor VII circulating in the plasma
C. The production of thrombin via the coagulation cascade with conversion of fibrinogen to
fibrin.
D. Cross-linking of fibrin by factor XIII.
E. All of the above

61. Advantages of epidural analgesia include:


A. Earlier mobilization after surgery.
B. Earlier return of bowel function.
C. Shorter hospitalizations.
D. Decreased stress response to surgery.
E. All of the above

62. Factors that decrease collagen synthesis include all of the following except:
A. Protein depletion.
B. Infection.
C. Anemia
D. Advanced age
E. Hypoxia
63. Which of the following statement is true concerning excessive scarring processes?
A. Keloids occur randomly regardless of gender or race
B. Hypertrophic scars and keloid are histologically different
C. Keloids tend to develop early and hypertrophic scars late after the surgical injury
D. Simple reexcision and closure of a hypertrophic scar can be useful in certain situations
such as a wound closed by secondary intention
140 ‫نفس السؤال صفحة‬
B ‫واالجابة‬

64. Which of the following statement is true concerning the vascular response to injury?
A. Vasoconstriction is an early event in the response to injury
B. Vasodilatation is a detrimental response to injury with normal body processes working to
avoid this process
C. Vascular permeability is maintained to prevent further cellular injury
D. Histamine, prostaglandin E2 (PGE2) and prostacyclin (PGI2) are important mediators of
local vasoconstriction

65. All are true ligamentous attachments of liver except


A. Falciform ligament
B. Coronary Ligaments
C. Hepatoduodenal ligaments
D. Glisson's capsul

66. The appropriate surgical treatment for suspected carcinoma of the testis is:
A. Inguinal exploration, control of the spermatic cord, biopsy, and radical orchectomy if
tumor is confirmed

67. Nasotracheal intubation all true except :


A. Is preferred for the unconscious patient without cervical spine injury.
B. Is preferred for patients with suspected cervical spine injury.
C. Maximizes neck manipulation.
D. Is contraindicated in the patient who is breathing spontaneously.

68. The radiographic findings indicating a torn thoracic aorta include all except :
A. Widened mediastinum.
B. Presence of an apical “pleural cap.”
C. Tracheal deviation to the right.
D. right hem thorax.
69. Which of the following steps is not a part of the primary survey in a trauma patient?
A. Insuring adequate ventilatory support
B. Measurement of blood pressure and pulse
C. Neurologic evaluation with the Glasgow Coma Scale
D. Examination of the cervical spine

70. Which of the following statement(s) is/are true concerning the Advanced Trauma Life
Support (ATLS) classification system of hemorrhagic shock?
A. Class I shock is equivalent to voluntary blood donation
B. In Class II shock there will be evidence of change in vital signs with tachycardia,
tachypnea and a significant decrease in systolic blood pressure
C. Class IV hemorrhage can usually be managed by simple administration of crystalloid
solution
D. Class III hemorrhage involves loss of over 40% of blood volume loss and can be
classified as life-threatening

71. The clinical picture of gallstone ileus includes all except ?


A. Air in the biliary tree
B. Small bowel obstruction.
C. A stone at the site of obstruction.
D. Acholic stools.
E. Associated bouts of cholangitis.

72. The evaluation of a comatose patient with a head injury begins with:
A. Establishment of an airway.

73. Which of the following signs does Horner's syndrome include?


A. Ptosis.
B. Facial hyperhidrosis.
C. Exophthalmos.
D. Mydriasis.

74. The goals of proper fracture reduction include all except?


A. Providing patient comfort and analgesia
B. Allowing for restoration of length of the extremity.
C. Correcting angular deformity and rotation.
D. Enabling immediate motion of all fractured extremities.
75. The neurovascular structure most commonly injured as a result of an anterior
dislocation of the shoulder is the:
A. Musculocutaneous nerve
B. Axillary nerve
C. Axillary artery.
D. Median nerve

76. The radial nerve is at greatest risk for injury with which fracture?
A. Fracture of the surgical neck of the humerus.
B. Fracture of the shaft of the humerus.
C. Supracondylar fracture of the humerus.

77. The most consistent sign of a fracture of the carpal scaphoid is:
A. Wrist pain during attempted push-ups.
B. Diffuse swelling on the dorsum of the wrist.
C. Localized tenderness in the anatomic snuffbox.
D. Wrist popping on movement.

78. Hematogenous osteomyelitis most frequently affects:


A. The diaphysis of long bones.
B. The epiphysis.
C. The metaphysis of long bones.
D. Flat bones.
E. Cuboidal bones.

79. The most common hernia in females is:


A. Femoral hernia
B. Direct inguinal hernia
C. Indirect inguinal hernia

80. Which of the following most often initiates the development of acute appendicitis?
A. A viral infection.
B. Acute gastroenteritis.
C. Obstruction of the appendiceal lumen.

81. The most helpful diagnostic radiographic procedure in small bowel obstruction is:
A. CT of the abdomen.
B. Contrast study of the intestine
C. Supine and erect x-rays of the abdomen.
D. Ultrasonography of the abdomen.
82. Which of the following treatments should never be recommended to a patient with
purely intraductal carcinoma?
A. Modified radical mastectomy.
B. Lumpectomy to clear surgical margins, followed by observation.
C. Incisional biopsy with an involved margin, followed by radiation.
D. Excisional biopsy to clear margins, followed by radiation.

83. In role of nine extent of burn if entire trunk is burned it will be equal to:
A. 9% body surface area
B. 18% body surface area
C. 36% body surface area
D. 27% body surface area

84. Which of the following does not describe intermittent claudication?


A. Is elicited by reproducible amount of exercise
B. Abates promptly with rest.
C. Is often worse at night.
D. May be an indication for bypass surgery.

85. Which of the following statements are true?


A. Patients with critical limb ischemia have paralysis and paresthesias.
B. All arterial injuries are associated with absence of a palpable pulse
C. Preoperative arteriography is required to diagnose an arterial injury.
D. The presence of Doppler signals indicates that an arterial injury has not occurred
E. In all patients with multiple trauma, arterial injuries should be repaired before other
injuries are addressed

86. A patient with acute urinary tract infection (UTI) usually presents with:
A. Chills and fever.
B. Flank pain.
C. Nausea and vomiting.
D. Painful urination.

87. The most precise diagnostic screening procedure for differentiating benign thyroid
nodules from malignant ones
A. Fine-needle-aspiration biopsy (FNAB).

88. Hyperthyroidism can be caused by all of the following except:


A. Graves' disease
B. Plummer's disease
C. Struma ovarii.
D. Hashimoto's disease
E. Medullary carcinoma of the thyroid
_______________________________________________________________________

1. Which of the following anatomic features of the biliary system are important
considerations in operative cholangiography?
A. The left hepatic duct comes off farther anterior than the right one.
B. At the confluence there may be more than just a right and a left hepatic duct.
C. Dissection of the triangle of Calot is more important than cholangiography in preventing
bile duct injury.
D. Segments V, VII, or VIII sometimes join the biliary system below the confluence.
E. All of the above are correct.

2. Which of the following statements characterize amebic abscess?


A. Mortality is higher than that for similarly located pyogenic abscesses.
B. The diagnosis of amebic abscess may be based on serologic tests and resolution of
symptoms.
C. In contrast to pyogenic abscess, the treatment of amebic abscess is primarily surgical.
D. Patients with amebic abscess tend to be older than those with pyogenic abscess.
5:09 end (28p [ in 110 mi
2:50
3. Which of the following statement(s) is/are true about benign lesions of the liver?
A. Adenomas are true neoplasms with a predisposition for complications and should usually
be resected.
B. Focal nodular hyperplasia (FNH) is a neoplasm related to birth control pills (BCPs) and
usually requires resection.
C. Hemangiomas are the most common benign lesions of the liver that come to the surgeon's
attention.
D. Nodular regenerative hyperplasia does not usually accompany cirrhosis.

4. Which of the following statements about hemobilia are true?


A. Tumors are the most common cause.
B. The primary treatment of severe hemobilia is an operation.
C. Percutaneous cholangiographic hemobilia is usually minor.
D. Ultrasonography usually reveals a specific diagnosis.

5. Ligation of all of the following arteries usually causes significant hepatic enzyme
abnormalities except:
A. Ligation of the right hepatic artery.
B. Ligation of the left hepatic artery.
C. Ligation of the hepatic artery distal to the gastroduodenal branch.
D. Ligation of the hepatic artery proximal to the gastroduodenal artery
6. Which of the following is the most common acid-base disturbance in patients with
cirrhosis and portal hypertension?
C. Metabolic alkalosis.

7. Cytokines are endogenous signals that stimulate:


A. Septic shock
B. The central nervous system to initiate fever.
C. The production of “acute-phase proteins.”
D. Hypoferremia.
E. All of the above are correct.

8. Which of the following statements about maxillofacial trauma is/are false?


A. Asphyxia due to upper airway obstruction is the major cause of death from facial injuries.
B. The mandible is the most common site of facial fracture.
C. The Le Fort II fracture includes a horizontal fracture of the maxilla along with nasal bone
fracture.
D. Loss of upward gaze may indicate either an orbital floor or orbital roof fracture.

9. A 28-year-old male was injured in a motorcycle accident in which he was not wearing a
helmet. On admission to the emergency room he was in severe respiratory distress and
hypotensive (blood pressure 80/40 mm. Hg), and appeared cyanotic . He was bleeding
profusely from the nose and had an obviously open femur fracture with exposed bone .
Breath sounds were decreased on the right side of the chest. The initial management
priority should be:
A. Control of hemorrhage with anterior and posterior nasal packing.
B. Tube thoracostomy in the right hemithorax.
C. Endotracheal intubation with in-line cervical traction.
D. Obtain intravenous access and begin emergency type O blood transfusions.
E. Obtain cross-table cervical spine film and chest film.

10. Regarding the diagnosis and treatment of cardiac tamponade, which of the following
statements is/are true?
A. Accumulation of greater than 250 ml. of blood in the pericardial sac is necessary to impair
cardiac output.
B. Beck's classic triad of signs of cardiac tamponade include distended neck veins, pulsus
paradoxicus, and hypotension.
C. Approximately 15% of needle pericardiocenteses give a false-negative result.
D. Cardiopulmonary bypass is required to repair most penetrating cardiac injuries.
11.Which of the following statement(s) is true concerning the diagnosis of a peripheral
vascular injury?
A. The presence of a Doppler signal over an artery in an extremity essentially rules out an
arterial injury
B. Doppler examination is a valuable tool in the diagnosis of venous injuries
C. A gunshot wound in the proximity of a major vessel is an absolute indication for
arteriography
D. Both the sensitivity and specificity of arteriography of the injured extremity approaches
100%

12. An 18-year-old male suffers a gunshot wound to the abdomen, resulting in multiple
injuries to the small bowel and colon. Which of the following statement(s) is/are true
concerning this patient’s perioperative management?
A. A multi-agent antibiotic regimen is indicated
B. Antibiotics should be continued postoperatively for at least 7 days
C. Laparotomy, as a diagnostic test for postoperative sepsis, should be considered
D. The incidence of postoperative wound or intraabdominal infection would be increased in
association with a colon injury

13. A 75-year-old man is involved in a motor vehicle accident. Which of the following
statement(s) is/are true concerning this patient’s injury and management?
A. Acceptable vital sign parameters are similar across all age groups
B. Hypertonic solutions should not be used for resuscitation due to concerns for fluid
overload
C. The patient would be more prone to a subdural hematoma than a younger patient
D. There is no role for inotropic agents in the management of this patient

14. Valid points in the management of burns on special areas include:


A. The large majority of genital burns are best managed by immediate excision and
autografting
B. All digits with deep dermal and full-thickness burns should be immobilized with six
weeks of axial Kirschner wire fixation
C. Deep thermal burns of the central face are best managed with immediate excision and
autografting
D. Burns of the external ear are commonly complicated by acute suppurative chondritis if
topical mafenide acetate is not applied

15. Which of the following statements regarding unusual hernias is incorrect?


A. An obturator hernia may produce nerve compression diagnosed by a positive Howship-
Romberg sign.
B. Grynfeltt's hernia appears through the superior lumbar triangle, whereas Petit's hernia
occurs through the inferior lumbar triangle.
C. Sciatic hernias usually present with a painful groin mass below the inguinal ligament.
D. Littre's hernia is defined by a Meckel's diverticulum presenting as the sole component of
the hernia sac.
E. Richter's hernia involves the antimesenteric surface of the intestine within the hernia sac
and may present with partial intestinal obstruction.

16. The following statement(s) is/are true concerning umbilical hernias in adults.
A. Most umbilical hernias in adults are the result of a congenital defect carried into adulthood
B. A paraumbilical hernia typically occurs in multiparous females
C. The presence of ascites is a contraindication to elective umbilical hernia repair.
D. Incarceration is uncommon with umbilical hernias

17. Which of the following statements about esophageal anatomy is correct?


A. The esophagus has a poor blood supply, which is segmental in distribution and accounts
for the high incidence of anastomotic leakage.
B. The esophageal serosa consists of a thin layer of fibroareolar tissue.
C. The esophagus has two distinct muscle layers, an outer, longitudinal one and an inner,
circular one, which are striated in the upper third and smooth in the distal two thirds.
D. Injury to the recurrent laryngeal nerve results in vocal cord dysfunction but does not affect
swallowing.
E. The lymphatic drainage of the esophagus is relatively sparse, localized primarily to
adjacent paraesophageal lymph nodes.

18. Which of the following is most reliable for confirming the occurrence of a significant
esophageal caustic injury?
A. History of the event.
B. Physical examination of the patient.
C. Barium esophagraphy.
D. Endoscopy.

19. Numerous epidemiologic associations have been made between (1) environmental and
dietary factors and (2) the incidence of gastric cancer, including all except:
A. Dietary nitrites.
B. Dietary salt.
C. Helicobacter pylori infection.
D. Dietary ascorbic acid.
20. Complete mechanical small bowel obstruction can cause dehydration by:
A. Interfering with oral intake of water.
B. Inducing vomiting.
C. Decreasing intestinal absorption of water.
D. Causing secretion of water into the intestinal lumen.
E. All of the above

21. Meckel's diverticulum most commonly presents as:


A. Gastrointestinal bleeding.
B. Obstruction.
C. Diverticulitis.
D. Intermittent abdominal pain.

22. A 45-year-old man with a history of previous right hemicolectomy for colon cancer
presents with colicky abdominal pain which has become constant over the last few hours.
He has marked abdominal distension and has had only minimal vomiting of a feculent
material. His abdomen is diffusely tender. Abdominal x-ray shows multiple air fluid levels
with dilatation of some loops to greater than 3 cm in diameter. The most likely diagnosis is:
A. Proximal small bowel obstruction
B. Distal small bowel obstruction
C. Acute appendicitis
D. Closed-loop small bowel obstruction

23. Which of the following statement(s) is true concerning laboratory tests which might be
obtained in the patient discussed above?
A. The presence of a white blood cell count > 15,000 would be highly suggestive of a closed-
loop obstruction
B. Metabolic acidosis mandates emergency exploration
C. An elevation of BUN would suggest underlying renal dysfunction
D. There is no rapidly available test to distinguish tissue necrosis from simple bowel
obstruction

24. Recurrence after operation for Crohn's disease:


A. Occurs after operations for ileal Crohn's but not colonic Crohn's.
B. Is usually found just proximal to an enteric anastomosis.
C. Rarely requires reoperation.
D. Occurs in 1% of patients at risk per year during the first 10 years after the operation.
E. Is prevented by maintenance therapy with corticosteroids.
25. The most common indication for surgery secondary to acute diverticulitis is:
A. Abscess.

26. Many patients with ulcerative colitis are operated upon electively with total abdominal
colectomy, rectal mucosectomy, formation of a small intestinal reservoir, and ileoanal
anastomosis. The most common postoperative complication after this operation is which of
the following?
B. Small bowel obstruction

27. A 30-year-old male two years postoperative after total abdominal colectomy with
ileoanal anastomosis reports a sudden increase in stool frequency, nocturnal leakage, and
low-grade fevers. Physical examination is unremarkable . Flexible endoscopic examination
of the small intestinal pouch reveals a friable erythematous mucosa .Biopsies of the mucosa
are obtained. While awaiting biopsy results, which of the following is the most appropriate
empiric therapy?
C. Oral metronidazole

28. Useful methods for detection of splenic injury, in descending order of sensitivity, are:
A. Magnetic resonance imaging (MRI)
B. CT.
C. Ultrasonography.
D. Isotope scan.

29. Which of the following comments does not describe hypersplenism?


A. It may occur without underlying disease identification.
B. It may be secondary to many hematologic illnesses.
C. It is associated with work hypertrophy from immune response.
D. It requires evaluation of the myeloproliferation.
E. It is associated with antibodies against platelets.

30.You are consulted regarding a 50-year old male with Laennec’s cirrhosis, portal
hypertension and hypersplenism. He has no history of gastrointestinal bleeding. You would
recommend which of the following?
D. Observation
31. Which of the following statements about the differential diagnosis of hypercalcemia is
correct.
A. Malignant tumors typically cause hypercalcemia by ectopic production of parathyroid
hormone (PTH).
B. The diagnosis of primary hyperparathyroidism is supported by these serum levels:
calcium, 10.8 mg. per dl.; chloride, 104 mmol. per liter; bicarbonate 21 mmol. per liter;
phosphorus, 2.4 mg. per dl.; elevated parathyroid hormone.
C. Familial hypocalciuric hypercalcemia is distinguished from primary hyperparathyroidism
by parathyroid imaging.
D. Although serum albumin binds calcium, the measured total calcium value is usually
unaffected in patients with severe hypoproteinemia.
E. Thiazide diuretics are a good treatment for hypercalcemia and can be given to patients
with apparent hypercalcemia of malignancy.

32. The parathyroid glands:


A. Develop from the second and third pharyngeal pouches, along with the palatine tonsil and
the thymus.
B. Migrate caudally in the neck in normal development but can be found anywhere from the
pharyngeal mucosa to the deep mediastinum.
C. Secrete PTH and calcitonin to manage calcium homeostasis.
D. Usually number four, but frequently number only two or three.
E. Contain enzymes that catalyze the conversion of 25(OH) vitamin D 3 to 1,25(OH) 2
vitamin D 3.

33. Which of the following breast lesions are noninvasive malignancies?


A. Intraductal carcinoma of the comedo type.
B. Tubular carcinoma and mucinous carcinoma.
C. Infiltrating ductal carcinoma and lobular carcinoma.
D. Medullary carcinoma, including atypical medullary lesions.

34. A 45-year-old woman presents with a weeping eczematoid lesion of her nippl E.
Which of the following statement(s) is/are true concerning her diagnosis and management?
A. Treatment is with warm compresses and oral antibiotics
B. Biopsy of the nipple revealing malignant cells within the milk ducts is invariably
associated with an underlying invasive carcinoma
C. The appropriate treatment is mastectomy
D. The lesion always represents a high-risk disease with a significant risk of subsequent
metastatic disease

35. The effect of high positive end-expiratory pressures (PEEP) on cardiac output is:
A. None.
B. Increased cardiac output.
C. Decreased cardiac output because of increased afterload to the left ventricle.
D. Decreased cardiac output because of decreased effective preload to the left ventricle.

36. The diagnosis of myasthenia gravis can be confirmed most reliably using:
D. Single-fiber EMG.

37. In an infant with suspected PDA, which of the following would be the optimal method
of confirming the diagnosis?
D. Two-dimensional echocardiography with continuous-wave and color-flow Doppler
echocardiography.

38. A 5-year-old girl is found on routine examination to have a pulmonic flow murmur,
fixed splitting of P2 and a right ventricular lift. The following is/are true:
A. Cardiac catheterization is indicated if the chest film shows cardiomegaly
B. Radiology report of “scimitar syndrome” findings on the chest film would indicate need
for an arteriogram
C. If the catheterization report is “ostium secondum defect,” at least one pulmonary vein
drains anomalously
D. Measured pulmonary vascular resistance of 14 Woods units/m2 with an ASD mandates
early repair
E. An ASD with Qp/Qs of 1.8 can be observed until symptoms occur

39. The single most important indication for emergency pulmonary embolectomy is:
C. The presence of persistent and intractable hypotension.

40. A 67-year-old man with acute popliteal arterial embolism has a negative cardiac echo
for source of the thrombus. The following is/are true:
A. Most likely non-cardiac source is a thoracic aortic aneurysm
B. Embolism is more common from femoral than popliteal arterial aneurysms
C. Emboli from popliteal aneurysms are often clinically silent
D. Embolism is rare from subclavian artery aneurysms

41. Which of the following statements rejection are true?


A. In the absence of immunosuppression, the time and intensity of rejection of transplants
between unrelated donors and recipients is highly variable.
B. Allograft rejection may be mediated by antibodies or by cells.
C. Allograft rejection is thought to be caused by Th2 cells.
D. Acute cellular rejection is the major cause for loss of clinical organ transplants.
E. An individual with “tolerance” is unable to reject an allograft.
42. Which of the following are contraindications to lung transplantation?
A. Age 65 years or older.
B. Current corticosteroid therapy.
C. History of thoracotomy.
D. Ventilator-dependent respiratory failure.

43. Familial hypercholesterolemia has been proposed as a disease to be treated by gene


therapy. The molecular basis of familial hypercholesterolemia is which of the following?
A. Absence of hepatic low density lipoprotein receptors
B. Overproduction of high density lipoprotein
C. Absence of lipoprotein lipase
D. Overproduction of hepatic ornithine transcarbamylase

44. The pentalogy of Cantrell includes all of the following except:


A. Epigastric omphalocele.
B. Sternal cleft.
C. Intracardiac defect.
D. Pericardial cyst.
E. Ectopia cordis.

45. Patients with Wilms’ tumors most frequently present with which of the following?
E. A unifocal, unilateral lesion

46. Renal adenocarcinomas:


A. Are of transitional cell origin.
B. Usually are associated with anemia.
C. Are difficult to diagnose.
D. Are extremely radiosensitive.
E. Frequently are signaled by gross hematuria.

47. A 55-year-old male presents with severe flank pain radiating to the groin associated
with nausea and vomiting. Urinalysis reveals hematuria .A plain abdominal film reveals a
radiopaque 5 mm stone in the area of the ureterovesical junction. Which of the following
statement(s) is/are true concerning this patient’s diagnosis and management?
A. A likely stone composition for this patient would be uric acid
B. The stone will likely pass spontaneously with the aid of increased hydration
C. Stone analysis is of relatively little importance
D. Patients with a calcium oxalate stone and a normal serum calcium level should undergo
further extensive metabolic evaluation

48. The most common physical findings in a patient with median nerve compression at the
wrist (carpal tunnel syndrome) are:
A. Diminished two-point discrimination and dryness of the index and long fingers.
B. Atrophy of the abductor pollicis brevis and opponens pollicis.
C. A positive percussion test at the wrist and a positive wrist flexion test producing
paresthesias at the thumb, index, and long fingers.
D. A weak grip in addition to hand cramping and difficulty writing.

49. Isolated flexor digitorum superficialis tendon function is determined by assessing:


A. Flexion of the metacarpophalangeal joint.
B. Flexion of the proximal interphalangeal joint with the adjacent digits held in extension.
C. Flexion of the distal interphalangeal joint.
D. Flexion of the proximal interphalangeal joint.

50. Which of the following statements are true?


A. The Hoffmann-Tinel sign localizes the level of a nerve injury.
B. Causalgia is a term used to denote the etiology of pain.
C. Secondary repair of a lacerated nerve 3 to 8 weeks after injury has several advantages.
D. A surgeon who finds at delayed (3 to 8 weeks) exploration that a clinically nonfunctioning
nerve is in continuity should resect the injured portion of the nerve and suture together the ends.
E. If a nerve is found to be disrupted at delayed (3 to 8 weeks) exploration, the surgeon
should find the two ends of the nerve and suture them together.

51. What is the critical difference between frame-based and frameless stereotactic
procedures?
A. The use of digitized imaging studies such as CT and MRI.
B. The use of rendered three-dimensional images and a three-dimensional digitizer.
C. Rigid fixation of the patient's head to the operating room table.
D. The presence of a lesion in the brain on digitized imaging studies.
E. The absence of a lesion in the brain on digitized imaging studies.

52. The following are absorbable sutures


A. Catgut
B. Silk
C. Polyamide (Nylon)
D. Polyglyconate (Maxon)
E. Polyglactin (Vicryl)
53. During surgery on the submandibular gland
A. An incision on the lower border of the mandible is safe
B. The submandibular gland is seen to wrap around the posterior border of mylohyoid
C. The facial artery and vein are divided as they course through the deep part of the gland
D. The hypoglossal nerve is seen to loop under the submandibular duct
E. Damage to the lingual nerve will cause loss of sensation to the posterior third of the
tongue

54. Regarding pancreatic carcinoma false


A. 90% are ductal adenocarcinomas
B. Less than 20% occur in the head of the gland
C. The usual presentation is with pain, weight loss and obstructive jaundice
D. Ultrasound has a sensitivity of 80-90% in the detection of the tumour
E. Less than 20% of patients are suitable for curative surgery

55. Regarding the management of major trauma


A. Deaths follow a trimodal distribution
B. X-rays after the primary survey should be of AP Cervical spine, chest and pelvis
C. Cardiac tamponade is characterised by raised BP, low JVP and muffled heart sounds
D. Assessment of uncomplicated limb fractures should occur during the primary survey
E. Deterioration of the casualty during the primary survey should lead to the secondary
survey

56. Regarding appendicitis


A. The risk of developing the illness is greatest in childhood
B. Mortality decreases with age .
C. 20% of appendices are extraperitoneal in a retrocaecal position
D. Faecoliths are present in 75-80% of resected specimens
E. Appendicitis is a possible diagnosis in the absence of abdominal tenderness

57. Regarding stones in the gallbladder


A. Cholesterol stones are the least common
B. Pigment stones are due increased excretion of polymerised conjugated bilirubin
C. Are not a risk factor for the development of gallbladder carcinoma
D. 90% of gallstones are radio-opaque
E. A mucocele of the gallbladder is caused by a stone impacted in Hartmann's pouch

58. Stones in the common bile duct the false is


A. Are found in 30% of patients undergoing cholecystectomy (Without pre-op ERCP)
B. Can present with Charcot's Triad
C. Are suggested by an bile duct diameter >8mm on ultrasound
D. ERCP, sphincterotomy and balloon clearance is now the treatment of choice
E. If removed by exploration of the common bile duct the T-tube can be removed after 3 days

59. Regarding crystalloid solutions the false is


A. Normal saline contains 154 mmol sodium and 154 mmol of chloride
B. 3 litres of dextrose saline in a day will provide 90 mmol of sodium
C. 2 grams of potassium chloride is equal to 57 mmol of the salt
D. Hartmann's solution contains calcium and bicarbonate
E. The daily maintenance potassium requirement of a 40 Kg woman is about 40 mmol

60. Solitary thyroid nodules the false is


A. Are more prevalent in women
B. In the adult population less than 10% are malignant
C. Less than 20% of scintigraphically cold nodules are malignant
D. The risk of a hot module being malignant is negligible
E. Should be surgically removed in all patients
3:50
3:58
61. Regarding abdominal wall hernias the false is
A. Almost 100,000 hernia operations are performed annually in the United Kingdom
B. Over 60% of inguinal hernias are indirect
C. In women inguinal hernias are as common as femoral hernias
D. The mortality assocaited with strangulation is over 10%
E. The mortality has reduced dramatically over the past 30 years

62. The femoral canal the false is


A. Lies medial to the femoral vein
B. Has the inguinal ligament as its anterior border
C. Has the lacunar ligament as its lateral border
D. Has the pectineal ligament as its posterior border
E. Contains the lymph node of Cloquet

63. Intermittent claudication the false is


A. Affects less than 1% of men over the age of 50 years
B. At 5 years 10% of claudicants will have progressed to an amputation
C. At 5 years 20% of claudicants will have died from ischaemic heart disease
D. Is usually associated with an ankle / brachial pressure index (ABPI) 0.3- 0.7
E. Is associated with a fall in the ABPI on exercise with delayed recovery
64. The pathology of ulcerative colitis the false is
A. Shows full thickness inflammation
B. The rectum is almost always involved
C. 10% patients have terminal ileal disease
D. Enterocutaneous or intestinal fistulae are less common
E. The serosa is usually normal

65. Regarding benign breast disease the false is


A. Cyclical mastalgia is the commonest reason for referral to the breast clinic
B. Fibroadenomas are derived from the breast lobule
C. Lactational breast abscesses are usually due to Staph aureus
D. Duct ectasia is less common in smokers
E. Atypical lobular hyperplasia is associated with an increased risk of breast cancer

66. Regarding anal fissures the false is


A. 10% occur in the posterior midline
B. Multiple fissures suggest a diagnosis of tuberculosis or Crohn's Disease
C. 50% of acute fissures heal with the use of a bulking agent
D. Sphincterotomy has a success rate of over 90%
E. Sphincterotomy is associated with minor faecal incontinence in over 15% of patients

67. Fibroadenomas the false is????


A. Are benign monoclonal neoplasms
B. Most commonly present in late adolescence or the early 20s
C. Should be diagnosed by triple assessment
D. At least 30% reduce in size over a 2 year period
E. Can be safely managed conservatively

68. Warfarin the false is


A. Reduces the concentration of vitamin A dependent clotting factors
B. Has a half life of about 36 hours
C. Crosses the placenta and should be avoided in pregnancy
D. Doses should be reduced in liver disease
E. An INR of between 2.0 and 3.0 is appropriate in the treatment of DVT

69. Heparin the false is


A. Is a heterogeneous mixture of sulphated polysaccharides
B. Potentiates the actions of antithrombin III
C. Has a half life of 12 hours
D. Can be reversed by protamine sulphate
E. Can induce an idiosyncratic thrombocytopenia

70. The following cause hypercalcaemia except


A. Sarcoidosis
B. Primary hyperparathyroidism
C. Acute pancreatitis
D. Metastatic bronchial carcinoma
E. Milk-Alkali syndrome

71. Regarding oesophageal cancer the false is


A. Squamous carcinomas predominate in the upper 2/3 of the oesophagus
B. Overall 5 year survival is greater than 50%
C. Tylosis genetically predisposes to the disease
D. 15% of adenocarcinomas are associated with Barrett's oesophagus
E. For palliation an Atkinson tube can be inserted endoscopically

72. Infantile hypertrophic pyloric stenosis the false is


A. Occurs with a male : female ratio of 4:1.
B. Sons of affected mothers have a 20% risk of developing the lesion
C. Invariably presents between three and four weeks of age
D. Typically presents with nonbilious vomiting
E. Surgical treatment is by Heller's Cardiomyotomy

73. Oesophageal atresia the false is


A. Is often associated with a distal trachea-oesophageal fistula
B. Polyhydramnios is often present late in pregnancy
C. 50% have other associated congenital abnormalities
D. Contrast X-ray studies are necessary to confirm the diagnosis
E. Post-operatively over 30% develop oesophageal strictures

74. The following are features of Fallot's Tetralogy except


A. An atrial septal defect
B. Pulmonary stenosis
C. Right ventricular hypertrophy
D. A right to left cardiac shunt
E. Cyanotic attacks during feeding and crying
75. The functions of a tracheostomy are to except
A. Bypass an upper airway obstruction
B. Reduce the anatomical dead space
C. Increase airway resistance
D. Protect against aspiration
E. Allow frequent airway suction

76. Medullary carcinoma of the thyroid the false is


A. Is a tumour of the parafollicular C cells
B. Produce thyroxine as the principle hormone
C. 90% of cases are sporadic
D. Can occur as part of the MEN type II syndrome
E. Total thyroidectomy is the surgical treatment of choice

77. The following are features of thyrotoxicosis except


A. Weight gain
B. Palpitations
C. Proximal myopathy
D. Increased skin pigmentation
E. Pretibial myxoedema

78. Regarding abdominal aortic aneurysms the false is


A. They commonly remain symptomless until they rupture
B. The risk of rupture increases with aortic diameter
C. Elective repair should have a 30-day mortality of less than 5%
D. Emergency repair has a 30-day mortality of less than 10%
E. The benefits of surgery for small (4.0-5.5 cm) is unproven

79. In patients with carotid artery disease ???


A. A bruit is a reliable sign of the degree of stenosis
B. Atheroma is most commonly seen in the external carotid artery
C. An embolic event often results in an ipsilateral hemiplegia
D. Prophylactic aspirin reduces the risk of a stroke
E. Surgery is of proven benefit in those with asymptomatic stenoses

80. Hepatocellular carcinoma the false is


A. Has a high incidence in East Africa and South-east Asia
B. Its worldwide incidence parallels the prevalence of Hepatitis B
C. Mycotoxins (e.g. aflatoxin) are an important aetiological factor
D. Over 80% of tumours are surgically resectable
E. Liver transplantation offers the only chance of cure in those with irresectable disease

81. Serum alpha fetoprotein the false is


A. Is increased in testicular tumours
B. In testicular tumours is produced by trophoblastic elements
C. Is increased in over 70% patients with hepatocellular carcinoma
D. In patients with hepatocellular carcinoma levels correlate well with size of the tumour
E. In patients with hepatocellular carcinoma levels fall following resection of the tumour

82. Regarding testicular tumours


A. They are the commonest malignancy in young men
B. Peak incidence for teratomas is seen at the age of 25 years
C. Seminomas are radiosensitive
D. Over 75% of Stage I teratomas are cured by surgery alone
E. Chemotherapy rarely produces a cure in those with metastatic disease

83. In patients with ascites the false is


A. A exudate has a protein content of greater than 30 g per litre
B. Both malignancy and infection result in a transudate
C. Ovarian carcinoma is the commonest malignant cause of ascites
D. Cirrhosis results in marked sodium retention
E. Tumour cells increase the permeability of the peritoneum

84. Regarding opiate analgesia the false is


A. Patient controlled analgesia (PCA) is more effective than intermittent parenteral dosing
B. The total opiate dose is usually reduced with a PCA
C. Fentanyl is more lipid soluble than morphine
D. Epidural morphine can result in late respiratory depression
E. Epidural and parenteral morphine can be safely administered together

85. Intussusception the false is


A. Is most common in children from 6 to 12 years
B. Presents with colicky abdominal pain, rectal bleeding and an abdominal mass
C. 10% present with diarrhoea and vomiting suggestive of gastroenteritis
D. If no shock or peritonitis hydrostatic reduction can be attempted
E. A Meckel's diverticulum can induce an intussusception
86. Hirschsprung's disease the false is
A. Is due to absent ganglion cells in Auerbach's plexus
B. 10% cases have involvement of the recto-sigmoid segment
C. 80% cases present with delayed passage of meconium in the first 24 hours after birth
D. The affected segment of bowel appears as cone-shaped contracted zone on barium enema
E. On rectal biopsy there increased acetylcholinesterase containing cells in the muscularis

87. Laparoscopic cholecystectomy the false is


A. Is usually performed using a four port technique
B. The Veress needle is an 'open' technique for inducing the pneumoperitoneum
C. A supraumbilical abdominal scar is a contraindication to laparoscopic cholecystectomy
D. Dissection of Calot's triangle should be performed before the cystic duct is clipped
E. Most series report a conversion rate of less than 5%

88. In obstructive jaundice the false is


A. Urinary conjugated bilirubin is increased
B. Serum unconjugated bilirubin is increased
C. Urinary urobilinogen is reduced
D. Serum conjugated bilirubin is increased
E. Faecal stercobilinogen is reduced

89. Regarding the anatomy of the inguinal canal the false is


A. The internal ring lies midway between the symphysis pubis and anterior superior iliac
spine
B. The internal ring lies medial to the inferior epigastric vessels
C. The external oblique aponeurosis forms the anterior boundary
D. The inguinal ligament forms the inferior boundary
E. The conjoint tendon forms the medial part of the posterior wall

90. Papillary carcinoma of the thyroid the false is


A. Can be reliably diagnosed using fine needle aspiration cytology
B. Is almost always unifocal
C. Histologically displays Psammoma bodies
D. Typically spread to the cervical lymph nodes
E. Requires a total thyroidectomy for large tumours

91. Regarding bladder tumours the false is


A. 90% are squamous carcinomas
B. Painless haematuria is the commonest presentation
C. Cigarette smoking is an important aetiological factor
D. 80% of tumours are superficial (i.e no muscle invasion)
E. Superficial tumours are often well controlled by transurethral resection

92. Regarding ureteric calculi the false is


A. Are most often composed of calcium oxalate or phosphate
B. Most stones of those less than 5 mm in diameter pass spontaneously
C. Extracorporeal lithotripsy is useful for stones in the upper third of the ureter
D. About 30% of patients require open surgery to remove the stone
E. An obstructed ureter in the presence of infection is a surgical emergency

93. Regarding bladder calculi the false is


A. The incidence has fallen markedly in this country since the late 19th century
B. They may be totally asymptomatic
C. They are more prevalent in patients with chronic urinary sepsis
D. They are associated with squamous metaplasia of the bladder mucosa
E. They increase the risk of transitional cell carcinoma

94. Ectopic pregnancy the false is


A. Occurs in about 1% of pregnancies
B. The risk is increased in those with a history or pelvic inflammatory disease
C. Usually presents between 2 and 4 months of gestation
D. Patients usually have a positive pregnancy test
E. if shocked early laparotomy is essential

95. Tetanus the false is


A. Is due to an infection with a gram-positive spore forming rod
B. The organism produces a powerful endotoxin
C. The toxin prevents the release of inhibitory neurotransmitter
D. Clostridium tetani is sensitive to penicillin

96. Regarding pathological terms


A. Hypertrophy is an increase in tissue size due to increased cell number
B. Hyperplasia is an increase in tissue size due to an increase in cell size
C. Atrophy is an increase in tissue size to disuse
D. Metaplasia is a change form one abnormal tissue type to another
E. A hamartoma is a developmental abnormality

_____________________________________________________________________

1. The ilio-inguinal nerve:


A. supplies the rectus abdominis muscle
B. supplies skin on inner side of thigh
C. supplies the cremasteric muscle
D. supplies the urethra
E. does none of the above

2. The skin of the tip of the index finger is supplied by the:


A. Radial nerve only
B. Radial & median nerves
C. Median & ulnar nerves
D. Ulnar nerve only
E. Median nerve only

3. Hypovolaemic shock is characterized by:


A. A low central venous pressure , high cardiac output, low peripheral resistance
B. A high central venous pressure, high cardiac output, high peripheral resistance
C. A low central venous pressure , low cardiac output, high peripheral resistance
D. A low central venous pressure , high cardiac output, high peripheral resistance
E. A high central venous pressure, low cardiac output, low peripheral resistance

4. Which of the following in NOT true of Hodgkin's disease?


A. Usually starts from several groups of nodes simultaneously
B. Usually involves liver & spleen
C. Sometimes manifests itself as pyrexia of unknown origin
D. Severe pain follows ingestion of alcohol
E. Shows increased susceptibility to opportunistic infection

5. Tetanus toxoid:
A. Is produced by injecting animals with antititanic serum
B. Is administered to previously immunized subjects with potentially infected wounds
C. Frequently gives rise to anaphylactic reaction
D. Is used to induce active immunity against tetanus

6. The most probable cause of shock in a patient with multiple injuries & craniocerebral
trauma is:
A. Depression of vital medullary centres
B. Hypoperfuion control over subcortical areas
C. Loss of cortical control over subcortical areas
D. Hypovolaemia
E. Inadequate ADH response

7. The most sensitive guide to acute changes in fluid balance in a surgical patient is:
A. Accurate daily weight
B. Serial serum Na concentration
C. Fluid balance sheets recording inputs & outputs
D. Daily urine output
E. Serial anion gap measurements
8. cellullitis is:
A. Inflammation of the bone marrow
B. Inflammation of the mastoid cells
C. Inflammation of the subcutaneous tissues
D. Infiltration of the skin by gaint cells
E. A malignant condition

9. secondary haemorrhage occurs:


A. Within 6 hours of operation
B. 7-14 days after operation
C. As a result of violent coughing on recovery from anaesthesia
D. Due to a blood transfusion line disconnected
E. When a ligature slips

10. the minimum urine output for 24 hours required to excrete end products of protein
metabolism is:?????????????
A. 200 ml
B. 300 ml
C. 400 ml
D. 500 ml
E. 600 ml

11. Potassium deficiency is present if the plasma-potassium level is:


A. 6.0 mmol/l
B. 5.0 mmol/l
C. 4.5 mmol/l
D. 4.0 mmol/l
E. 3.0 mmol/l

12. in health the pH of the blood lies between the range:


A. pH 7.05-7.19
B. pH 7.20-7.35
C. pH 7.36-744
D. pH 7.45-7.59
E. pH 7.60-7.80

13. TNM classification of a malignant tumour was designed as:


A. An histological staging
B. A clinical staging
C. A staging carried out at operation
D. A staging dependent upon radio scanning & skeletal survey
E. A staging dependent upon ultrasound

14. a blue-green discharge from an ulcer will be seen to contain:


A. Pseudomonas pyocyaneus
B. Streptococcus viridians
C. Candida albicans
D. Staphylococcus aureus
E. Haemophilius influenzae
15. a rodent ulcer is:
A. A squamous cell carcinoma
B. A basal cell carcinoma
C. Only occur on the face
D. Contains epithelial pearls
E. A venous ulcer

3:52
5:34
16. the space of Parona is:
A. In the wrist between the deep flexor tendons & the pronator quadratus
B. Above the patella between the quadriceps muscle & the femur
C. Benath the tendon of the iliopsos
D. Between the achills tendon & the posterior aspect of the tibia
E. The web space of the palm

17. 'rest pain' occurs:


A. Anywhere in the body at rest
B. In the thigh of a patient with Buerger's disease
C. In the calf of a patient with intermittent claudicating
D. In the foot of a patient with severe vascular disease
E. In the back

18. ischaemia means:


A. Pain in the ischial tuberosities
B. Anaemia due to malignant seconderies in the ischial part of the pelvis
C. Lack of blood flow
D. Increased blood flow
E. Polycythaemia
19. Colles' fracture is:
A. A common in adolescence
B. A fracture about the ankle joint
C. Common in elderly women
D. A fracture of the head of the radius
E. A fracture of scaphoid

20. Pott's disease is;


A. A fracture dislocation about the ankle
B. A neuropathic joint
C. Traumatic ostechondritis of the spine
D. Tuberculosis of the spine
E. A secondary tumour in the skull

21. Vincent's angina is a form of angina associated with:


A. Spasm of the oesophagus
B. Diphtheria
C. An infection of the mouth
D. Coronary artery spasm
E. Carcinoma of the bronchus
22. Ludwig's angina is due to :
A. A type of coronary artery spasm
B. Oesophageal spasm
C. Retropharyngeal infection
D. A virulent infection of the cellular tissue around the submandibular salivary gland
E. Infection with candida

23. in simple nodular goiter:


A. Carcinoma occurs in 30% of cases
B. The nodular stage is irreversible
C. Operation is contraindicated
D. The patient does not develop hyperthyroidism
E. Cretinism is the presenting feature

24. Hashimoto's disease is:


A. A granulomatous thyroiditis
B. An auto-immune thyroiditis
C. An infiltrating fibrosis of the thyrois & the adjacent muscles
D. Focal thyroiditis
E. A parathyroid tumour
25. A thyroglossal fistula:
A. Is never congenital
B. Follows inadequate removal of a thyroglossal cyst
C. Has a hood of skin with its concavity upwards
D. Is lined throughout by squamous epithelium
E. Occurs in carcinoma of the tongue

26. The following are clinical signs supporting an early diagnosis of carcinoma of the
breast:
A. A prickling sensation in a breast lump
B. Peau d'ornge
C. Brawny arm
D. Cancer en cuirasse
E. A krukenderg tumour

27. The gastroduodenal artery is a branch of the:


A. Celiac axis
B. Hepatic artery
C. Superior mesenteric artery
D. Gastroepiploic artery
E. Splenic artery

28. Chronic gastric ulcers most often occur in patients with:


A. Blood groub A
B. Tend to occur in alkaline mucosa
C. Muscularis mucosae is separated from the muscularis at the edge of the ulcer
D. Are malignant when there is epithelial proliferation & downgrowths
E. Are never large
29. Meckel's diverticulum:
A. Is present in 20% of the human race
B. Arises from the mesenteric border of the jejunum
C. May contain heterotopic pancreas
D. Is only present in the male sex
E. Is a diverticulum of the bladder

30. Intussusception is related to:


A. Mucoviscidosis
B. Swollen Peyer's patches
C. Volvulus
D. A littre's hernia
E. A patent vitello intestinal duct
31. The site of the neck of a femoral hernia is the:
A. Transversalia fascia
B. Iliopectineal ligament
C. Femoral ring
D. Cribriform fascia
E. Obturator foramen

32. Regarding operation for an indirect inguinal hernia:


A. It should not be performed on patients who have chronic bronchitis
B. General anaesthesia has to be used
C. In infants the posterior inguinal wall should be repaired
D. In adults the internal inguinal ring usually needs to be strengthened
E. Mesh implants are mandatory

33. Cystic fibrosis of the pancreas


A. is inherited as autosomal recessive
B. islets of Langerhans are affected
C. diagnosis is pssible by DNA analysis
D. all the above
E. a and c only

34. the following are clinical criteria of Bank ad Wise


A. pulmonary
B. neurological
C. renal
D. all the above
E. a and c only

35. antibiotics in acute pancreatitis


A. must be given to all cases
B. should not be given
C. given only to modarate & sever cases
D. should cover anaerobic & Gam positive bactria
E. none of the above
36. pancreatic carcinoma
A. occure usually at the age of 50years
B. usually is cystadenocarcinoma
C. more favaorable in the body and tail
D. less favourable if occure with back pain
E. all the above
37. infected pancreatic necrosis
A. is a collection of pus around the pancreas
B. diagnosed by CT scan
C. more serious than pancreatic abscess
D. all the above
E. none of the above

38. secondarey survay in polytrauma


A. should be done in the first hour
B. done as soon as the patient is stable
C. detects life threateing problems
D. a and c
E. none of the above

39. spontaneous ruptur of the spleen


A. occure less in tropical countries
B. the spleen may be enlarged
C. malaria is a common cause
D. all the above

40. spleic aretry aneurysm


A. incidence is around 5%
B. usually symptomless
C. more conmmon in males
D. all the above

41. wolf skin graft


A. used to cover large area of burn
B. success rate is less than split thckness graft
C. both thighs are common donor areas
D. usually contracts

42. medullary carcinoma of the thyroid


A. is a tumor of the G cells
B. familial in 70% of the cases
C. diagosed by low level of calcitonin
D. amyloid stroma is charactrestic
43. solitary thyroid nodule
A. 70% are follicular adenoma
B. 30% are malignant
C. more favourable in young males
D. all the above
E. none of the above

44. rupture of the diaphragm


A. is a life threatenig problem
B. usually missed
C. poly galctan suture is used for repair
D. all the above
E. noe of the above

45. Complication of vascular graft includes All the following except:


A. Infection
B. Aneurysm
C. Graft failure
D. Hemorrhage
E. a & c only

46. Gangrene:
A. is necrosis of tissue
B. The cause may be venous occlusion
C. usually painful
D. All the above
E. None of the above

47. Lynphedema:
A. may be Congenital
B. should be bilateral
C. may be pitting in early stage
D. A & C only
E. None of the above

48. Suprapubic cystostomy:


A. indicated in case of bladder out flow obstruction
B. indicated in case of urethral injury
C. indicated in case of urethral stricture
D. All the above
E. B& C only

49. differential diagnosis of Acute scrotal swelling in young adult includes all the follow
except:
A. Incarcerated inguinal hernia .
B. Torsion of testes
C. Acute epididymo- orchitis
D. Teratoma
E. Seminoma

50. Horizontal spread of infection across the external sphincter can result in which type of
anorectal abscess:
A. ischiorectal
B. perianal
C. supralevator
D. intersphincteric
E. intermuscular

51. All of the following statements are true of diffuse esophageal spasm EXCEPT:
A. chest pain is frequently seen
B. high amplitude esophageal contraction are present.
C. it is best diagnosed with barium esophogram.
D. usual surgical treatment is long esohagomyotomy.
E. most patients do not have significant coronary artery disease.

52. The treatment of an esophageal burn with a caustic agent may include all of the
following EXCEPT:
A. expeditious administration of an antidote.
B. induction of vomiting.
C. steroids and antibiotics.
D. bougienage.
E. gastrectomy.

53. All of the following substances are irritating to the peritoneum EXCEPT:
A. bile.
B. meconium.
C. blood.
D. gastric content.
E. pus.
54. Complications of truncal vagotomy and pyloroplasty include all of the following
EXCEPT:
A. dumping syndrome.
B. recurrent ulcer.
C. diarrhea.
D. alkaline reflux gastritis.
E. steatorrhea.

55. Gastric polyps:


A. are most commonly adenomatous.
B. require gastrotomy and removal if greater than 2 cm and are pedunculated.
C. are rarely multiple.
D. are clearly premalignant.
E. are more frequent in achlorhydric patients.

56. Vascular compression of the duodenum resulting in obstruction:


A. is present primarily in patients who are overweight.
B. should be given a trial of conservative management.
C. is common in pediatric patients.
D. is best diagnosed by identifying a "double bubble" sign on abdominal x-ray.
E. includes as medical therapy lying in the supine position after meals
57. Conditions associated with gastric cancer include all of the following EXCEPT:
A. higher socioeconomic groups.
B. pernicious anemia.
C. chronic atrophic gastritis.
D. adenomatous polyps.
E. a high intake of dietary nitrates

58. Patients with morbid obesity have an increased incidence of all of the following
EXCEPT:
A. gastric carcinoma.
B. diabetes.
C. stroke
D. gallbladder disease.
E. joint deterioration.

59. All of the following contribute to malabsorption following truncal vagotomy and
antrectomy EXCEPT:
A. increased rate of gastric emptying.
B. poor mixing of pancreatic secretions and bile salts with food.
C. increased release of secretions and bile salts with food.
D. decreased small intestinal transit time.
E. malabsorption of fat and carbohydrates.
60. All of the following statements are true about patients with carcinoid tumors EXCEPT:
A. they often have evidence of serotonin production.
B. tumor growth is often slow.
C. the majority have carcinoid syndrome.
D. they have a much better prognosis if the tumors are less than 2 cm.
E. the combination of streptozotocin and 5-fluorouracil can often result in objective response.

61. The followings are usually associated with acute appendicitis EXCEPT
A. Abdominal pain and nausea
B. CT scan with water soluble enema is the most helpful imaging study
C. Deep right lower abdominal tenderness by rectal examination
D. Positive Rovsing sign
E. WBCs around 20,000/mm3

62. The least problem that cause right lower abdominal pain in a 18 years female is
A. Ectopic pregnancy
B. Acute appendicitis
C. Ovarian torsion
D. Perforated peptic ulcer
E. Mittleschmerz

63. A patient with high hichitic fever, severe tenderness and rigidity at the right ileac fossa,
WBCs 18000/mm3 and abdominal ultrasound showed a heterogeneous mass in the right
iliac fossa with a central fluid collection. Management may include any of the followings
EXCEPT
A. Open drainage
B. I.V antibiotics
C. Appendix should be resected in the open drainage
D. Percutaneous drainage under U/S or CT guidance
E. Oral feeding shouldn’t be delayed in the absence of ileus

64. Regarding the lower esophageal sphincter the following are correct EXCEPT
A. It is a physiological sphincter
B. Located in the distal 3-5cm of the esophagus
C. Has a resting pressure of 20-60 mmHg (10-20)
D. Abdominal pressure play a role
E. Its pressure can be estimated by esophageal manometry
65. In esophageal perforation, the most sensitive diagnostic study is
A. Plain film abdomen
B. Plain film chest and neck
C. Esophagogram
D. Esophagoscopy
E. CT chest and neck

66. Achalasia can be presented with all of the followings EXCEPT


A. Recurrent pulmonary infections
B. Weight loss
C. Regurgitation
D. Irregular narrowing of the distal esophagus by Ba. Swallow
E. Retrosternal chest pain

67. Complications of reflux esophagitis include the followings EXCEPT


A. Dysmotility
B. Schatzki's ring
C. Barrett's esophagus
D. Epiphrenic esophageal diverticulum
E. Hemorrhage

6:16
6:45
68. The pressure in the lower esophageal sphincter decreases by all of the following
EXCEPT
A. Alcohol
B. Nitroglycerin
C. Anticholinergics
D. Alpha adrenergics
E. Cholecystokinin hormone

69. The genetic predisposing factors to gastric cancer include the followings EXCEPT
A. Family history of gastric cancer
B. Black race
C. P 53 mutation
D. Germline mutation of e-cadherin
E. BRCA2 mutation

70. Regarding the diffuse gastric cancer, the followings are true EXCEPT??
A. The commonest type of gastric adenocarcinoma
B. Not associated with intestinal metaplasia
C. More incidence in young ages
D. Less related to environmental influences
E. Results from single cell mutation

71. The best diagnostic study for gastric adenocarcinoma


A. Upper endoscopy
B. Endoscopic ultrasound
C. Upper gastrointestinal double contrast barium study
D. Laparoscopy
E. Abdominal CT scanning

72. In primary gastrointestinal stromal tumors (GIST), which is NOT true


A. The most common site is the stomach
B. Bleeding is the commonest manifestation
C. Almost never metastasize to regional lymph nodes
D. The traditional cytotoxic chemotherapy greatly suppresses its growth
E. Endoscopic ultrasound guided FNAC gives the definitive diagnosis

73. Primary gastric lymphoma


A. 2% of all hodgkin's lymphoma
B. Greatly differs in presentation from gastric adenocarcinoma
C. One third of all gastrointestinal lymphomas
D. The most common extranodal lymphoma
E. Worse prognosis than adenocarcinoma

74. In gastric adenocarcinoma which is NOT true


A. Risk increased 3—6 times in patient with gastric H pylori infection
B. Blumer's shelf results from omental invasion in pelvic cavity
C. Krukenburg's tumor is due to ovarian metastasis
D. Weight loss and vague abdominal pain are the commonest presentation
E. Endoscopic ultrasound is 90% accurate in determining T stage

75. Mucosa associated lymphoid tissue ( MALT ) is found in all of the followings EXCEPT
A. Small bowel ( Peyer's patches )
B. Waldeyer's ring
C. Appendix
D. Stomach
E. Bronchus

76. All of the following are associated with Barrett's esophagus EXCEPT
A. GERD
B. Squamous carcinoma
C. Esophageal mucosal dysplasia
D. Increased incidence of p53 mutations
E. adenocarcinoma

77. The best test to establish the presence of gastroesophageal reflux (GERD) disease is
A. An upper gastrointestinal series
B. Bernstein test (acid perfusion)
C. 24-hour pH monitoring
D. Esophageal manometry
E. Endoscopic biopsy

78. The T and N status of esophageal carcinoma is most accurately assessed by


A. Upper gastrointestinal series
B. Computed tomographic scan of the chest with double contrast
C. Endoscopic ultrasound (EUS)
D. Positron emission tomography (PET scan)
E. Magnetic resonance imaging (MRI)

79. Salivary gland stones


A. Most arise in the sublingual gland
B. Usually present with persistent pain
C. Are composed predominantly of magnesium phosphate
D. Predispose to infection of the involved gland
E. Never occur in parotid duct

80. 35-year-old alcoholic is admitted with acute pancreatitis. He complains of Numbness of


his fingers and toes. On examination he has hyperactive Tendon reflexes. The most likely
cause of these symptoms is
A. Hyponatremia
B. Hypocalcemia
C. Hypophosphatemia
D. Hypermagnesemia
E. Hyperkalemia

81. Regarding benign salivary gland adenomas


A. Pleomorphic adenomas are only seen in parotid gland
B. Pleomorphic adenomas cannot undergo malignant change
C. Warthin's tumor is otherwise known as an adenolymphoma
D. Adenolymphomas usually occur in young men
E. Adenolymphomas are often bilateral
82. Massive lower gastrointestinal hemorrhage
A. Rarely stops after resuscitation
B. Is caused by large bowel lesions only
C. can be most accurately localized by colonoscopy
D. Is frequently related to right colon lesions
E. Is most commonly caused by adenocarcinoma of the large intestine

83. Regarding carcinoid tumors all of the following are true EXCEPT
A. Liver metastases can result in the carcinoid syndrome
B. The appendix is the commonest primary site for gastrointestinal tumors
C. Gastric carcinoid tumors produce little 5-hydroxyindoleacetic acid
D. If discovered in the appendix right hemicolectomy should always be done
E. Octreotide scintigraphy may identify both the primary and secondary lesions

84. A Meckel's diverticulum


A. Occurs in 10% of the population
B. Will be found on the mesenteric border of the small intestine
C. Consists of mucosa without a muscle coat
D. Usually found 20 cm from ileocecal valve
E. A fibrous band between the apex and umbilicus can cause intestinal obstruction

85. In overwhelming post-splenectomy infection, which is NOT true


A. Is usually due to unencapsulated bacterial infection
B. Strep. pneumonia is the commonest etiological agent
C. Despite aggressive therapy it can have a mortality of over 50%
D. The risk of infection can be reduced with pneumococcal and hemophilus vaccination
E. Penicillin antibiotic prophylaxis should be considered in all children

86. Clostridium tetani


A. Is a gram-negative rod
B. Is sensitive to penicillin
C. Is available in the expired canned food
D. Releases a heat-resistant endotoxin
E. The toxin acts on the post-synaptic membrane of inhibitory nerve fibers

87. Grade I hypovolemic shock


A. Occurs when more than 40% of the circulating blood volume has been lost
B. Tachycardia is a reliable clinical sign
C. A fall in pulse pressure is observed
D. Urine output is markedly reduced
E. Capillary return is delayed

88. Regarding Hydatid disease the followings are true EXCEPT


A. Recently surgery is no more the treatment of choice in liver disease
B. Man is an accidental intermediate host
C. The liver is the commonest site of infection
D. Can be diagnosed by the ELISA test
E. Treatment by benzimidazoles alone is 30% successful

89. In peptic ulcer disease??


A. H. pylori is a gram-positive bacillus
B. Surgery is always indicated in perforated ulcers
C. Sepsis after perforation is the commonest cause of death
D. Parietal cell vagotomy carries the highest rate of recurrence
E. Gastric drainage should be done in all types of vagotomies
90. Stones in the common bile duct, which is NOT true
A. Are mostly secondary stones
B. Can present with Charcot's Triad
C. Are suggested by a bile duct diameter >8mm on ultrasound
D. ERCP, sphincterotomy and balloon clearance is now the treatment of choice
E. If removed by exploration of the common bile duct the T-tube can be removed after 3 days

91. Regarding pancreatic carcinoma the followings are true EXCEPT


A. 90% are ductal adenocarcinomas
B. Less than 20% occur in the head of the gland
C. The usual presentation is with pain, weight loss and obstructive jaundice
D. Ultrasound has a sensitivity of 80-90% in the detection of the tumor
E. Less than 20% of patients are suitable for curative surgery

92. Regarding anal fissures all are true EXCEPT


A. 10% occur in the posterior midline
B. Multiple fissures suggest a diagnosis of tuberculosis or Crohn's Disease
C. 50% of acute fissures heal with the use of a bulking agent
D. Sphincterotomy has a success rate of over 90%
E. Sphincterotomy is associated with minor fecal incontinence in over 15% of patients

93. Medullary carcinoma of the thyroid


A. Concentrates Iodine 131
B. Produce thyroxine as the principle hormone
C. 10% of cases are sporadic
D. 90% can occur in association with MEN type II syndrome
E. All patients will need a 24-hour VMA level in urine

94. Which of the following is LEAST likely to be associated with the systemic inflammatory
response syndrome (SIRS)
A. Infection
B. Elevated/depressed temperature
C. Elevated heart rate
D. Elevated respiratory rate
E. Elevated/depressed WBC count

95. In papillary carcinoma of the thyroid the followings are true EXCEPT
A. Can be reliably diagnosed using fine needle aspiration cytology
B. Is almost always unifocal
C. Histologically displays Psammoma bodies
D. Typically spread to the cervical lymph nodes
E. Requires a total thyroidectomy for large tumors

96. Serum alpha-fetoprotein is increased in the following EXCEPT


A. Acute hepatitis
B. Hepatocellular carcinoma
C. Neuroblastoma
D. Teratomas
E. Bladder carcinoma
97. The following predispose to wound infection EXCEPT
A. Malnutrition
B. Hypovolemia
C. Malignancy
D. Obstructive jaundice
E. Steroid therapy

98. Regarding acute respiratory distress syndrome (ARDS) which is NOT true
A. Hypoxia in spite of high inspired oxygen
B. Increased lung compliance
C. Non-cardiac edema
D. Diffuse or patchy infiltrates in chest X ray
E. Deposition of proteinaceous fluid in the respiratory membrane

99. Which of the following statements about serum thyroglobulin is TRUE?


A. Elevation after total thyroidectomy justify the use of iodine131 therapy
B. It suppresses thyroid-stimulating hormone (TSH)
C. It suppresses thyroid-releasing hormone (TRH)
D. It is an effective tumor marker because it is specifically elaborated by the malignant cells
of papillary cancer
E. It is only about 50% effective in detecting recurrence

100. Women who have hereditary nonpolyposis colorectal cancer (HNPCC) should also be
screened for
A. Endometrial cancer
B. Papillary cancer of thyroid
C. Ampullary cancer
D. Pheochromocytoma
E. Hepatoma

________________________________________________________________________

1. Which is the shortest phase of the normal cell cycle?


A. G1 phase
B. S phase
C. G2 phase
D. M phase
E. All phases approximately equal in length

2. A 67 year-old women with rectal cancer is admitted to gereral surgical floor which of the
following laboratory studies should be included in the surgeon’s initial nutritional
assessment :
A. Transferrin
B. Prealbumin
C. Albumin
D. Glutamine
E. All of above

3. In which of the following conditions is the entral route appropriate for nutrition :
A. Upper gastrointestinal obstruction
B. Complete small bowel obstruction
C. Acute flare-up of Crohn’s disease
D. Low out put colonic fistula
E. Non of the above

4. Which is the most commonly cultured hospital acquired organism in critical care with
aspiration pneumonia:
A. Staphylococcus aureus
B. Streptococcus pneumonia
C. Anaerobic species
D. Pseudomonas aeroginosa
E. Haemophlus influenzae

5. Which is the most appropriate single agent for empiric coverage of the above patient :
A. Metranidazole
B. Clindamycin
C. Pipracillin_tazobactam
D. Vancomycin
E. First generation penicillin

6. All of the following are true about neurogenic shock except:


A. There is a decrease in systemic vascular resistance and an increase in venous capacitance.
B. Tachycardia or bradycardia may be observed, along with hypotension.
C. The use of an alpha agonist such as phenylephrine is the mainstay of treatment.
D. Severe head injury, spinal cord injury, and high spinal anesthesia may all cause
neurogenic shock.
E. A and B

7. Which of the following statements about head injury and concomitant hyponatremia are
true?
A. There are no primary alterations in cardiovascular signs.
B. Signs of increased intracranial pressure may be masked by the hyponatremia.
C. Oliguric renal failure is an unlikely complication.
D. Rapid correction of the hyponatremia may prevent central pontine injury.
E. This patient is best treated by restriction of water intake

8. Regarding Cushing's Syndrome


A. 20% of cases are due to pituitary adenomas (Cushing's Disease)
B. Most ACTH secreting pituitary adenomas are more than 2 cm in diameter
C. Is characterised by loss of the diurnal rhythm of cortisol secretion
D. Cortisol production is suppressed by low-dose dexamethasone
E. Adrenal carcinomas are more common than adrenal adenomas

9. Which of the following statements about extracellular fluid are true?


A. The total extracellular fluid volume represents 40% of the body weight.
B. The plasma volume constitutes one fourth of the total extracellular fluid volume.
C. Potassium is the principal cation in extracellular fluid.
D. The protein content of the plasma produces a lower concentration of cations than in the
interstitial fluid.
E. The interstitial fluid equilibrates slowly with the other body compartments.

10. In patients receiving massive blood transfusion for acute blood loss, which of the
following is/are correct?
A. Packed red blood cells and crystalloid solution should be infused to restore oxygen-
carrying capacity and intravascular volume.
B. Two units of FFP should be given with every 5 units of packed red blood cells in most
cases.
C. A “six pack” of platelets should be administered with every 10 units of packed red blood
cells in most cases.
D. One to two ampules of sodium bicarbonate should be administered with every 5 units of
packed red blood cells to avoid acidosis.
E. One ampule of calcium chloride should be administered with every 5 units of packed red
blood cells to avoid hypocalcemia.

11. Which of the following statements about the presence of gallstones in diabetes patients
is/are correct?
A. Gallstones occur with the same frequency in diabetes patients as in the healthy population.
B. The presence of gallstones, regardless of the presence of symptoms, is an indication for
cholecystectomy in a diabetes patient.
C. Diabetes patients with gallstones and chronic biliary pain should be managed
nonoperatively with chemical dissolution and/or lithotripsy because of severe complicating
medical conditions and a high operative risk.
D. The presence of diabetes and gallstones places the patient at high risk for pancreatic
cancer.
E. Diabetes patients with symptomatic gallstones should have prompt elective
cholecystectomy, to avoid the complications of acute cholecystitis and gallbladder necrosis.

12. concerning Tetanus all true except:


A. Is due to an infection with a gram-negative spore forming rod
B. The organism produces a powerful exdotoxin
C. The toxin prevents the release of inhibitory neurotransmitter
D. Clostridium tetani is sensitive to penicillin
E. Risus sardonicus is the typical facial spasm
13. when should parentral antibiotics be given perioperatively?
A. The night before
B. 6 hr prior to surgery
C. 30 minutes prior to incision .
D. at the time of incision
E. 30 minutes after incision

14. Which of the following statements about esophageal anatomy is correct?


A. The esophagus has a poor blood supply, which is segmental in distribution and accounts
for the high incidence of anastomotic leakage.
B. The esophageal serosa consists of a thin layer of fibroareolar tissue.
C. The esophagus has two distinct muscle layers, an outer, longitudinal one and an inner,
circular one, which are striated in the upper third and smooth in the distal two thirds.
D. Injury to the recurrent laryngeal nerve results in vocal cord dysfunction but does not affect
swallowing.
E. The lymphatic drainage of the esophagus is relatively sparse, localized primarily to
adjacent paraesophageal lymph nodes.

15. Wich of the following medication should be given in preparation of a patient with
pheochromocytoma?
A. Phnoxybenzamine
B. Nifedipine
C. Linsinopril
D. Hydrochlorothiazide
E. Propranolol

16. Which of the following statement(s) is true concerning excessive scarring processes?
A. Keloids occur randomly regardless of gender or race
B. Hypertrophic scars and keloid are histologically different
C. Keloids tend to develop early and hypertrophic scars late after the surgical injury
D. Simple reexcision and closure of a hypertrophic scar can be useful in certain situations
such as a wound closed by secondary intention
E. Non of the above

17. A 22-year-old man sustains a single stab wound to the left chest and presents to the
emergency room with hypotension. Which of the following statement(s) is true concerning
his diagnosis and management?
A. The patient likely is suffering from hypovolemic shock and should respond quickly to
fluid resuscitation
B. Beck’s triad will likely be an obvious indication of compressive cardiogenic shock due to
pericardial tamponade
C. Echocardiography is the most sensitive noninvasive approach for diagnosis of pericardial
tamponade
D. The placement of bilateral chest tubes will likely resolve the problem

18. Which of the following statement(s) is/are true concerning septic shock?
A. The clinical picture of gram negative septic shock is specifically different than shock
associated with other infectious agents
B. The circulatory derangements of septic shock precede the development of metabolic
abnormalities
C. Splanchnic vascular resistance falls in similar fashion to overall systemic vascular
resistance
D. Despite normal mechanisms of intrinsic expansion of the circulating blood volume,
exogenous volume resuscitation is necessary

19. During surgery on the submandibular gland


A. An incision on the lower border of the mandible is safe
B. The submandibular gland is seen to wrap around the posterior border of mylohyoid
C. The facial artery and vein are divided as they course through the deep part of the gland
D. The hypoglossal nerve is seen to loop under the submandibular duct
E. Damage to the lingual nerve will cause loss of sensation to the posterior third of the
tongue
7:29
10:12
20. Regarding benign breast disease (except)
A. Cyclical mastalgia is the commonest reason for referral to the breast clinic
B. Fibroadenomas are derived from the breast lobule
C. Lactational breast abscesses are usually due to Staph aureus
D. Duct ectasia is more common in smokers
E. Atypical lobular hyperplasia is associated with an decreased risk of breast cancer

21. Which of the following statements regarding unusual hernias is incorrect?


A. An obturator hernia may produce nerve compression diagnosed by a positive Howship-
Romberg sign.
B. Grynfeltt's hernia appears through the superior lumbar triangle, whereas Petit's hernia
occurs through the inferior lumbar triangle.
C. Sciatic hernias usually present with a painful groin mass below the inguinal ligament.
D. Littre's hernia is defined by a Meckel's diverticulum presenting as the sole component of
the hernia sac.
E. Richter's hernia involves the antimesenteric surface of the intestine within the hernia sac
and may present with partial intestinal obstruction
22. Staples may safely be placed during laparoscopic hernia repair in each of the following
structures except:
A. Cooper's ligament.
B. Tissues superior to the lateral iliopubic tract.
C. The transversus abdominis aponeurotic arch.
D. Tissues inferior to the lateral iliopubic tract.
E. The iliopubic tract at its insertion onto Cooper's ligament.

23. The following statements about the repair of inguinal hernias are true except:
A. The conjoined tendon is sutured to Cooper's ligament in the Bassini hernia repair.
B. The McVay repair is a suitable option for the repair of femoral hernias.
C. The Shouldice repair involves a multilayer, imbricated repair of the floor of the inguinal
canal.
D. The Lichtenstein repair is accomplished by prosthetic mesh repair of the inguinal canal
floor in a tension-free manner.
E. The laparoscopic transabdominal preperitoneal (TAPP) and totally extraperitoneal
approach (TEPA) repairs are based on the preperitoneal repairs of Cheattle, Henry, Nyhus, and
Stoppa.
24. A number of special circumstances exist in the repair of inguinal hernias. The following
statement(s) is correct.
A. Simultaneous repair of bilateral direct inguinal hernias can be performed with no
significant increased risk of recurrence
B. The preperitoneal approach may be appropriate for repair of a multiple recurrent hernia
C. A femoral hernia repair can best be accomplished using a Bassini or Shouldice repair
D. Management of an incarcerated inguinal hernia with obstruction is best approached via
laparotomy incision
E. All are correct.

25. Which of the following statement(s) is true about benign lesions of the liver?
A. Adenomas are true neoplasms with a predisposition for complications and should usually
be resected.
B. Focal nodular hyperplasia (FNH) is a neoplasm related to birth control pills (BCPs) and
usually requires resection.
C. Hemangiomas are the most common benign lesions of the liver that come to the surgeon's
attention.
D. Nodular regenerative hyperplasia does not usually accompany cirrhosis

26. Ligation of all of the following arteries usually causes significant hepatic enzyme
abnormalities except:
A. Ligation of the right hepatic artery.
B. Ligation of the left hepatic artery.
C. Ligation of the hepatic artery distal to the gastro-duodenal branch.
D. Ligation of the hepatic artery proximal to the gastroduodenal artery.

27. Which of the following is the most effective definitive therapy for both prevention of
recurrent variceal hemorrhage and control of ascites?
A. Endoscopic sclerotherapy.
B. Distal splenorenal shunt.
C. Esophagogastric devascularization (Sugiura procedure).
D. Side-to-side portacaval shunt.
E. End-to-side portacaval shunt.

28. which of the following is associated with best prognosis for patient with breast cancer?
A. Male sex
B. Estrogen receptor positive
C. Patient age <35 years
D. Pregnant patient
E. Tumor with overexpression of HER/ner.

29. A 49-year-old women has a palpable breast mass in the upper outer quadrant. The size
of the mass has increased over the last month . exicisional biopsy reveals cystic carcinoma
with invasion .appropriate management now would be :
A. Re-excision with wide margins
B. Axillary node dissection and hormonal therapy
C. Simple mastectomy
D. Modified radical mastectomy
E. Bilateral mastectomies

30. A contraindication to stereotactic core biopsy of the breast is the mammographic


presence of:
A. Microcacification
B. A radial scar
C. A nonpalpable mass lesion
D. Lesions<8 mm in diameter
E. Mutifocal lesions.

31. Which of the following statements about epiphrenic diverticula of the esophagus is/are
correct?
A. They are traction diverticula that arise close to the tracheobronchial tree.
B. They characteristically arise proximal to an esophageal reflux stricture.
C. The degree of dysphagia correlates with the size of the pouch.
D. They are best approached surgically through a right thoracotomy.
E. The operation of choice is a stapled diverticulectomy, long esophagomyotomy, and partial
fundoplication.

32. Which of the following statements about Schatzki's ring is correct?


A. The ring represents a panmural fibrotic stricture resulting from gastroesophageal reflux.
B. Dysphagia occurs when the ring diameter is 13 mm. or less.
C. The ring occurs within 1 to 2 cm. of the squamocolumnar epithelial junction.
D. Schatzki's ring indicates reflux esophagitis.
E. Schatzki's ring signifies the need for an antireflux operation.

33. Which of the following statements about pathology encountered at esophagoscopy is/are
correct?
A. Reflux esophagitis should be graded as mild, moderate, or severe, to promote consistency
among different observers.
B. An esophageal reflux stricture with a 2-mm. lumen is not dilatable and is best treated with
resection.
C. A newly diagnosed radiographic distal esophageal stricture warrants dilation and
antireflux medical therapy.
D. In patients with Barrett's mucosa, the squamocolumnar epithelial junction occurs 3 cm. or
more proximal to the anatomic esophagogastric junction.
E. After fasting at least 12 hours, a patient with megaesophagus of achalasia can safely
undergo flexible fiberoptic esophagoscopy.

34. Infantile hypertrophic pyloric stenosis


A. Occurs with a female : male ratio of 4:1.
B. Sons of affected mothers have a 20% risk of developing the lesion
C. Invariably presents between six and eights months of age
D. Typically presents with bile stained projectile vomiting
E. Surgical treatment is by Heller's Cardiomyotom

35. Which of the following statements regarding the pathology of esophageal carcinoma
is/are correct?
A. Worldwide, adenocarcinoma is the most common esophageal malignancy.
B. Squamous cell carcinoma is most common in the distal esophagus, whereas
adenocarcinoma predominates in the middle third.
C. Patients with Barrett's metaplasia are 40 times more likely than the general population to
develop adenocarcinoma.
D. Metastases from esophageal carcinoma are characteristically localized to regional
mediastinal lymph nodes adjacent to the tumor.
E. Achalasia, radiation esophagitis, caustic esophageal stricture, Barrett's mucosa, and
Plummer-Vinson syndrome are all premalignant esophageal lesions that predispose to the
development of squamous cell carcinoma.

36. 45-year-old with isolated 6-cm colorectal metastasis in the liver 2 years after colectomy,
otherwise healthy pest treatment would be:
A. Radiofrequency ablation
B. Systemic chemotherapy
C. Hepatic lobectomy
D. Liver transplantation
E. Cryosurgical ablation

37. Oesophageal atresia all true except:


A. Is often associated with a distal trachea-oesophageal fistula
B. Polyhydramnios is often present late in pregnancy
C. 50% have other associated congenital abnormalities
D. Contrast X-ray studies are necessary to confirm the diagnosis
E. Post-operatively over 30% develop oesophageal strictures

38. All are true about the dumping syndrome except:


A. Symptoms can be controlled with a somatostatin analog.
B. Diarrhea is always part of the dumping syndrome.
C. Flushing and tachycardia are common features of the syndrome.
D. Separating solids and liquids in the patient's oral intake alleviates some of the symptoms
of the syndrome.
E. Early postoperative dumping after vagotomy often resolves spontaneously.

39. Which of the following statements about gastric polyps is/are true?
A. Like their colonic counterparts, gastric epithelial polyps are common tumors.
B. They are analogous to colorectal polyps in natural history.
C. Endoscopy can uniformly predict the histology of a polyp based on location and
appearance.
D. In a given patient, multiple polyps are generally of a multiple histologic type.
E. Gastric adenomatous polyps greater than 2 cm. in diameter should be excised because of
the risk of malignant transformation.

40. All of the following statements about surgical management of gastric lymphomas are
true except:
A. Stage I gastric lymphomas (small lesions confined to the stomach wall) can be cured
completely with surgical therapy alone.
B. Extensive gastric lymphomas that initially are treated with radiation and/or chemotherapy
occasionally perforate during treatment and require secondary resection.
C. Patients explored with a presumptive diagnosis of gastric lymphoma should undergo an
attempt at curative resection when this is safe and feasible.
D. Without a preoperative diagnosis resection for gastric mass should not be attempted unless
lymphoma can be excluded.
E. Appropriate staging for primary gastric lymphoma includes bone marrow biopsy.

41. the most accurate test to confirm diagnosis of infected necrotizing pancreases is:
A. Abdominal ultrasound study
B. Indium-labeled leeukocte scan
C. Cimputed tomographic scan
D. Elevated serum level of interleukain 6 and 8
E. Percutaneous needle aspiration

42. Which of the following variables best predicts prognosis for patients with a recent
diagnosis of cutaneous melanoma and no clinical evidence of metastatic disease?
A. Breslow thickness.
B. Clark's level.
C. Ulceration.
D. Gender.
E. Celtic complexion.

43. the following are true about intracranial tumors except:


A. The most common location of brain tumors of childhood is the posterior cranial fossa.
B. With few exceptions, examination of the CSF is of no value in the diagnosis of an
intracranial tumor.
C. Even the most malignant of primary brain tumors seldom spread outside the confines of
the central nervous system (CNS).
D. The majority of astrocytomas can be cured surgically.
E. Primary neoplasms of astrocytic, oligodendroglial, or ependymal origin represent
gradations of a spectrum from slowly growing to rapidly growing neoplasms.
44. A right-sided disc herniation at the L5–S1 level typically may cause:
A. Low back pain and left sciatica.
B. Weakness of dorsiflexion of the right foot.
C. A diminished or absent right ankle jerk.
D. Diminution of sensation over the medial aspect of the right foot, including the great toe.
E. Weakness of dorsiflexion of the left foot.

45. The preferred operation for initial management of a thyroid nodule that is considered
suspicious for malignancy by FNAB is:
A. Excision.
B. Partial lobectomy.
C. Total lobectomy and isthmusectomy.
D. Total thyroidectomy.
E. All methods are correct
46. the most common presentation of Meckel,s diverticulum in an adult is:
A. GIT bleeding
B. GIT obstruction
C. Intussuception
D. Litter,s hernia
E. Diverticulitis

47. Optimal front-line treatment of squamous cell carcinoma of the rectum includes:
A. Abdominal perineal resection.
B. Low anterior resection when technically feasible.
C. Radiation therapy.
D. Chemotherapy.
E. Combined radiation and chemotherapy.

48. 65-year-old man presents with complaints of mucous discharge and perianal
discomfort. Physical examination reveals a fistulous opening lateral to the anus. Anoscopic
examination permits passage of a probe through the fistula tract. The fistula traverses the
internal anal sphincter, the intersphincteric plane, and a portion of the external anal
sphincter. The fistula is categorized as which type?
A. Intersphincteric
B. Transsphincteric
C. Suprasphincteric
D. Extrasphincteric
E. Non of the above

49. Warthin's tumor:


A. Is a pleomorphic adenoma of salivary gland
B. Should be treated by total paritidectomy
C. Is considered a benign salivary gland neoplasia
D. Respond well to preoperative radiotherapy
E. Often present with facial nerve compression

50. A 38 year old woman presents with right upper quadrant pain and bouts of vomiting.
She is known to have gallstones and has had similar episodes in the past. Which of the
following might support a diagnosis of acute cholecystitis rather than biliary colic
A. duration of symptoms
B. Severity of vomiting
C. Presence of Murphy's sign
D. Presence of gas under right hemidiaphragm on erect CXR.

51. Adequate urine out put for adult postoperative surgical patient is greater than:
A. 35 ml per hour regardless of body size
B. 50 ml per hour regardless of body size
C. 0.5 ml per kg. per hour
D. 1 ml per kg. per hour

52. the MOST important finding in the diagnosis of acute appendicitis is:
A. vomiting
B. Fever
C. leukocytosis
D. right lower quadrant tenderness
E. referred rebound tenderness[ Rovsing sign ]

53. the most common site for intestinal obstruction due to cholecystoenteric fistula is the
A. pylorus
B. duodenum
C. jejunum
D. ileum
E. sigmoid colon.

54. Common characteristics of small bowel obstruction include all of the following
EXCEPT:
A. ascites
B. frequent progression to strangulation
C. failure to pass flatus
D. distention
E. vomiting

55. Gastroesophageal reflux is BEST diagnosed with:


A. radiography
B. 24-hour pH monitoring of lower esophagus
C. esophagoscopy
D. documentation of a decrease in esophageal pH after HCI is placed in the stomach .
E. acid-clearing swallowing test .

56. The risk of esophageal cancer increases with all of the following EXCEPT:
A. alcohol ingestion.
B. smoking .
C. chronic ingestion of hot beverages.
D. aflatoxin.>>(HCC)
E. poor oral hygiene.

57. Achalasia is associated with all of the following EXCEPT:


A. chagas ’ disease in South America
B. dysphasia.
C. weight loss.
D. relaxation of the lower esophageal sphincter with swallowing .
E. aspiration pneumonia, which may cause lung abscesses.

58. Nonsurgical causes of abdominal pain include all of the following EXCEPT:
A. pneumonia.
B. diabetic ketoacidosis.
C. acute salpingitis.
D. head trauma.
E. myocardial infarction.
59. Gastric acid secretion is stimulated by all of the following EXCEPT:
A. sight of food.
B. presence of food in the stomach.
C. fat in the duodenum.
D. gastrin.
E. histamine.

60. All the following statementare true of anal fissure EXCEPT:


A. is often associated with constipation
B. pain during and after defication is typical
C. the fissure is usually anterior
D. the lesion is commonly seen with crohns disease
E. division of the lower half of the internal sphincter is usually curative

61. Factors that are associated with the development of acute pancreatitis include all of the
following EXCEPT:
A. alcohol.
B. gallstones.
C. celiac sprue.
D. hyperlipidemia.
E. pancreatic divisum.

62. Complications of untreated pancreatic pseudocysts include all of the following


EXCEPT:
A. gastrointestinal obstruction.
B. pancreatic necrosis.
C. free rupture.
D. abscess.
E. intracystic hemorrhage.

63. Common presenting conditions in patients with pancreatic carcinoma include all of the
following EXCEPT:
A. esophageal varices.
B. jaundice.
C. weight loss.
D. palpable gallbladder.
E. abdominal pain.

64. All of the following are associated with an increased risk of breast cancer EXCEPT:
A. dietary consumption of fat.
B. history of breast cancer in first-degree maternal relatives
C. age over 35.
D. early first pregnancy.
E. infertility

65. The MOST frequent histologic type of breast carcinoma is:


A. infiltrating papillary carcinoma.
B. infiltrating ductal carcinoma.
C. infiltrating lobular infiltrating.
D. colloid infiltrating.
E. medullary infiltrating.

66. The MOST frequent variety of Thyroid cancer is:


A. follicular carcinoma.
B. papillary carcinoma.
C. anaplastic carcinoma.
D. hashimoto’s associated lymphoma.

67. Acute mastitis MOST commonly occurs at or during:


A. birth.
B. puberty.
C. pregnancy.
D. lactation.
E. blunt trauma to the breast.

68. All of the following statements concerning nipple discharges are true EXCEPT:
A. they may be caused by multiple lesions.
B. when bloody , the discharge is due to a malignancy 70% of the time.
C. a milky discharge may be due to a pituitary adenoma .
D. benign duct papillomas are the most common cause of bloody discharges.
E. excision of involved duct may be necessary to determine the etiology
1:48

69. The MOST frequent site for breast cancer to develop is the :
A. upper inner quadrant.
B. lower inner quadrant.
C. lower outer quadrant.
D. upper outer quadrant.
E. subareolar zone.

70. The risk of bilateral breast cancer is HIGHEST if the first breast shows:
A. inflammatory carcinoma.
B. lobular carcinoma.
C. medullary carcinoma.
D. infiltrating ductal carcinoma.
E. paget’s disease.

71. All of the following are true statements concerning paget’s disease of the nipple
EXCEPT.
A. it is very uncommon, accounting for only 2% of all breast cancers.
B. it is an in situ squamous cell malignancy of the nipple.
C. it is an eczematoid lesion.
D. it has a better prognosis than the majority of other breast cancers.
E. it can be confused with malignant melanoma histologically.

72. Initial fluid resuscitation of a patient with multiple fractures and hypovolemic shock
should be :
A. blood transfusion
B. hypertonic saline
C. fresh frozen plasma
D. ringer ’s lactate
E. albumin

73. Which of the following findings suggests that shock in an injured patient may have a
cause other than hypovolemia:
A. hypotension/
B. distended neck veins.
C. decreased skin temperature.
D. diminished pulse pressure.
E. falling central venous pressure.

74. All of the following are physical signs of both massive hemothorax and tension
pneumothorax EXCEPT:
A. tracheal shift.
B. decreased breath sounds.
C. tachycardia.
D. hypotension.
E. distended neck veins.

75. What is the MOST common infecting organism in overwhelming postsplenectomy


infection:
A. Escherichia coli.
B. meningococcus.
C. streptococcus.
D. pneumococcus.
E. staphlococcus.

76. Decreased Paco2 levels should be attained in a patient at serious risk for cerebral
edema secondary to a head injury in order to :-
A. prevent neurogenic pulmonary edema
B. allow reciprocally high levels of PaO2 in the brain
C. prevent increased capillary permeability
D. prevent cerebral vasodilation
E. prevent metabolic acidosis

77. Brain injury alone


A. frequently causes shock
B. causes shock that is reversed by very simple measures
C. causes shock only if the skull is intact
D. rarely causes shock
E. causes shock if hypoxia is superimposed

78. The level of consciousness for a head injury patient is BEST evaluated by :
A. Glasgow coma scale
B. response to pain
C. CT scan
D. papillary responses
E. visual evoked potentials

79. All of the following are signs of acute vascular compromise of an extremity EXCEPT:
A. diminished sensation
B. pallor
C. absent pulses
D. gangrene
E. pain

80. Which of the following should NOT be used to control the pain caused by fractured
ribs:
A. morphine IV.
B. Demerol IM.
C. intercostals nerve blocks.
D. rib belts.
E. muscle relaxants.

81. Postthrombotic varicose veins are due to:


A. incompetent communicating veins
B. destruction of deep veins.
C. destruction of superficial veins.
D. iliofemoral incompetence.
E. block of the long saphenous vein.

82. Deep venous thrombosis prophylaxis is appropriate for all of the following patients
EXCEPT:
A. 67-year-old male undergoing a colectomy.
B. 21-year-old male undergoing an outpatient open inguinal hernia repair.
C. 21-year-old male in the ICU, comatose, with a closed head injury.
D. 60-year-old female undergoing open reduction and internal fixation (ORIF) of a hip
fracture.
C. 60-year-old female undergoing reduction of a lung carcinoma.

83. Causes of delayed union of fractures includes all the following EXCPET:
A. Compound fracture
B. Infection
C. Adequate immobilization
D. Poor blood supply

84. Indication for skull X-Ray in head injury are:


A. Loss of consciousness
B. Focal neurological symptoms
C. Difficulty in assessing the patient
D. A& B only
E. All the above

85. In Head injury C.T. scan is indicated in the following


A. Aphasia after the injury
B. Deterioration of level of consciousness
C. Skull fracture with persistent headache
D. A&B only
E. All the above

86. Most important steps in management of head injury include:


A. Prevent hypoxia
B. Prevent Dehydration
C. Assure Brain Metabolism
D. Prevent secondary brain injury
E. All the above

87. Most common cause of solitary thyroid nodule is:


A. Malignancy
B. Toxic goiter
C. Thyroid Adenoma
D. Thyroiditis
E. Hashimoto’s disease

88. Follicular carcinoma of thyroid gland:


A. Is the commonest tumor
B. Can be diagnosed by F.N.A.B
C. Usually multifocal
D. Commonest tumor of young age
E. Prognosis is worse than papillary carcinoma

89. Abnormal nipple discharge could be due to:


A. Duct papilloma
B. Duct ectasia
C. Retention cyst of montgomery gland
D. A&B only
E. All the above

90. Retracted nipple can be caused by the following EXCPET:


A. Carcinoma of Breast
B. Duct ectasia
C. Chronic periductal mastitis
D. A&B only
E. All the above

91. The commonest type of the breast cyst is:


A. Solitary mamary cyst
B. Galactocele
C. Lymphatic cyst
D. Simple multiple cysts.

92. Carcinoma of breast:


A. Is the third common malignant tumor of women
B. common in Ages between 45-55 years.
C. 10% of case are males.
D. usually tender rapidly enlarging
E. medullay carcinoma is the comment type .

93. Carcinoma of the breast


A. may be familial
B. infiltrating duct carcinoma is the commonest
C. treated by lumpectomy and axillary clearance
D. A&B only
E. None of the above

94. Fracture of the sternum


A. Is the comments fracture of thoracic cage.
B. Usually painful.
C. can’nt be associated with myocardial damage
D. usually treaded by internal fixation .
E. B&C only.

95. In massive haemothorax.


A. about 500ml of blood in pleural cavity
B. cause dyspnea & Neck veins distension
C. treated by thoracotomy tube only
D. All of the above
E. None of the above

96. Tension pneumothorax


A. is the commonest type of chest injuries
B. Needs urgent X-Ray chest
C. Is a clinical Diagnosis
D. Causes flat neek viens
E. Treated by thoracotomy tube after chest X-ray.

97. Varicose view.


A. Primary type usually affect one limb , below the knee
B. Short saphenous system is more affected
C. May be caused by D.VT.
D. A&B only
E. None of the above.

98. Anal Fissure:


A. Usually anterior
B. May be caused by previous anal surgery
C. Can cause dark bleeding PR.
D. Sometimes is painful
E. Treated by steroids

99. The comments type of Anorectal abscess is:


A. Ischio rectal
B. Perianal
C. Submucons
D. Pelvirectal

100. Varicose ulcer is:


A. Usually very painful & shallow
B. Deep with much necrotic tissue
C. More common with primary varicose vein
D. A & C only
E. None of the above

Dr.Ashraf Obaid

1. Which hypersensitivity reaction is associated with a tuberculin reaction?


A. Type I: immediate
B. Type II: cytotoxic
C. Type III: immune complex
D. Type IV: cell mediated

2. The most common location for a gastric ulcer is


A. Fundus
B. Greater curvature
C. Cardia
D. Body
E. Antrum

3. Regarding the management of major trauma


A. Deaths follow a trimodal distribution
B. Cardiac tamponade is characterised by raised BP, low JVP and muffled heart sounds
C. Assessment of uncomplicated limb fractures should occur during the primary survey
D. Deterioration of the casualty during the primary survey should lead to the secondary
survey
E. All are false

4. All of the following are true about neurogenic shock except:


A. There is a decrease in systemic vascular resistance and an increase in venous capacitance.
B. Tachycardia or bradycardia may be observed, along with hypotension.
C. The use of an alpha agonist such as phenylephrine is the mainstay of treatment.
D. Severe head injury, spinal cord injury, and high spinal anesthesia may all cause
neurogenic shock.

5. The following cause hypercalcaemia except :


A. Sarcoidosis
B. Primary hyperparathyroidism
C. Acute pancreatitis
D. Metastatic bronchial carcinoma
E. Milk-Alkali syndrome

6. For a 40-kg baby the maintenance daily fluid requirement is approximately which of the
following?
A. 1100 ml
B. 1250 ml
C. 1550 ml
D. 1700 ml
E. 2000 ml

7. Infantile hypertrophic pyloric stenosis


A. Occurs with a male: female ratio of 4:1.
B. Presents between six and eight months of age
C. Typically presents with bile stained projectile vomiting
D. Surgical treatment is by Heller's Cardiomyotomy
E. all are false

8. Which of the following do you consider to be the most important clinical sign in acute
appendicitis,
A. Abdominal tenderness around the umbilicus
B. Abdominal tenderness in the RIF
C. Tenderness over McBurney’s point
D. Rovsing’s sign positive
E. Suprapubic tenderness

9. Heparin
A. Acts as an anti-platelet
B. Acts as an anti-thromboplastin
C. Acts as an antithrombin
D. All of the above
E. None of the above

10. All of the following are mechanisms of urinary calculi formation except,
A. Hypoparathyroidism
B. Prolonged recumbency
C. Infection with urea-splitting organisms
D. Foreign body
E. Urinary tract obstruction

11. Which of the following are not found in peritonitis?


A. Patient is lying still
B. Guarding
C. Rebound tenderness
D. Hyperactive bowel sounds
E. Rigid abdomen

12. Which one of the following suggest a diagnosis of Hirschsprung's disease?


A. A contrast-study showing dilatation of the aganglionic bowel segment.
B. Early presentation with vomiting.  
C. Neonatal large bowel obstruction.  
D. Presentation after 1 year of age.
E. Red current jelly stools.

13. Which of the following regarding the anatomy of the heart is true?
A. The aortic valve is tricuspid.  
B. The ascending aorta is entirely outside the pericardial sac.  
C. The left atrial appendage is identified readily by transthoracic echocardiography. 
D. The pulmonary trunk lies anterior to the ascending aorta.
E. The right atrium is posterior to the left atrium.
14. Which of the following is true concerning Scaphoid fractures?
A. Rarely occur in young adults  
B. when complicated by avascular necrosis the proximal pole is usually affected 

C. should be treated by bone grafting and internal fixation even if undisplaced 


D. wrist fractures are uncommon
E. anteriorposterior and lateral radiographs reveal most fractures

15. Which of the following statements is true of upper limb nerve injuries?
A. Injury to the median nerve results in a wrist drop
B. Injury to the radial nerve results in loss of sensation over the palmar aspect of the index
finger
C. Injury to the median nerve results in loss of sensation in the anatomical snuffbox
D. Injury to the ulnar nerve results in a claw hand
E. Injury to the ulnar nerve results in loss of sensation over the thumb

16. Regarding intravenous solutions:


A. Normal saline contains 180mmol/l of sodium
B. Ringer's lactate solutions is designed for intracellular fluid replacement
C. Sodium bicarbonate 8.4% is a hyperosmolar solution
D. Normal saline with added potassium is appropriate therapy to correct a respiratory
alkalosis
E. All are True

17. Which of the following concerning the Femoral sheath is false:


A. Contains the femoral artery
B. Contains lymph nodes
C. Contains the femoral canal
D. Contains femoral nerve
E. Contains the femoral vein

18. The following is true of the spleen:


A. Is the largest lymphoid organ in the body
B. Lies obliquely between the seventh and tenth rib
C. The lower pole extends beyond the mid-axillary line
D. Is usually palpable when normal
E. Usually measures 16cm in maximum length when healthy

19. Breast cancer risk is increased in association with the following factors except:
A. Nulliparity
B. Immediately after pregnancy
C. Early menarche
D. Early age at first pregnancy
E. Late menopause

20. In tension pneumothorax the following signs are present except:


A. Hypoxia
B. Hyperresonance to percussion on the affected side
C. Tracheal deviation to the ipsilateral side
D. Distended neck veins
E. Tachycardia
21. The most common hernia in females is:
A. Femoral hernia.
B. Direct inguinal hernia.
C. Indirect inguinal hernia.
D. Obturator hernia.
E. Umbilical hernia.

22. The most helpful diagnostic radiographic procedure in small bowel obstruction is:
A. CT of the abdomen.
B. Contrast study of the intestine.
C. Supine and erect x-rays of the abdomen.
D. Ultrasonography of the abdomen.
E. MRI Abdomen

23. In role of nine extent of burn if entire trunk is burned it will be equal to:
A. 9% body surface area.
B. 18% body surface area.
C. 36% body surface area.
D. 27% body surface area.
E. 45% body surface area.

24. If torsion of the testicle is suspected, surgical exploration:


A. Can be delayed 24 hours and limited to the affected side.
B. Can be delayed but should include the asymptomatic side.
C. Should be immediate and limited to the affected side.
D. Should be immediate and include the asymptomatic side.

25. Hyperthyroidism can be caused by all of the following except:


A. Graves' disease.
B. Plummer's disease.
C. Struma ovarii.
D. Hashimoto's disease.
E. Medullary carcinoma of the thyroid.

26. A 9 month old boy presents with an acute scrotal swelling. The following diagnoses are
likely:
A. Epididymitis
B. Orchitis
C. Torsion of the testicular appendage
D. Irreducible inguinal hernia
E. Acute idiopathic scrotal oedema
27. The evaluation of a comatose patient with a head injury begins with:
A. The cardiovascular system.
B. Pupillary reflexes.
C. Establishment of an airway.
D. Computed tomography (CT) of the brain
E. insertion of Intravenous canula
28. The following is an indication for thoracotomy in chest injury,
A. Cardiac tamponade
B. Uncontrolled pulmonary air leakage
C. Perforation of thoracic esophagus
D. Blood loss of 200ml/hr for 2-3 hrs via chest tube
E. All of the above

29. Regarding Gallstones all of the following are true except:-


A. Prevalence increases with advancing age
B. 30% of gallstones are radio-opaque
C. Cholesterol stones result from a change in solubility of bile constituents
D. Biliary infection, stasis and changes in gallbladder function can precipitate stone
formation
E. Gram-negative organisms are the most common isolated

30. In obstructive jaundice:


A. Urinary conjugated bilirubin is increased
B. Serum unconjugated bilirubin is increased
C. Urinary urobilinogen increased
D. Serum conjugated bilirubin is reduced
E. Faecal stercobilinogen is increased

31. The ideal fluid therapy in a patient with pyloric stenosis and repeated vomiting is:
A. Isotonic crystaloid containing sodium chloride
B. Hypertonic crystaloid containing dextrose – saline
C. Isotonic solution containing dextrose
D. Large molecular weight colloid containing dextran

32. Which of the following do you consider to be the most important clinical sign in acute
appendicitis,
A. Abdominal tenderness around the umbilicus
B. Abdominal tenderness in the RIF
C. Tenderness over McBurney’s point
D. Rovsing’s sign positive
E. Suprapubic tenderness
33. All of the following are mechanisms of urinary calculi formation except,
A. Hypoparathyroidism
B. Prolonged recumbency
C. Infection with urea-splitting organisms
D. Foreign body
E. Urinary tract obstruction

34. Which of the following is incorrect in Paget’s disease of nipple,


A. Presents with eczema and redness of the nipple and areola
B. Tumour arises from epidermal layer of nipple and not from epidermal layer of underlying
ducts
C. Underlying carcinoma may be present beneath the nipple
D. Mastectomy is considered the treatment of choice
E. It is considered carcinoma-in-situ

35. Organisms most commonly isolated in UTIs,


A. Kleb
B. E. Coli
C. Proteus
D. Acinetobacter
E. All of the above

36. Excessive saliva in a newborn is due to


A. Esophageal atresia
B. Salivary gland tumour
C. Primary Hypertrophic Pyloric Stenosis
D. Midgut volvulus
E. Hirschprung’s disease

37. The most frequent congenital diaphragmatic hernia seen in infants is,
A. Paraesophageal hernia
B. Sliding hernia
C. Congenitally short esophagus
D. Hernia through the foramen of Bochdalek
E. Hernia through the foramen of Morgagni

38. An undescended testis.


A. Can be associated with contralateral testicular dysfunction
B. Should be treated by orchidopexy by age 2
C. Is associated with inguinal hernia
D. Predisposes to malignant change
E. All of the above

39. The wound made to remove a perforated appendix is classified as,


A. Clean
B. Clean, contaminated
C. Contaminated
D. Dirty
E. Doesn’t apply as the wound isn’t a traumatic one

40. Concerning the treatment of haemorrhoids,


A. Can be by diet
B. By injection sclerotherapy
C. By banding
D. Is mainly by haemorrhoidectomy
E. All of the above

41. The following is an indication for thoracotomy in chest injury,


A. Cardiac tamponade
B. Uncontrolled pulmonary air leakage
C. Perforation of thoracic esophagus
D. Blood loss of 200ml/hr for 2-3 hrs via chest tube
E. All of the above
42. In an upper GI bleed, in some cases a Sengstaken-Blakemore tube is incerted. What is
the primary purpose of the tube?
A. Aspirate blood from the stomach
B. Tube feeding
C. Tamponade for varices
D. To decompress bowel
E. All of the above

43. Risk factors for pulmonary embolism do not include which of the following?
A. DVT
B. Recent surgery
C. Old age
D. Myocardial infarction
E. Chest infection

44. Which of the following is not found in peritonitis?


A. Patient is lying still
B. Guarding
C. Rebound tenderness
D. Hyperactive bowel sounds
E. Rigid abdomen

45. Commonest site for CA breast


A. Upper outer quadrant
B. Upper inner quadrant
C. Lower outer quadrant
D. Lower inner quadrant
E. None of the above

46. The most common site of fracture of the clavicle is:


A. Medial end.
B. Lateral end.
C. Midpoint of the clavicle.
D. Junction of the medial two-thirds and the lateral third.
E. Junction of the lateral two-thirds and the medial third.

47. The back of the medial epicondyle is related to the:


A. Radial nerve.
B. Axillary nerve.
C. Ulnar nerve.
D. Median nerve.
E. None of the above.

48. Regarding Gallstones all of the following are true except:-


A. Prevalence increases with advancing age
B. 30% of gallstones are radio-opaque
C. Cholesterol stones result from a change in solubility of bile constituents
D. Biliary infection, stasis and changes in gallbladder function can precipitate stone
formation
E. Gram-negative organisms are the most common isolated

49. Clinical features of limb ischaemia includes


A. Paraesthesia
B. Pallor
C. Pulselessness
D. Paralysis
E. All of the above

50. Regarding Hydatid disease all of the following are true except:-
A. Due to infection with the helminth Ecchinococcus granulosa
B. Man is an accidental intermediate host
C. Lunges is the commonest organ involved
D. Diagnosis can be confirmed by indirect haemagglutinin assay
E. Aspiration should not be performed if hydatid disease is suspected

Dr.Marwan abu sa3da


1. A 40-year-old female has a 4-cm hemangioma in the right lobe of the liver on computed
tomography scan. She is asymptomatic. Appropriate action should be:
A. fine-needle biopsy
B. arrangement for elective resection
C. no further action
D. angiographic embolization

2. Regarding an amebic liver abscess:


A. surgical drainage is usually required
B. negative stool testing for amebiasis rules out the disease
C. it should be drained percutaneously under com- puted tomography guidance
D. it is treated with metronidazole

3. A single organism is usually the causative agent in:


A. pelvic inflammatory disease
B. perforated diverticulitis
C. acute cholecystitis
D. primary peritonitis
E. diabetic foot infections

4. Signs and symptoms of hemolytic transfusion reactions include


A. Hypothermia
B. Hypertension
C. Polyuria
D. Abnormal bleeding
E. Hypesthesia at the transfusion site
5. In a hemolytic reaction caused by an incompatible blood transfusion, the treatment that
is most likely to be helpful is:
A. Promoting a diuresis with 250 ml of 50% mannitol
B. Treating anuria with fluid and potassium replacement
C. Acidifying the urine to prevent hemoglobin precipitation in the renal tubules
D. Removing foreign bodies, such as Foley catheters, which may cause hemorrhagic
complications
E. Stopping the transfusion immediately

6. Which statements about extrahepatic bile duct cancer are correct?


A. Cholangiography is not essential in evaluating patients for resectability.
B. The prognosis is excellent when appropriate surgical and adjuvant therapy are given.
C. The location of the tumor determines the type of surgical procedure.
D. The disease usually becomes manifest by moderate to severe right-side upper quadrant
pain.

7. Which of the following statements about the diagnosis of acute calculous cholecystitis is
true?
A. Pain is so frequent that its absence almost precludes the diagnosis.
B. Jaundice is present in a majority of patients.
C. Ultrasonography is the definitive diagnostic test.
D. Cholescintigraphy is not definitive diagnostic test.

8. Which statement about acute acalculous cholecystitis is correct?


A. The disease is often accompanied by or associated with other conditions.
B. The diagnosis is often difficult.
C. The mortality rate is higher than that for acute calculous cholecystitis.
D. The disease has been treated successfully by percutaneous cholecystostomy
E. all are correct

9. Which of the following statements about laparoscopic cholecystectomy are correct?


A. The procedure is associated with less postoperative pain and earlier return to normal
activity.
B. The incidence of bile duct injury is lower than for open cholecystectomy.
C. Laparoscopic cholecystectomy should be used in asymptomatic patients because it is safer
than open cholecystectomy.
D. Pregnancy is a contraindication.
10. A 15-year-old female presents with RUQ abdominal pain. Workup reveals a
choledochal cyst. Which of the following statements is TRUE?
A. Choledochal cysts are more common in men.
B. Laparoscopic cholecystectomy is the recommended treatment.
C. Patients with a choledochal cyst have an increased risk of cholangiocarcinoma.
D. All patients with a choledochal cyst have abdominal pain, a RUQ mass, and jaundice.
E. The etiology is infectious.

11. An 85-year-old man is brought to the hospital with a 2-day history of nausea and
vomiting. He has not passed gas or moved his bowels for the last 5 days. Abdominal films
show dilated small bowel, no air in the rectum and air in the biliary tree. Which of the
following statements is TRUE?
A. Air in the biliary tree associated with small-bowel obstruction suggests a diagnosis of
gallstone ileus.
B. An enterotomy should be distal to the site of obstruction and the stone should be removed.
C. Gallstone ileus is more common in the young adults.
D. Cholecystectomy is contraindicated.
E. Small-bowel obstruction usually occurs in the distal jejunum.

12. A 45-year-old patient with chronic pancreatitis is suffering from malnutrition and
weight loss secondary to inadequate pancreatic exocrine secretions. Which is TRUE
regarding pancreatic secretions?
A. Secretin releases fluid rich in enzymes.
B. Secretin releases fluid rich mainly in electrolytes and bicarbonate.
C. Cholecystokinin releases fluid,predominantly rich in electrolytes, and bicarbonate.
D. All pancreatic enzymes are secreted in an inactive form.
E. The pancreas produces proteolytic enzymes only.

13. A 43-year-old woman has gallstone pancreatitis that resolves in 2 days with
conservative treatment. She has no abdominal complaints and her liver and pancreatic
laboratory values have returned to normal. She is scheduled for laparoscopic
cholecystectomy. Which of the following statements is TRUE?
A. Intraoperative cholangiography is associated with a decreased risk of biliary tract injury .
B. The procedure should be scheduled for 6 weeks after resolution of symptoms .
C. Intraoperative cholangiography in this patient will identify choledocholithiasis in 50% of
cases .
D. Preoperative endoscopic retrograde cholangiopancreatography (ERCP) should be
performed.
E. The sensitivity of magnetic resonance cholangiopancreatography (MRCP) for
choledocholithiasis in this patient is less than 50%.
14. A 39-year-old woman is admitted with gallstone pancreatitis and epigastric pain.
Pertinent data include amylase, 2000 U/L; bilirubin, 1.2 mg/dL; and WBC count,
15,000/mm3 . After 2 days of medical management, her epigastric pain resolves. Her
amylase is 340 U/L and her bilirubin and WBC count have returned to normal.
Laparoscopic cholecystectomy should be attempted:
A. after endoscopic retrograde cholangiopancreatography (ERCP) and sphincterotomy
B. prior to discharge
C. once her amylase is normal
D. 4 to 6 weeks later
E. only if the patient develops recurrent pancreatitis

15. A 48-year-old woman is admitted with acute cholecystitis. The bilirubin level is
elevated, as are the serum and urinary amylase levels. Which radiologic sign indicates
biliary obstruction in pancreatitis?
A. Pancreatic intraductal calcification
B. Smooth narrowing of the distal CBD
C. Stomach displaced anteriorly
D. Calcified gallstone
E. Air in the biliary tree)

16. Following a motor vehicle accident a truck driver complains of severe abdominal pain.
Serum amylase level is markedly increased to 800 U. Grey Turner’s sign is seen in the
flanks. Pancreatic trauma is suspected. Which statement is true of pancreatic trauma?
A. It is mainly caused by blunt injuries.
B. It is usually an isolated single-organ injury.
C. It often requires a total pancreatectomy.
D. It may easily be overlooked at operation.
E. It is proved by the elevated amylase level.

17. A 73-year-old woman is evaluated for obstructive jaundice after an injury to the CBD,
7 months previously at laparoscopic cholecystectomy. The alkaline phosphatase is elevated.
In obstructive jaundice, which of the following statements is true regarding alkaline
phosphatase?
A. Its level increases before that of bilirubin.
B. Its level is unlikely to be increased in pancreatic malignancy.
C. Its elevation indicates bone metastasis.
D. Its elevation excludes hepatic metastasis.
E. Its level falls after that of the bilirubin,following surgical intervention.
18. A48-year-old female travel agent presents with jaundice. Radiological findings confirm
the presence of sclerosing cholangitis. She gives a long history of diarrhea for which she has
received steroids on several occasions. She is likely to suffer from which of the following?
A. Pernicious anemia
B. Ulcerative colitis
C. Celiac disease
D. Liver cirrhosis
E. Crohn’s disease

19. A38-year-old male lawyer develops abdominal pain after having a fatty meal.
Examination reveals tenderness in the right hypochondrium and a positive Murphy’s sign.
Which test is most likely to reveal acute cholecystitis?
A. HIDA scan
B. Oral cholecystogram
C. Intravenous cholangiogram
D. CT scan of the abdomen
E. ERCP

20. A 65-year-old woman is admitted with RUQ pain radiating to the right shoulder,
accompanied by nausea and vomiting. Examination reveals tenderness in the RUQ and a
positive Murphy’s sign. A diagnosis of acute cholecystitis is made. What is the most likely
finding?
A. Serum bilirubin levels may be elevated.
B. Cholelithiasis is present in 40–60%.
C. Bacteria are rarely found at operation.
D. An elevated amylase level excludes this diagnosis.
E. A contracted gallbladder is noted on ultrasound.

21. A 32-year-old diabetic woman who has taken contraceptive pills for 12 years develops
RUQ pain. CT scan of the abdomen reveals a 5-cm hypodense lesion in the right lobe of the
liver consistent with a hepatic adenoma. What should the patient be advised to do?
A. Undergo excision of the adenoma
B. Stop oral contraceptives only
C. Stop oral hypoglycemic medication
D. Undergo right hepatectomy
E. Have serial CT scans every 6 months

22. A 9 month old boy presents with an acute scrotal swelling. The following diagnoses are
likely:
A. Epididymitis
B. Orchitis
C. Torsion of the testicular appendage
D. Irreducible inguinal hernia
E. Acute idiopathic scrotal oedema

23. A 76-year-old man presents with weight loss, dark urine, and pale stools which are
difficult to flush away. An excess of which of the following would account for this history?
A. Conjugated bilirubin
B. Hyperbilirubinaemia
C. Stercobilinogen
D. Unconjugated bilirubin
E. Urobilinogen
24. An otherwise well 13-year-old boy is admitted complaining of sudden onset severe left
sided testicular pain 2 hours prior to admission. He gives no history of trauma, dysuria or
frequency. On examination he is found to have a tender, high-riding testicle.What is the
most appropriate next step in this young man’s management?
A. Herniography
B. Scrotal Doppler ultrasound on the next available list
C. FBC and U&E
D. Scrotal Doppler ultrasound as an emergency
E. Surgical exploration of his scrotum

25. Acute scrotum


A. Torsion testis should be operated within 12 hour of presentation
B. Epidedimoorchitis pain increase by testicular elevation
C. If in doubt scrotum should be explored
D. Doppler ultrasound has no role in diagnosis
E. None of the above

26. stones in the common bile duct:


A. Are present in nearly 50 per cent of cases of cholecystitis.
B. Often give rise to jaundice, fever and biliary colic.
C. Are usually accompanied by progressive jaundice.
D. Are usually associated with a distended gallbladder.
E. A&D only.

27. Which of the following statements regarding whole blood transfusion is correct?
A. Whole blood is the most commonly used red cell preparation for transfusion in the
B. Whole blood is effective in the replacement of acute blood loss.
C. Most blood banks have large supplies of whole blood available.
D. The use of whole blood produces higher rates of disease transmission than the use of
individual component therapies.
E. Old Whole blood is effective in the replacement of platelets.
28. Acute cholecystitis all are true except
A. Commonest bacteria is E .coli
B. Wall thickness more than 3mm by ultrasound
C. WCC is between 10-15 000 cell/mm3
D. Mild elevated bilirubin may accompany it
E. HIDA scan has no role in diagnosis of acute cholecystitis

29. A 51-year-old male experiences the sudden onset of massive emesis of bright red blood.
There have been no prior episodes of hematemesis. He is known to be hepatitis B surface
antigen positive. His hematemesis is most likely a consequence of which of the following
abnormalities of the esophagus?
A. Varices
B. Barrett esophagus
C. Candidiasis
D. Reflux esophagitis
E. Squamous cell carcinoma

30. A 61-year-old male has had ascites for the past year. After a paracentesis with removal
of 1 L of slightly cloudy, serosanguinous fluid, physical examination reveals a firm, nodular
liver.Laboratory findings include positive serum HBsAg and presence of hepatitis B core
antibody. He has a markedly elevated serum alpha-fetoprotein (AFP) level. Which of the
following hepatic lesions is he most likely to have?
A. Hepatocellular carcinoma
B. Massive hepatocyte necrosis
C. Marked steatosis
D. Wilson disease
E. Autoimmune hepatitis

31. A 76-year-old man presents with weight loss, dark urine, and pale stools which are
difficult to flush away. An excess of which of the following would account for this history?
A. Conjugated bilirubin
B. Hyperbilirubinaemia
C. Stercobilinogen
D. Unconjugated bilirubin
E. Urobilinogen

32. Acute pancreatitis


A. Serum calcium start to rise after 48 hours
B. Hypoglycaemia is bad prognostic factor
C. Age is an important prognostic factor
D. Serum amylase is more specific than serum lipase
E. Severe pancreatitis compromise around 40% of cases

33. The most commonly used imaging method for diagnosis of acute cholecystitis is:
A. CT of the abdomen.
B. Ultrasonography of the gallbladder.
C. Oral cholecystogram.
D. Radionuclide (HIDA) scan of the gallbladder
E. MRI

34. A 23-year-old male presents to the emergency department after being involved in a
motor vehicle accident. On physical examination, he opens his eyes spontanously, he
occasionally mumbles incomprehensible sounds, he localizes to painful stimulation with his
right upper extremity, His pupils are 4 mm bilaterally and reactive. This patient’s Glasgow
Coma Scale (GCS) score:
A. 7
B. 9
C. 8
D. 11
E. 12

35. Complication of undescended testis include all of the following except :


A. Malignant degeneration.
B. Increased susceptability to trauma.
C. Increased spermatogenesis.
D. More liable to testiculer torsion.
E. Psychological complication
36. Neonatal duodenal obstruction:
A. May be associated with down's syndrome.
B. Is more frequently found in premature infants.
C. Typically presents with gross abdominal distension.
D. Usually presents with vomiting of non-bile stained fluid
E. B&C only.

37. Markedly elevated alpha-fetoprotein is diagnostic


A. Hepatic hemangioma
B. Angiosarcoma in the liver
C. Hepatic adenoma
D. Focal nodular hyperplasia
E. Hepatocellular carcinoma
38. on clinical examination of tortion testis all are true except:
A. testis is tender and swollen.
B. testis is elevated and raised.
C. loss of cremasteric reflex.
D. redness with possible reactive hydrocele.
E. pain decrease with elevation of the testis .

39. Regarding cryptorchidism(undesended testis) all are true except :


A. refers to the interruption of the normal descent of the testis into the scrotum.
B. The testicle may reside in the retroperitoneum, in the internal inguinal ring, in the inguinal
canal, or even at the external ring.
C. At birth, approximately 95% of infants have the testicles normally positioned in the
scrotum.
D. it’s a common disorder and incidence increased up to 30 % in premature .
E. undescended testis is always regarded as an ectopic testis.

40. Regarding GI(gastrointestinal bleeding ) all are true except:


A. Lower GI hemorrhage is defined as an abnormal intra luminal blood loss from a source
distal to the Treitz ligament.
B. The most cause of massive lower GI bleeding in adults are diverticulosis and
angiodysplasia.
C. Cancer colon is usually associated with massive lower GI hemorrhage.
D. Hemorrhage from diverticular disease stops spontaneously in 80% of patients.
E. patients with massive upper GI bleeding may present with maroon stools or bright red
blood from the rectum.

41. Signs of severe bood loss include the following except:


A. Pallor
B. Clammy skin
C. bradycardia
D. Tachycardia
E. Hypotension

42. regarding stigmata of bleeding in peptic ulcer :


A. its associated with increase risks for rebleeding .
B. adherent clot is the most significant stigmata.
C. arterial spurting hemorrhage associated with low risk of rebleeding .
D. risk of rebleeding in ulcer with clean base is above 50%
E. nonbleeding visible vessel is not associated with risk of rebleeding .
43. Features of inflammatory response syndrome (SIRS) include the following except:
A. Temperature> 38.4C
B. Temperature <36.C
C. WCC<4.ooo cells per ml
D. Respiratory rate >20 per minute
E. PCO2> 32 mmHg

44. The development of thrombocytopenia and arterial thrombosis with heparin requires:
A. Continuation of heparin and platelet transfusion
B. Continuation of heparin and thrombolysis
C. Doubling the heparin dosage
D. Changing the route of heparin administration
E. Discontinuation of heparin

45. Regarding Heparin-induced thrombocytopenia (HIT) all are true except


A. Is a special case of drug-induced immune thrombocytopenia.
B. The platelet count typically begins to fall 5 to 14 days after heparin has been started.
C. Thrombocytopenia is usually severe.
D. HIT should be suspected if the platelet count falls to less than 100,000 or if it drops by
50% from baseline in a patient receiving heparin
E. HIT is more common with full-dose unfractionated heparin (1 to 3%)

46. Regarding gall bladder and bile secretion all are true except :
A. The gallbladder is a pear-shaped, about 7 to 10 cm long with an average capacity of 30 to
50ml.
B. When obstructed, the gallbladder can distend markedly and contain up to 300 mL
C. Anomalies of the hepatic artery and the cystic artery are quite common, occurring in as
many as 50% of cases.
D. liver produces 500 to 1000 mL of bile a day
E. Vagal stimulation decreases secretion of bile

47. Regarding gall bladder stones all are true except:


A. Prevalence increases with advancing age
B. Over 10% of those with stones in the gallbladder have stones in the common bile duct.
C. 10-20% become symptomatic
D. cholesterol stones are the most common type.
E. pigment stones are associated with secondary common bile duct stones .

48. In acute cholecystitis all true except :


A. Most common organisms are E. coli
B. 90% cases result from obstruction to the cystic duct by a stone.
C. patient present with constant pain usually greater than 6 hours duration in right upper
quadrant .
D. presence of gall stones and percholecystic fluid on US is diagnostic
E. Cholecystectomy is contraindicated in acute stage .

49. complication of acute cholecystitis include all the following except:


A. Gangrenous cholecystitis
B. Gallbladder perforation
C. Cholecystoenteric fistula
D. mesnteric ischemia
E. Gallstone ileus

50. Regarding choledocholithisis all are true except:


A. may be silent and or may cause obstruction, complete or incomplete
B. may manifest with cholangitis or gallstone pancreatitis.
C. present with severe jaundice and cholangitis in case of stone impaction
D. Rt upper quadrant pain ,fever,and jaundice are called charcots triad in cholangitis.
E. impaction of small stones has no relation with acute pancreatitis .

51. Regarding acalculous cholecystistis all are true except:


A. Acute inflammation of the gallbladder can occur without gallstones
B. Acalculous cholecystitis typically develops in critically ill patients in the intensive care
unit.
C. Patients on parenteral nutrition with extensive burns, sepsis, major operations are at risk
for developing acalculous cholecystitis.
D. US is not a good diagnostic tool.
E. can be managed by cholecystectomy or percutanous cholecyststomy .

52. Regardind gall bladder cancer all are true except:


A. Larger stones (>3 cm) are associated with a 10-fold increased risk of cancer.
B. up to 95% of patients with carcinoma of the gallbladder have gallstones.
C. Polypoid lesions of the gallbladder are not associated with increased risk of cancer
D. Patients with choledochal cysts have an increased risk of developing cancer
E. Sclerosing cholangitis is risk factor for developing gall bladder cancer .

53. Regarding tumors of the liver all are true except :


A. Hemangioma is the most common solid benign lesion .
B. Spontaneous rupture in hemangioma (bleeding) is rare.
C. Hepatic adenomas carry a significant risk of spontaneous rupture with intraperitoneal
bleeding.
D. Hepatic adenomas have a risk of malignant transformation to a well-differentiated
HCC(hepatocellular carcinoma).
E. focal nodular hyperplasia lesions(FNH) lesions usually rupture spontaneously and have
significant risk of malignant transformation .

54. Regarding pyogenic liver abscess all are true except :


A. arise as a result of biliary sepsis.
B. associated with high mortality.
C. appendicitis is unlikely the cause pyogenic liver abscess.
D. 30% of patient have pleural effusion on presentation .
E. lab.investiation show elevated WBC and abnormal liver function .

55. Regarding acute pancreatitis all are true except:


A. Gallstones less than 5mm diameter are more likely to cause pancreatitis than larger ones
B. The mortality associated with infected necrosis is about 40%
C. Cullen's sign is a sign of retroperitoneal hemorrhage in severe hemorrhagic pancreatitis .
D. elevated serum amylase is a significant predictor of severity .
E. 50% of deaths occur within first week due to multi-organ failure .

56. All occur as a complication of acute pancreatitis except:


A. pancreatic fluid collection
B. colonic necrosis
C. coagulopathy
D. hypercalcemia
E. respiratory failure

57. Regarding head trauma all are true except:


A. basal skull fracture regarded when one of the orbital roof ,sphenoidal bone or
petromastoid portion are involved .
B. epidural hematoma is an Lens shape hematoma between dura and the skull.
C. subdural hematoma is crescent shaped hematoma ,between brain and dura
D. secondary brain injury is preventable.
E. GCS glascow Coma scale 3/15 indicate uncomprehensive sounds .

58. Regarding chronic lower limb ischemia all are true except:
A. Claudication distance is distance after which the pain is felt.
B. Rest pain is continous severe burning pain in the foot which indicate critical ischemia.
C. trophic changes include tapering digits ( loss of S.C fat ) and muscle wasting .
D. usuall presentations of patients with lower limb ischemia are pain,trophic changes and
gangrene.
E. venous filling time more than 2 minutes indicates mild lower limb ischemia.
59. Regarding 4 weeks 4 Kg bodywt. old full term neonate presented with rapidly
progressive projectile non bilious vomiting and palpable upper abdominal mass all are
true except :
A. dehydration and alkalosis are prominent features.
B. maintenance fluid therapy is about 4ml /Kg /hour.
C. administration of IV fluids with 5% dextrose, 0.5% normal saline, and KCl usually
corrects the alkalosis .
D. Estimated total blood volume is about 320 cc.
E. the most likely diagnosis is high jejunal atresia

60. All are true regarding jejunoileal atresia except:


A. present with bile stained vomiting .
B. Failure to pass meconium or small amounts of mucus or meconium maybe passed per
rectum.
C. present with abdominal distention.
D. X-ray show double bubble appearance .
E. The x-rays usually show multiple air-fluid levels.

61. regarding malrotation of the gut all are true except:


A. The patient might be asymptomatic and then develop the symptoms when he is older.
B. Commonest abnormality results in caecum lying close to DJ flexure.
C. Fibrous bands may be present between caecum and DJ flexure (Ladd's bands).
D. the patient is unlikely to have clinical picture of duodenal obstruction .
E. In malrotation midgut mesentery is abnormally narrow and liable to volvulus.

62. Which of the following conditions is considered to increase the risk of gastric cancer?
Except
a. Pernicious anemia
b. Prior partial gastrectomy
c. Gastric hyperplastic polyps
d. Gastric adenomatous polyps
____________________________________________________________________

1. Which of the following statements about the anatomic course of the esophagus is
correct?
E. The esophagus deviates anteriorly and to the left as it enters the abdomen.

2. Which of the following statements about esophageal anatomy is correct?


C. The esophagus has two distinct muscle layers, an outer, longitudinal one and an inner, circular
one, which are striated in the upper third and smooth in the distal two thirds.
6. Which of the following statements about achalasia is/are correct?
D. Manometry demonstrates failure of LES relaxation on swallowing and absent or weak
simultaneous contractions in the esophageal body after swallowing.

8. Which of the following statements about epiphrenic diverticula of the esophagus is/are
correct?
E. The operation of choice is a stapled diverticulectomy, long esophagomyotomy, and partial
fundoplication

9. Which of the following statements about Schatzki's ring is correct?


B. Dysphagia occurs when the ring diameter is 13 mm. or less.

13. Which of the following statements regarding the pathology of esophageal carcinoma
is/are correct?
C. Patients with Barrett's metaplasia are 40 times more likely than the general population to
develop adenocarcinoma

15. The best management for a 48-hour-old distal esophageal perforation is:
A. Antibiotics and drainage.
B. Division of the esophagus and exclusion of the perforation.
C. Primary repair with buttressing.
E. T-tube fistula and draina

16. A 50-year-old patient develops sudden left lower chest pain and epigastric pain after
vomiting. The patient shows diaphoresis, breath sounds are decreased on the left, and there
is abdominal guarding. The most appropriate diagnostic test is:
A. Aortography.
B. Esophagoscopy.
C. Electrocardiogram.
D. Film of the chest.
E. White blood count

20. The presence of a nonmalignant mid- or upper esophageal stricture always indicates the
presence of:
A. Alkaline reflux esophagitis.
B. Barrett's esophagus.
C. Idiopathic reflux disease.
D. Mediastinal fibrosis.

21. Which of the following is most reliable for confirming the occurrence of a significant
esophageal caustic injury?
A. History of the event.
B. Physical examination of the patient.
C. Barium esophagraphy.
D. Endoscopy.

22. Indications for surgical reconstruction of the esophagus include which of the following?
ex
A. Continuing requirement for frequent dilation of an extensive esophageal stricture for a
minimum of 2 years.
B. Failure or refusal of the patient to comply with a treatment regimen of regular dilation.
C. Development of a fistula between the esophagus and tracheobronchial tree.
D. Iatrogenic perforation of the esophagus during attempted dilation.
Answer: BCD

23. First-line therapy for routine peptic duodenal ulcer disease includes:
A. Vagotomy and antrectomy.
B. Upper endoscopy and biopsy to rule out tumor.
C. Evaluation for Helicobacter pylori.
D. Serum gastrin determination.

24. Appropriate management of severe vomiting associated with gastric outlet obstruction
from peptic ulcer disease includes all of the following except:
A. Nasogastric suction.
B. Intravenous hydration.
C. Nutritional assessment; upper endoscopy to rule out malignancy.
D. Intravenous H 2 antagonist.
E. Oral antacid therapy.

25. All of the following are complications of peptic ulcer surgery except:
A. Duodenal stump blowout.
B. Dumping.
C. Diarrhea.
D. Delayed gastric emptying.
E. Steatorrhea.

26. The presentation of Zollinger-Ellison syndrome includes all of the following except:
A. Hyperparathyroidism in patients with multiple endocrine neoplasia type 1 (MEN 1)
syndrome.
B. Diarrhea.
C. Migratory rash.
D. Jejunal ulcers.
E. Duodenal ulcers.

27. All are true about the dumping syndrome except:


A. Symptoms can be controlled with a somatostatin analog.
B. Diarrhea is always part of the dumping syndrome.
C. Flushing and tachycardia are common features of the syndrome.
D. Separating solids and liquids in the patient's oral intake alleviates some of the symptoms of
the syndrome.
E. Early postoperative dumping after vagotomy often resolves spontaneously.

28. In patients with bleeding duodenal ulcers, the endoscopic finding associated with the
highest incidence of rebleeding is:
A. Visible vessel.
B. Cherry-red spot.
C. Clean ulcer bed.
D. Duodenitis.
E. Shallow, 3-mm. ulcer.

29. All of the following are contraindications for highly selective vagotomy except:
A. Intractable duodenal ulcer disease.
B. Peptic ulcer disease causing gastric outlet obstruction.
C. Fundic peptic ulceration.
D. Cigarette chain smoking.
E. Perforated peptic ulcer disease with more than 24 hours' soilage.

30. All the following are true of omeprazole except:


A. It is the only drug available that has the potential to achieve pharmacologically induced
achlorhydria.
B. It works by blocking the hydrogen-potassium ATPase in the parietal cell.
C. It is parietal cell specific.
D. It has a short half-life (about 90 minutes) when taken orally.
E. It has been associated with gastric neoplasm in a rat model.

31. All of the following statements about gastrin-releasing peptide (GRP) are true except:
A. In species other than man and dog GRP is commonly referred to as bombesin.
B. GRP serves as a neurotransmitter.
C. GRP inhibits pancreatic secretion when given intravenously.
D. GRP stimulates gastric acid secretion when given intravenously.
E. GRP is released in response to cholinergic stimulation of the parietal cells to stimulate
.release of gastrin

32. Cholecystokinin (CCK) is believed to function in all of the following processes except:
A. It physiologically delays gastric emptying.
B. It appears to have a role in satiety regulation.
C. It contracts the gallbladder.
D. It stimulates pancreatic secretion.
E. It is important in the control of the anal sphincter.

33. All of the following measures have been recommended for control of acid secretion in
patients with Zollinger- Ellison syndrome except:
A. Antrectomy.
B. Highly selective vagotomy.
C. Total gastrectomy.
D. Vagotomy and pyloroplasty.
E. Medical therapy with Prilosec (omeprazole).

34. All of the following contribute to peptic ulcer disease except:


A. Cigarette smoking.
B. Nonsteroidal anti-inflammatory drugs.
C. Helicobacter pylori.
D. Gastrinoma.
E. Spicy foods.

35. Which of the following statements about gastric polyps is/are true?
A. Like their colonic counterparts, gastric epithelial polyps are common tumors.
B. They are analogous to colorectal polyps in natural history.
C. Endoscopy can uniformly predict the histology of a polyp based on location and appearance.
D. In a given patient, multiple polyps are generally of a single histologic type.
E. Gastric adenomatous polyps greater than 2 cm. in diameter should be excised because of the
risk of malignant transformation.

36. Which of the following statements about gastric leiomyomas is/are true?
A. They are the most common type of gastric tumor of the stomach at autopsy.
B. The leiomyoblastoma cell type reflects malignant transformation of gastric leiomyomas.
C. A conservative surgical approach is indicated for their resection since regional
lymphadenectomy has not been proved
reliable even when they turn out to be malignant.
D. Severe hemorrhage may occur from deep ulcerations overlying the intramural tumor.

39. Which of the following risk factors have been shown to increase significantly the
incidence of gastrointestinal bleeding from stress gastritis in intensive care unit (ICU)
patients?
A. Glucocorticoid administration.
B. Respiratory failure.
C. Coagulopathy.
D. Organ transplantation.
E. Jaundice.

43. Numerous epidemiologic associations have been made between (1) environmental and
dietary factors and (2) the incidence of gastric cancer, including all except:
A. Dietary nitrites.
B. Dietary salt.
C. Helicobacter pylori infection.
D. Dietary ascorbic acid.

44. All of the following benign conditions are associated with increased rates of gastric
cancer except:
A. Pernicious anemia.
B. Multiple endocrine neoplasia type I (MEN 1).
C. Adenomatous polyps.
D. Chronic atrophic gastritis.

45. Which of the following statements concerning the pathology of gastric cancer is true?
A. Distal gastric cancers are becoming more common.
B. Intestinal-type gastric tumors resemble colon carcinomas and have a better prognosis than
diffuse type.
C. Early gastric cancers are confined to the mucosa and are lymph node negative.
D. Broders' histologic grading system correlates well with survival: patients with grade IV
tumors have 5-year survival
rates around 65%.

46. An 80% distal gastrectomy is performed for a 6-cm. antral cancer with extension to the
muscularis propria and three positive lymph nodes less than 3 cm. from the tumor. The
stage of this tumor was:
A. Stage I.
B. Stage II.
C. Stage III A.
D. Stage III B.

47. Which of the following statements about the surgical treatment of gastric cancer is
false?
A. Patients with tumors of the middle and proximal thirds should undergo total gastrectomy.
B. Adenocarcinoma of the cardia-gastroesophageal junction may require reconstruction in the
abdomen, chest, or neck.
C. Palliative resection yields better results than palliative bypass.
D. Japanese patients who undergo gastric resection are, on average, 10 years younger and much
leaner than their Western counterparts.

51. Which of the following is/are contraindications to gastric bypass surgery?


A. Diabetes mellitus.
B. Hypertension.
C. Pickwickian syndrome.
D. Failure to agree to long-term follow-up.
E. Sleep apnea.

53. Metabolic complications of subtotal gastrectomy with Billroth I or Billroth II


reconstruction include: except
A. Hypothyroidism.
B. Anemia.
C. Reactive hypoglycemia.
D. Dumping syndrome.
E. Metabolic bone disease.

62. Which of the following statement(s) concerning pharyngoesophageal disorders is/are


true? except
a. In neuromuscular diseases, dysphagia is often worse for liquids than for solids
b. Cricomyotomy may be indicated for a wide variety of neuromuscular disorders involving the
pharyngoesophageal phase of swallowing
c. Excision of a Zenker’s diverticulum is indicated to prevent malignant change in the sac
d. Complications of all operations on the cervical esophagus include hematoma formation and
recurrent nerve paralysis

67. Which of the following statement(s) is/are true concerning the surgical anatomy of the
esophagus?
b. Spontaneous esophageal perforation tends to be associated with leakage into the left chest

79. Which of the following statement(s) is/are true concerning caustic injury to the
esophagus?
a. Alkaline injury is more destructive than acid injury
b. Acid ingestion is not injurious to the stomach due to its normal acidic pH
c. Ingested caustic agents rapidly pass through the esophagus and stomach into the small
intestine
d. Unless perforation occurs, clinical manifestations resolve quickly with initial clinical
improvement noted
e. Children are less likely to form a late esophageal stricture than adults

85. At a cellular level, the major stimulant(s) of acid secretion by the gastric parietal cell
is/are:
a. Histamine
b. Prostaglandin E2
c. Acetylcholine
d. Gastrin
e. Norepinephrine

86. Which of the following statement(s) regarding the vagus nerves is/are true?
a. The right and left vagus nerves derive from a nerve plexus inferior to the tracheal bifurcation
b. The posterior vagus nerve is closely applied to the intrathoracic esophagus
c. The anterior vagus supplies a hepatic division which passes to the right in the lesser omentum
d. Approximately 90% of vagal fibers are afferent, transmitting information from the
gastrointestinal tract to the central nervous system
e. The vagus nerves transmit gastroduodenal pain sensations associated with peptic ulceration

87. Important stimulants of gastrin release from endocrine cells in the antrum include:
a. Acidification of the antral lumen
b. Small peptide fragments and amino acids from luminal proteolysis
c. Locally released somatostatin
d. Dietary fats

88. Which of the following statements regarding human gastric acid secretion is/are true?
a. Fasting acid secretion, normally 2 to 5 mEq/h, is due to ambient vagal tone and histamine
secretion
b. Truncal vagotomy decreases basal secretion by 80%
c. Histamine2 receptor antagonist administration can decrease basal acid secretion by 80%
d. Fasting acid secretion, normally 5 to 10 mEq/h, is due to circulating levels of gastrin

93. Which of the following statements regarding intrinsic factor is/are correct?
a. Intrinsic factor is produced in chief cells located in the gastric fundus
b. Total gastrectomy is followed by folate deficiency due to vitamin malabsorption secondary to
intrinsic factor
deficiency
c. Intrinsic factor secretion, like that of acid, is stimulated by gastrin, histamine, and
acetylcholine
d. Intrinsic factor deficiency accompanies H pylori-caused antral gastritis

94. A 24-year-old woman develops epigastric pain and has a diagnosis of duodenal ulcer
confirmed by esophagogastroduodenoscopy. The patient is in the third month of a
pregnancy. The most appropriate treatment would be:
a. Proximal gastric vagotomy
b. Misoprostol 400 mg b.i.d.
c. Sucralfate 1 gm q.i.d.
d. Cimetidine 400 mg b.i.d.

104. Which of the following clinical circumstances have been identified as predisposing
factors for the development of stress ulceration?
a. Intraperitoneal sepsis
b. Hemorrhagic shock
c. Isolated tibial fracture
d. 50% total surface area second degree burn
e. Adult respiratory distress syndrome

107. With regard to benign gastric ulceration, the most common location of disease is which
of the following?
a. Along the greater curvature
b. Immediately distal to the esophagogastric junction along the lesser curvature
c. In the area of the incisura angularis along the lesser curvature
d. Within the gastric antrum

110. Agents demonstrated to have an efficacy of greater than 90% for prophylactic
treatment of stress ulceration include which of the following?
a. Antacids
b. H2 receptor antagonists
c. Sucralfate
d. Misoprostil

Ortho
1. Which of the following statements about open fractures are correct? except
A. Intravenous antibiotics should be administered as soon as possible.
B. They should be regarded as an emergency.
C. Wound closure is necessary within 8 hours.
D. Systematic wound débridement and irrigation should be performed.

2. The goals of proper fracture reduction include which of the following? Except
A. Providing patient comfort and analgesia.
B. Allowing for restoration of length of the extremity.
C. Correcting angular deformity and rotation.
D. Enabling immediate motion of all fractured extremities.

6. The neurovascular structure most commonly injured as a result of an anterior


dislocation of the shoulder is the:
A. Musculocutaneous nerve.
B. Axillary nerve.
C. Axillary artery.
D. Median nerve.

7. The classification of fractures of the proximal humerus is based on:


A. The number of fracture segments and amount of displacement.
B. The mechanism of injury.
C. Presence or absence of associated dislocations.

8. The radial nerve is at greatest risk for injury with which fracture?
A. Fracture of the surgical neck of the humerus.
B. Fracture of the shaft of the humerus.
C. Supracondylar fracture of the humerus.
D. Olecranon fractures.

9. The best method of treating a supracondylar fracture of the humerus in a child that is
unstable when the elbow is flexed to 90 degrees is:
A. Hyperflexion of the elbow to 130 degrees and casting.
B. Open reduction and internal fixation.
C. Percutaneous pinning.

10. Both-bone forearm fractures in adults are best managed by:


A. Closed reduction and casting.
B. Closed reduction and application of an external fixator.
C. Open reduction and placement of intramedullary rods.
D. Open reduction and internal fixation with compression plates.

11. The most consistent sign of a fracture of the carpal scaphoid is:
A. Wrist pain during attempted push-ups.
B. Diffuse swelling on the dorsum of the wrist.
C. Localized tenderness in the anatomic snuffbox.
D. Wrist popping on movement.

12. A patient describes a fall on the outstretched hand during sports activities. Multiple
radiographic views show no distinct fracture. He is tender to palpation in the anatomic
snuffbox. The most suitable method of management is:
A. Diagnose “sprained wrist” and apply an elastic bandage.
B. Diagnose suspected scaphoid fracture and apply a short-arm cast to include the thumb.
C. Apply a canvas wrist splint for immobilization.
D. Prescribe salicylates and permit continued activity.

13. Median nerve compression syndrome in which the patient has motor weakness of the
flexor pollicis longus and the flexor digitorum profundus of the index finger without
alteration in sensibility is due to:
A. Compression of the median nerve at the elbow by the lacertus fibrosus.
B. Compression of the median nerve in the axilla.
C. Compression of the anterior interosseous nerve by the arcade of Frohse.
D. Compression of the anterior interosseous nerve by an aberrant accessory forearm muscle.

15. The most common physical findings in a patient with median nerve compression at the
wrist (carpal tunnel syndrome) are:
C. A positive percussion test at the wrist and a positive wrist flexion test producing paresthesias
at the thumb, index, and long fingers.

16. Which of the following describes the most desirable position in which to immobilize the
hand?
E. Wrist is extended, MCP joints are flexed, and IP joints are extended.

17. An early sign of compartment syndrome in the hand includes:


A. Pain with passive stretch of the digits.
B. Absent radial pulse.
C. Motor paralysis.
D. Swelling of the digits.
E. Stiffness of the digits.

18. Palmar dislocation of the PIP joint with fracture:


D. If not splinted properly, will cause a boutonniere deformity.

19. Fracture of the fifth metacarpal neck:


C. Is called a “boxer's fracture.”

20. A Bennett's fracture is:


B. Displaced by the pull of the abductor pollicis longus and adductor pollicis.

27. Prognosis of healing in tibial fractures correlates best with:


A. Energy absorption at the time of fracture.
B. Amount of soft tissue damage.
C. Location of the fracture (i.e., in the proximal, middle, or distal third).
D. Age of patient.

28. Management of a III-b tibia fracture is best treated initially by:


A. Plaster immobilization.
B. Immediate plating.
C. Reamed intramedullary nailing.
D. External fixation.

29. The most frequent forces acting on the foot that cause ankle fractures are:
A. External rotation.
B. Internal rotation.
C. Plantar flexion.
D. Dorsiflexion.

30. Patients who have abduction injuries to the foot are prone to injure the following
structures:
A. Medial malleolus and deltoid.
B. Lateral malleolus and deltoid ligament.
C. Interosseous ligament.
D. Posterior tibiofibular ligament.

31. Of the following bones in the foot, the tarsal bone that is most prone to vascular
compromise is the:
A. Calcaneus.
B. Navicular.
C. Talus.
D. Cuboid.

32. A Lisfranc fracture is a fracture-dislocation involving:


B. Tarsometarsal joint.

33. The most common reason for surgical amputation in the general population is:
E. Ischemia.

36. Hematogenous osteomyelitis most frequently affects:


A. The diaphysis of long bones.
B. The epiphysis.
C. The metaphysis of long bones.
D. Flat bones.

47. The zone of flexor tendon injury that carries the poorest prognosis following injury and
repair is:
B. Zone II.
53. The most important structural component of connective tissue is collagen. Which of the
following statement(s) is/are true concerning types of collagen? except
a. All collagen is fiber forming
b. Type 1 collagen is the most abundant in the human body
c. Type 2 collagen is found in cartilage
d. The basement membrane collagens, type 4 and 5, do not form regular fibers

57. Which of the following statement(s) is/are true concerning bone remodeling? except
a. Remodeling can occur only on the surface of trabeculi
b. The remodeling process takes approximately 120 days in an adult
c. Trabecular bone remodeling occurs up to 10 times faster than cortical bone remodeling
d. Bone modeling involves bone formation without resorption

____________________________________________________________________________

Final Exam / 6th year


1- A 44-year-old man presents with painless rectal bleeding of 1 month's duration. He
reports a history of constipation. He works in heavy labor.
For this patient, which of the following statements regarding internal hemorrhoids is true?

Stapled hemorrhoidectomy should be done for grade 1 and 2 hemorrhoids


All of the above
Internal hemorrhoids are located proximal to the dentate line and therefore are usually painless
A grade 1 internal hemorrhoid represents bleeding with prolapse

2- 39. A 34-year-old woman presents for evaluation of severe and frequent bloody bowel
movements, as well as abdominal pain, dehydration, and anemia. She has had these
symptoms for 2 days. She has not had any similar symptoms in the past, and she has been
in relatively good health.
If the patient has toxic megacolon, under what circumstances emergency surgical
management is indicated?
There is a perforation
Any of the above
The patient's clinical or radiographic status worsens
There is no improvement in 24 to 36 hours after aggressive medical therapy

3 - Complications of untreated pancreatic pseudocysts include all of the following


EXCEPT:
intracystic hemorrhage
abscess
pancreatic necrosis
free rupture
gastrointestinal obstruction
4 - Platelets in the wound form a hemostatic clot and release clotting factors to produce:
thrombin
Fibrin
Fibrinogen
Fibroblasts
thromboplastin

5- A 43-year-old man presents to the office for evaluation of recent weight loss and
frequent loose stools. He is concerned because his father was diagnosed with colon cancer
at the age of 50.
Besides family history, what are some other risk factors for colorectal cancer?
Hypertension
Diabetes
All of the above
Inflammatory bowel disease

6- A 40-year-old woman presents to the office for evaluation of yellowish skin. She states
that over the past few weeks, she has noticed that her eyes and skin have developed a
yellow tint. She also reports that she has dark urine and pale-colored stools. Further
history elicits periodic bouts of right upper quadrant pain after eating. She is otherwise
healthy. She denies using any medications. On physical examination, a yellowish tint is
observed on the patient's skin, sclera, and mucous membranes.
On the basis of this patient's history and clinical examination, which type of bilirubin
would you expect to predominate?
Mixed
Conjugated
Unconjugated
Indirect

7- A 38-year-old man presents with a complaint of a slow-growing mass over his right
parotid gland. The lesion is fixed to the underlying structures and has recently become
painful.
Which of the following features strongly suggests that this patient's lesion is a malignancy?
Ipsilateral numbness of the tongue
All of the above
Overlying skin involvement
Facial nerve paralysis

8- Axillary lymph nodes are classified according to the relationship with the
pectoralis minor muscle
pectoralis major muscle
axillary vein
serratus anterior muscle
latissimus dorsi muscle
9- A 78-year-old man is recovering from abdomino-perineal (A-P) resection for Ca rectum,
which was performed 3 days ago. The patient is now complaining of mild shortness of
breath and chest pain. On physical examination, the patient's right leg is slightly more
swollen than his left. The pulse oximetry reading is 90%.
What is the principal method of diagnosing acute pulmonary embolism?
Magnetic resonance imaging
Chest x-ray
Ultrasound
Spiral computed tomography scanning

10- Compartment syndrome


Passive stretch decrease muscle pain
Due to decrease pressure in muscle compartments
Pulse is the first thing to disappear
Cause severe pain in the limb
Treatment is by delayed fasciotomy

11- The most significant risk factor for the development of adenocarcinoma of the
esophagus is:
lye stricture
alcohol abuse
Barrett's esophagus
long-standing achalasia
smoking

12- All of the following statements are true about patients with carcinoid tumors EXCEPT:
the combination of streptozotocin and 5-fluorouracil (chemotherapy) can often result in objective
response.
tumor growth is often slow
they often have evidence of serotonin production
the majority have carcinoid syndrome
they have a much better prognosis if the tumors are less than 2 cm.

13- A 67-year-old man presents with left-lower-quadrant pain and low-grade fever. He has
had these symptoms for 1 day. The patient denies experiencing any rectal bleeding, but for
the past week, his bowel movements have been irregular.
For this patient, which of the following statements is true regarding diverticular disease?
The sigmoid colon is the most common site of diverticula
All of the above
Most diverticula of the colon involve the muscular layer
Smoking does not seem to be related to the development of diverticular disease

14- All of the following are components of the MEN type 2B syndrome except:
Multiple neuromas on the lips, tongue, and oral mucosa
Medullary thyroid carcinoma
Pheochromocytoma
Hyperparathyroidism.

15- The risk of bilateral breast cancer is HIGHEST if the first breast shows:
inflammatory carcinoma
medullary carcinoma
infiltrating ductal carcinoma
paget’s disease
lobular carcinoma

16- The best initial therapy for deep venous thrombosis of the common femoral vein is:
warfarin
streptokinase
Heparin
venous thrombectomy
placement of a vena caval filter

17- Complications after thyroidectomy include all the following EXCEPT:


recurrent laryngeal nerve paralysis
parathyroid insufficiency
thyrotoxic crisis(storm) on operating on inadequitly prepared thyrotoxic patient
tracheomalacia
hypercalcemia

18
A 55-year-old man presents with hematemesis that began 2 hours ago. He is hypotensive
and has altered mental status. No medical history is available.
For this patient, which of the following statements regarding nasogastric aspiration is true?
None of the above
A clear, nonbilious aspirate rules out the need for EGD
A clear, bilious aspirate rules out the need for EGD
A bloody aspirate is an indication for esophagogastroduodenoscopy (EGD)

19
The major cause of impaired wound healing is:
steroid use
malnutrition
diabetes mellitus
local tissue infection
anemia

20
Common presenting conditions in patients with pancreatic carcinoma include all of the
following EXCEPT:
esophageal varices.
palpable gallbladder
weight loss.
abdominal pain

21
All of the following statements about keloids are true EXCEPT:
Keloid tissue contains an abnormally large amount of collagen
A keloid does not regress spontaneously
Keloid tissue contains an unusually large amount of soluble collagen
A keloid extends beyond the boundaries of the original wound
Keloids or hypertrophic scars are best managed by excision and careful reapproximation of the
wound

22
The treatment of choice for a 40-year-old man who is found on endoscopy and biopsy to
have a gastric lymphoma would be:
wide local excision
subtotal gastrectomy
chemotherapy
subtotal gastrectomy and radiotherapy
Radiotherapy

23
The treatment of an esophageal burn with a caustic agent may include all of the following
EXCEPT:
boogieing. (dilatation)
induction of vomiting
gastrectomy
expeditious administration of an antidote
steroids and antibiotics.

24
A 32-year-old man with a family history of familial adenomatous polyposis (FAP) presents
with hematochezia. He denies having any diarrhea, abdominal pain, or fever.
For this patient, which of the following statements regarding FAP is true?
a. CRC does not occur in patients with FAP if they are given adequate medical treatment
b. Total proctocolectomy (TPC) is considered the only option for the surgical management of
FAP
c. For patients with FAP, there is a 10% risk of CRC by age 40 if prophylactic colectomy is not
performed
d. In the setting of FAP, colorectal cancer (CRC) is more commonly located on the left side

25
When stage I breast cancer is treated by partial mastectomy and axillary dissection,
further therapy should include:
a. antiestrogen agents.
b. radiation of the affected breast.
c. oophorectomy if premenopausal.
d. nothing
e. chemotherapy

26
For the patient in Question 68, which of the following statements is true regarding an
esophageal varix as the site of bleeding?
I.V. propranolol should be administered first
Balloon tamponade should be performed first
Rubber banding or intravariceal sclerotherapy should be performed first
I.V. somatostatin should be administered first

27
Drugs which may produce gynecomastia include all of the following EXCEPT:
furosemide
cimetidine
Verapamil
Diazepam
Tamoxifen

28
Splenectomy is commonly indicated for the following EXCEPT:
hypersplenism associated with cirrhosis
hereditary spherocytosis
splenic tumor
immune thrombocytopenic purpura
grade four splenic injury in trauma

29
An ischiorectal abscess is characterized by all of the following EXCEPT:
Requires deroofing
Should be treated entirely by antibiotics
May be tuberculous in origin
Can be followed by anal fistula
Is an infective necrosis of the fat of the ischiorectal fossa

30
The most common presentation of Meckel’s diverticulum in an adult is:
intussuception
Littre’s hernia
Gastrointestinal bleeding
diverticulitis

31
For the patient in Question 65, which of the following is an indication for immediate
surgery?
Closed-loop obstruction
All of the above
Complete bowel obstruction
Incarcerated hernia

32
Choledocholithiasis in a patient who previously had cholecystectomy is BEST treated with:
endoscopic sphincterotomy
choledochoduodenostomy
choledochojejunostomy.
dissolution with mono-octanoin
open common bile duct exploration with stone removal

33
A 77-year-old man undergoes endoscopic ultrasonography as part of a workup for
jaundice. He is found to have a tumor in the head of the pancreas.
For this patient, which of the following findings would indicate that the tumor is
unresectable?
All of the above
Peritoneal metastases
Invasion of the superior mesenteric artery
Metastases to celiac lymph nodes

34
Regarding polyps of the colon
Villous polyps are usually pedunculated
Villous polyps occur more proximal in colon
Adenomatous polyps are usually solitary
Cancer risk is not related to size of polyp
Metaplastic polyps are not precancerous

35
The most likely diagnosis in elderly patient with abdominal pain and colonoscopy finding
of patchy mucosal ulceration at the splenic flexure of the colon is :
ulcerative colitis
crohns disease
ischemic colitis
diverticulitis
lymphogranuloma venerum

36
A 54-year-old man presents with a neck mass of 2 weeks' duration. He has no significant
medical history. He smokes two packs of cigarettes a day and has been doing so since he
was 21 years of age.
For this patient, which of the following statements is true?
- Low cervical nodes are more likely to contain metastases from a primary source other than the
head and neck, whereas upper cervical nodes are more likely to contain metastases from the head
and neck
- Soft or tender nodes are more likely to derive from an inflammatory or infectious condition,
whereas hard, fixed, painless nodes are more likely to represent metastatic cancer
- All of the above
- Enlarged lymph nodes are by far the most common neck masses encountered

37
The first-choice diagnostic study for suspected deep venous thrombosis of the lower
extremity is:
real-time Doppler imaging
contrast sonography
radioactive labeled fibrinogen uptake
impedance plethysmography
isotope injection with gamma scintillation scanning

38
Decreased PaCO2 levels should be attained in a patient at serious risk for cerebral edema
secondary to a head injury in order to :
prevent neurogenic pulmonary edema
prevent increased capillary permeability
prevent metabolic acidosis
allow reciprocally high levels of PaO2 in the brain
prevent cerebral vasodilation

39
Causes of third space loss include all except:
Acute pancreatitis
Pancreatic fistula
Necrotizing fasciitis
Site of major surgery
Crush syndrome

40
Causes of metabolic acidosis include all except:
Small bowel fistula
Shock
CO poisoning
severe anemia
All of the above

41
A 39-year-old man comes in for evaluation of intermittent anal pain and bleeding after
bowel movements. He also has hard stools. He has had these symptoms for over 1 year .
For this patient, which of the following is included in the classic triad of signs of chronic
anal fissures?
An anal fissure
All of the above
A sentinel skin tag
Hypertrophy of the anal papilla

42
All the following statements about achalasia are true EXCEPT:
it occurs most commonly in persons between the ages of 30 and 50 years
in most affected persons, ganglion cells in the body of the esophagus either are absent or have
degenerated
esophageal cancer is seven times as common in affected persons as in the general population
pressure in the body of the esophagus is lower than normal
affected persons usually experience more difficulty swallowing cold foods than warm foods

43
A patient with the Zollinger-Ellison syndrome is found to have the multiple endocrine
neoplasia type I (MEN-I) syndrome. Appropriate management for the ulcer symptoms
should be:
Omeprazole
pancreatic resection
streptozocin
cimetidine
total gastrectomy

44
Fat absorption occurs primarily in the:
Ileum
third portion of the duodenum
Stomach
Jejunum
first portion of the duodenum

45
39. A 56-year-old woman has been experiencing abdominal pain for 4 hours. The pain is in
right upper quadrant and radiates into the scapular region. She has had multiple episodes
of vomiting.
For this patient, which of the following signs on physical examination is associated with
acute cholecystitis?
Carnett sign
Kehr sign
Murphy sign
Rovsing sign
46
Mammary duct ectasia is characterized by the following EXCEPT :
Is treated usually by simple mastectomy
May present with nipple retraction and Peau d'orange picture
Anaerobic superinfection cmmmonly occurs in this recurrent periductal plasma cell mastitis
Is defined as primary dilatation of major ducts of breast in middle aged women
Is commonly pre-malignant

47
Regarding veins of lower limbs all are true except:
Valves allow flow from deep to superficial system
Venous return from lower limbs is aided by respiratory movements
Superficial veins lie in subcutaneous tissue
The pressure in veins of the foot while standing is 100mmHg
Stasis of blood is important factor in developing varicose veins

48
Most common complication of central venous access is:
major artery damage.
Catheter problems.
Thrombosis of central vein
Catheter related sepsis
Pleural space damage, pneumothorax

49
Factors associated with increase risk of death in acute pancreatitis include all except:
Ranson score more than five
Obesity
High APACHE_II score
Age more than 70 years
sterile necrosis

50
For the patient in Question 56, which of the following chronic conditions can cause a neck
mass?
Tuberculosis
Sarcoidosis
AIDS
All of the above

51
For the patient in Question 56, if metastatic cancer is suspected initially, which of the
following would be the most appropriate step to take next in the workup?
CT scan
Empirical therapy with antibiotics
Fine-needle aspiration (FNA)
Observation only

52
The largest component of intestinal gas is:
Nitrogen
carbon dioxide
Oxygen
ammonia
hydrogen

53
A 66-year-old woman presents to the office complaining of a sharp, constant pain in her
lower abdomen. She has had this pain for the past 2 weeks. Examination of her abdomen is
normal. However, subsequent ultrasound reveals a 4.5 cm Aortic abdominal aneurysm
(AAA). Spiral computed tomography confirms the ultrasound findings.
Aside from rupture, which of the following is a complication associated with (AAA) ?
Aortoenteric fistula
Lower-extremity atheroemboli
All of the above
Thrombosis

54
Diagnosis of esophageal perforation is best established by:
transesophageal ultrasound study
esophagoscopy with a flexible esophagoscope
contrast esophagograms
esophagoscopy with a rigid esophagoscope
upright X-rays of the chest including lateral and oblique films

55
For the patient in Question 68, which of the following is an indication for surgery?
Ongoing hemorrhage occurs from a gastric ulcer in a hemodynamically unstable patient
All of the above
Bleeding continues from either a duodenal ulcer or a gastric ulcer despite medical and
endoscopic therapy
Substantial bleeding occurs from a duodenal ulcer that is not controlled by EGD

56
Yesterday, a 38-year-old woman underwent a laparoscopic cholecystectomy for
cholelithiasis and was discharged home 8 hours after surgery. She returns this morning
complaining of worsening abdominal pain. The oral narcotics that the patient was
prescribed are ineffective in controlling the pain. The patient's temperature is (38.3 C).
Laboratory studies reveal an elevated white blood cell count. Abdominal ultrasonography
shows a large subhepatic fluid collection. The fluid is percutaneously aspirated and reveals
enteric contents.
What step should be taken next in the management of this patient?
Immediate laparotomy
I.V. antibiotics and close observation
None of the above
Observation in the hospital until pain improves

57
The most common symptom after major pulmonary embolism is:
cough
Dyspnea
Hemoptysis
pleural pain
fear of death

58
Treatment of paralytic ileus includes all of the following EXCEPT :
intravenous fluids
cessation of oral intake
correction of electrolyte imbalance
early operation
nasogastric suction

59
Lymphedema :
e) None of the above
b) should be bilateral
c) may be pitting in early stage
d) A & C only
a) may be Congenital

60
The level of consciousness for a head injury patient is BEST evaluated by :
CT scan
visual evoked potentials
Glasgow coma scale
papillary responses
response to pain

61
A 52-year-old male weighing 70 kg, sustained a 65% total body surface area (TBSA) burn.
What are his fluid requirements?
a. 18,200 cc in 16 h
b. 9100 cc in the first 8 h
c. 8000 cc in 24 h
d. 12,800 cc in 24 h

62
Management of cholangitis may include all of the following EXCEPT:
a. decomperession of the common bile duct.
b. cholecystostomy
c. IV antibiotics.
d. correct underlying cause.
e. percutaneous transhepatic cholangiography.

63
A 48-year-old woman presents to the emergency department complaining of right upper
quadrant pain, which began 4 hours ago. She reports the pain as being spasmodic and
sharp and that it radiates to her right shoulder blade. She says that she has had similar
episodes over the past few months, especially after eating large meals. Associated with the
pain is nausea and vomiting. Her blood pressure is 120/85 mm Hg, and her pulse is 100
beats/min. On physical examination, the patient is found to have a nontender abdomen
with no palpable masses. Her chest and cardiovascular examinations are normal. The
nurse notices that her sclerae are slightly icteric. On subsequent laboratory studies, her
serum bilirubin level is found to be 10 mg/dl.
What imaging study should be performed next for this patient with presumed posthepatic
jaundice?
a. Percutaneous transhepatic cholangiography (PTC)
b. Ultrasonography
c. Endoscopic retrograde cholangiopancreatography (ERCP)
d. Magnetic resonance imaging

64
An HIV-positive man presents for evaluation of new oral cavity lesions he discovered last
month. Physical examination reveals purple exophytic masses involving the palate mucosa
and gingiva.
What is the most likely diagnosis of these oral cavity lesions?
a. Oral hairy leukoplakia
b. Non-Hodgkin lymphoma
c. Syphilis
d. Kaposi sarcoma

65
Brain injury alone
a. causes shock only if the skull is intact
b. rarely causes shock
c. causes shock that is reversed by very simple measures
d. frequently causes shock
e. causes shock if hypoxia is superimposed
66
All of the following substances are irritating to the peritoneum EXCEPT:
a. bile.
b. blood
c. gastric content.
d. meconium
e. pus

67
Regarding volvulus of the sigmoid colon, each of the following is true except :
a. likely results from redundant sigmoid colon with an elongated narrow mesocolon
b. there appears to be a congenital predisposition sigmoid volvulus.
c. diagnostic barium enema for sigmoid colon is essential
d. diagnostic x-ray for sigmoid volvulus shows a dilated loop of colon which points toward the
right upper quadrant.

68
Gallstones are characterized by all the following EXCEPT:
a. cause mucocoele of the gall bladder
b. are present in the common bile duct in 40% of patients with stones in the gall bladder
c. are becoming common in post-partum primipara who were pre pregnancy ‘Pill’ takers
d. are frequently the cause of flatulent dyspepsia
e. may be present in the newborn

69
Following a burn, the agent responsible for early increased capillary permeability is
a. thromboxane A2
b. serotonin
c. histamine?
d. prostacyclin PGI2
e. bradykinin

70
Incisonal hernias all are true except:
a. 10 % of all hernias
b. Usually easy to reduce
c. Multiloculated sac
d. Operative technique is most important cause
e. Complication is common

71
In the treatment of gastric cancer, all of the following are true EXCEPT:
a. five-year survival rates in the continue to be between 10% and 25%.
b. Palliative resection is frequently helpful with advanced disease.
c. Lymph node involvement is associated with a poorer prognosis.
d. Finding early disease at the time of operation is associated with a better prognosis.
e. total gastrectomy is mandated in most patient. ***

72
of the involved area and : Severe cases of hidradenitis suppurativa in the groin area are
best managed by excision
a. transfer of a rectus abdominus muscle flap
b. split thickness skin grafting
c. primary closure
d. delayed primary closure
e. closure by secondary intention

73
Patients at increased risk for gastric carcinoma include all the following EXCEPT:
a. those who have undergone gastric bypass for morbid obesity
b. those who have undergone gastric resection for duodenal ulcer
c. those with pernicious anemia
d. those with a high consumption of smoked fish
e. those with blood group A

74
A 56-year-old woman presents with symptoms of abdominal pain, weight loss, and rectal
bleeding. She is anemic and hypotensive, but she is stable.
For this patient, which of the following should be done first if the gastric lavage yields
copious amounts of bile?
a. Arteriography
b. Emergency laparotomy
c. Colonoscopy
d. Esophagogastroduodenoscopy

75
A 39-year-old man presents with lower GI bleeding. He has no abdominal discomfort and
has experienced no loss of weight.
For this patient, which of the following statements regarding the etiology of lower GI
bleeding is true?
a. Lower GI bleeding from diverticulosis often requires surgery
b. Arteriovenous malformations (AVMs) are the most common cause of lower GI bleeding
c. 6% to 10% of patients with ulcerative colitis have lower GI bleeding severe enough to
necessitate emergency surgical resection
d. None of the above

76
For the patient in Question 65, which of the following statements regarding abdominal
radiographic findings is true?
a. In gastric outlet obstruction, no gastric air will be seen, but large amounts of air will be seen in
the small bowel and colon
b. Mechanical small bowel obstruction usually shows no air-fluid levels, nor will distended
bowel loops of similar sizes be seen
c. All of the above
d. High-grade obstruction of the colon in association with an incompetent ileocecal valve may
mimic small bowel obstruction on x-ray

77
A 41-year-old female presents to the emergency department after sustaining a gunshot
wound to the abdomen, with injuries to the liver and large bowel. Despite successful
resuscitation and operative intervention, the patient dies 2 weeks later of multisystem
organ failure in the intensive care unit. Which organ most likely first experienced
dysfunction?
a. lung
b. heart
c. liver
d. gastrointestinal tract
e. kidney

78
39. A 35-year-old man presents with severe acute abdominal pain of sudden onset. The
pain initially began in the upper abdomen and has now settled in the whole abdomen. On
examination, the abdomen was rigid. Chest X-Ray showed free gas under diaphragm.
Which of the following is the most appropriate course of management for this patient?
a. Observation
b. Urgent laparotomy
c. Nonurgent laparotomy
d. Laparoscopy

79
All are true about the dumping syndrome except:
a. Symptoms can be controlled with a somatostatin analogue.
b. Early postoperative dumping after vagotomy often resolves spontaneously.
c. Flushing and tachycardia are common features of the syndrome.
d. Diarrhea is always part of the dumping syndrome.
e. Separating solids and liquids in the patient's oral intake alleviates some of the symptoms of the
syndrome.

80
An anal fissure is characterized by the following EXCEPT:
a. Is more common in women.
b. Is an ulcer of the anal mucosa.
c. Can be treated coservatively.
d. Usually lies anteriorly.
81
All of the following statements are true of esophageal carcinoma EXCEPT
a. squamous cell tumor is the most frequent histology.
b. operation is frequently curative.
c. patient often require a pyloroplasty with operation.
d. patients usually die within one year of diagnosis
e. patient can have intestinal continuity reestablished using the stomach after esophageal
resection.

82
A serum tumor marker correlated with recurrence after management of colon cancer is:
a. carcinoembryonic antigen (CEA)
b. Calcitonin
c. 5-hydroxyindoleacetic acid
d. CA 15-3
e. Alpha-fetoprotein

83
Conditions associated with gastric cancer include all of the following EXCEPT:
a. adenomatous polyps.
b. a high intake of dietary nitrates.
c. pernicious anemia.
d. higher socioeconomic groups.
e. chronic atrophic gastritis.

84
Which of the following is required for addressing any pressure sore?
a. pressure reduction
b. hyperbaric oxygen
c. antibiotics
d. skin flap coverage

Treatment of pressure sores requires relief of pressure with special cushions and beds and
nutritional support to promote healing.

85
The most effective treatment of achalasia is:
a. antireflux surgical procedures
b. Esophagomyotomy
c. dilation of the lower esophageal sphincter
d. antispasmodic medication
e. resection of the cardioesophageal junction

86
Regarding desmoid tumors all are true except:
a. Treatment is wide local excision with safety margins
b. Has high rate of local recurrence
c. Is a low grade fibro sarcoma
d. More common in females
e. Is radio resistant

87
Which of the following variables best predicts prognosis for patients with a recent
diagnosis of cutaneous melanoma and no clinical evidence of metastatic disease?
a. Ulceration.
b. Clark's level.
c. Gender
d. Bleeding
e. Breslow thickness.

88
In the treatment of acute cholecystitis, most patient are BEST served with:
a. endoscopic sphincterotomy.
b. cholecystostomy.
c. percutaneous drainage of the gallbladder.
d. early cholecystectomy (within 3 days of onset of symptoms).
e. IV antibiotics and cholecystectomy in 6 to 8 weeks.

89
Regarding crohn's disease all are true except
a. Cause non caseating granuloma
b. commonly affect small bowel
c. Causes sub mucosal fibrosis
d. It is full thickness inflammation
e. Cause depletion of goblet cell mucin

90
Advantages of laparoscopic versus open cholecystectomy include all of the following
EXCEPT:
a. decreased pain.
b. less risk of bile duct injury.
c. reduced hospitalization.
d. improved cosmetic.
e. reduced ileus.

91
The most common cause of massive hemorrhage in the lower gastrointestinal tract is:
a. carcinoma
b. Diverticulosis
c. Diverticulitis
d. Polyp
e. ulcerative colitis

92
Carcinoma of the gallbladder is
a. rarely associated with jaundice.
b. associated with a good prognosis.
c. usually not diagnosed preoperatively.
d. most commonly metastatic to the lung.
e. best treated with radiation and chemotherapy

93
All the following statements concerning nipple discharges are true EXCEPT:
a. benign duct papillomas are the most common cause of bloody discharges.
b. when bloody , the discharge is due to a malignancy 70% of the time
c. a) excision of involved duct may be necessary to determine the etiology
d. they may be caused by multiple lesions.
e. a milky discharge may be due to a pituitary adenoma .

94
A 28-year-old woman presents with a complaint of a growing, painless mass in her neck.
Physical examination reveals a firm, fixed nodule measuring 2 cm on the right lobe of her
thyroid. The surgeon recommends fine-needle aspiration (FNA) of the lesion instead of
excisional biopsy.
What are the advantages of FNA over excisional biopsy?
a. FNA is associated with a decreased risk of tumor seeding
b. FNA requires only an office visit
c. Anesthesia is not necessary
d. All of the above

95
For the patient in Question 74, which of the following is a sign of a colovesical fistula
associated with diverticulitis, as seen on computed tomography with contrast?
a. Thickening of the bladder and the colon
b. Sigmoid diverticula
c. All of the above
d. Air in the bladder

96
All of the following are true statements concerning paget ’ s disease of the nipple EXCEPT:
a. it is an eczematoid lesion.
b. it is very uncommon, accounting for only 2% of all breast cancers.
c. it is an in situ squamous cell malignancy of the nipple.
d. it can be confused with malignant melanoma histologically
e. it has a better prognosis than the majority of other breast cancers.
97
The classical picture of Acute arterial embolism include all the following except:
a. peripheral pulses
b. Parasthesia
c. All the above
d. Pain
e. Pallor

98
Extra colonic manifestation of inflammatory bowel disease include all except:
a. Erythema nodosum
b. Mixed connective tissue disease
c. Primary Sclerosing cholangitis
d. Polyarthopathy
e. Episcleritis

99
A 45-year-old woman presents with abdominal pain and vomiting of 1 day's duration. The
patient underwent an exploratory laparotomy after a motor vehicle accident 8 years ago.
Which of the following statements is true for this patient?
a. Pain from mechanical obstruction is usually more severe than pain from ileus
b. The severity of pain from mechanical obstruction may decrease over time
c. Pain from mechanical obstruction is usually localized in the middle of the abdomen, whereas
pain from ileus or pseudo-obstruction is diffuse
d. All of the above

100
A 4-year-old girl has been experiencing dyspnea on exertion. Chest x-ray shows a
Morgagni hernia. For this patient, which of the following statements is true?
a. Morgagni hernias can be repaired with a subcostal, a paramedian, or a midline incision
b. Morgagni hernias are most commonly seen on the right side
c. All of the above
d. The average age at diagnosis is typically greater for patients with Morgagni hernia than for
patients with Bochdalek hernia

____________________________________________________________________________

28.A midline neck mass, located between the thyroid bone and suprasternal notch that moves
upward when the tongue is stuck out most likely is:
thyroglossal duct cyst-

29.A smooth cystic neck mass located along the anterior border of the sternocleidomastoid
muscle most likely is:
brachial cleft cyst –
30.A lymphatic neck cyst located in the posterior triangle just above the clavicle most likely is:
cystic hygroma-

65.Battle's sign of basilar skull fracture refers to blue-gray discoloration:


behind the mastoid process =

118.The most common complication of thyroidectomy is:


hypoparathyroidism =

What is the most common tumor of the appendix?


carcinoid =

123.The classical triad for a ruptured abdominal aneurysm includes a pulsatile abdominal aortic
mass, back pain, and:
jaundice
vomiting
headache

126.Sigmoid volvulus should be initially treated with:


decompression=

133.Compression of the oculomotor (3rd) nerve by an aneurysm, most likely from:


posterior communicating artery=

210.The most common complication of a major hip fracture is:


hemorrhage =

211.The most common complication of dislocation of the knee is:


popliteal artery injury =

247.Which of the following is the most common indication for major surgery in females?
uterine leiomyomas (fibroids)=

300.Schatzki's ring is associated with:


hiatal hernia =

311.Benign bone tumors are not painful EXCEPT for:


endochondroma
osteoid osteoma -
giant cell tumors
unicameral bone cysts

567.Osteoid osteomas:
are malignant tumors
generally occur after 50 years of age
cause progressive localized ache -
occur in females more commonly than in males

312.Ewing's tumor is a:
malignant sarcoma-

329.A pseudocyst following an acute pancreatitis does NOT:


rupture
form an abscess
resolve spontaneously -
cause an internal hemorrhage

330.Which of the following suggests bad prognosis in a 58 year old man with acute pancreatitis
total bilirubin of 1.0 mg/dL
blood glucose of 240 mg/Dl-
serum amylase of 120 U/L
white blood cell count of 14,000/mm3

568.The most common malignant tumor of the spine is:


multiple myeloma –

332.In trauma patients, which of the following should be treated first?


flail chest-
pleural effusion
laceration of the liver
fractured tibia and ulna

334.12 year old male complains of painful defecation, the most likely cause is:
anal cancer
anal abscess
anal fissure -
internal hemorrhoids

336.Following a major surgery, a 68 year old man develops massive colonic dilatation and
constipation, the most likely cause is:
paralytic ileus
toxic megacolon
fulminating colitis
Ogilvie's syndrome (pseudo-obstruction) -

353.The most common surgical procedure performed in patients with sickle cell anemia is:
splenectomy
gastrectomy
appendectomy
cholecystectomy-
366.Which of the following is NOT a risk factor for colorectal cancer?
Turner's syndrome-
Turcot's syndrome
Gardner's syndrome
Oldfield's syndrome

507.Kohler disease is osteochondrosis of the:


navicular-

508.Sever disease is osteochondrosis of the:


apophysis of the os calis -

511.(Blade of grass) or (flame like) lesion on long bone X-ray and osteoporosis circumscripta are
features of:
Paget's disease of the bone -

524.Serum alkaline phosphatase is normal in:


osteoporosis -
osteomalacia
primary hyperparathyroidism

525.Bone X-ray shows pseudofractures (Looser's zones) in:


osteomalacia-

526.The characteristic dorsal kyphosis and cervical lordosis (dowager's hump) are features of:
osteoporosis-

527.Following are secondary causes of osteoporosis, EXCEPT:


obesity -
anorexia nervosa
Marfan's syndrome
hyperprolactinemia

528.72 year old woman presents with back pain, X-ray shows compression fractures and
kyphosis in the spine, the most likely diagnosis is:
osteoporosis-

529.Back pain with increased urinary hydroxyproline and serum alkaline phosphatase in a 70
year old man is most likely due to:
Paget's disease of the bone -

530.Following are laboratory abnormalities seen patients with Paget's disease, EXCEPT
hypocalcemia -
hyperuricemia
increased urinary hydroxyproline
increased serum alkaline phosphatase

535.An acute upper GI bleeding from longitudinal esophageal tears after repeated vomiting
occurs in:
Mallory-Weiss syndrome -

548.Breast cancer my occur in women who:


are overweight -
had 5 or more children
went in menopause at the age of 38
had their first menstruation at the age of 20

549.Definitive diagnosis of breast cancer is made by:


X-ray
CT scan
mammogram
histological examination-

558.Bright red blood on the surface of the feces with pain during defecation in an 8 year old boy
is most likely due to:
anal fissure -

560.The best management for a ganglion tumor is:


surgery -

570.64 year old man complains of nocturia, dribbling and urinary hesitancy, the most likely
diagnosis is:
benign prostatic hypertrophy-

407.The most common site of bone metastasis from prostate cancer is:
pelvis-

408.The most common indication for surgery in regional enteritis is:


fistula
abscess
bleeding
obstruction-

413.Anal fissure is associated with:


ischemic colitis
Crohn's disease -
ulcerative colitis
irritable bowel syndrome

415.2 year old child has hematuria, hypertension and a palpable mass in the left flank, the most
likely diagnosis is:
Wilms' tumor-

416.Intussusception is characterized by:


projectile vomiting
currant-jelly stools -
olive-shaped mass in upper abdomen
all of the above

418.Which of the following types of kidney stones is radiotransparent?


uric acid -

423.The most common cause of hypercalcemia in a 32 year old woman is:


primary hyperparathyroidism-

424.What type of breast cancer has the worst prognosis?


Lobular-
medullary
inflammatory
infiltrating ductal

430.Which of the following is NOT a risk factor for osteoporosis?


Multiparity-
lack of exercise
low calcium intake
excessive alcohol intake

440.A transilluminated, soft mass in the posterior triangle just above the clavicle of an infant is
most likely due to:
cystic hygroma-

441.What percentage of congenital diaphragmatic hernias in neonates are located on the left
side?
85% -

446.The most common bacteria found in wound infections is:


staphylococcus-

459.The most common cause of trochlear nerve palsy is:


trauma -

480.Diagnosis of Meckel's diverticulum is made by:


technetium scan -

493.What percentage of abdominal aortic aneurysms are diagnosed while they are still
asymptomatic?
75% -
495.36 year old man presents with acute abdomen due to small bowel obstruction, the most
likely cause is:
adhesions -

496.The mortality rate for colonic obstruction is about:


20%-

Ewing sarcoma most commonly occur in:


femur and tibia-

Tachypnea, cyanosis, dyspnea, and severe acidosis within few hours after birth is most likely due
to:
diaphragmatic hernia--

Renal biopsy is contraindicated in the presence of:


solitary or ectopic kidney --

(coffee-bean) shape of dilated bowel loop on plain X-ray and (ace of spades) shape on barium
enema are typical of:
volvulus --

957.Massive lower gastrointestinal bleeding in elderly patients is due to:


diverticulosis
angiodysplasia
both--

.A fragment of fractured bone pulled off by ligament or tendon at its attachment, this is called:
avulsion --

964.A break with 3 or more fragments of a fractured bone is called:


comminuted--

968.Approximately what percentage of patients with cirrhosis have esophageal varices?


50%--

933.Paralysis of abduction and adduction of all fingers is due to injured:


ulnar nerve --
radial nerve
median nerve

Pain, paresthesia, and numbness over the bottom of the foot at night is most likely due to:
tarsal tunnel syndrome --

862.The most common bone tumor in a 64 year old man is:


metastatic tumor--
873.The most common glenohumeral dislocation is:
anteroinferior--

32 year old woman presents with a solitary thyroid nodule, the best management is:
fine-needle aspiration--

816.What is the most common cause of primary hyperparathyroidism?


single adenoma --

820.Carpal tunnel syndrome is LEAST likely to be a feature of:


leukemia
myxedema
hypocalcemia
Parkinson's disease --

62 year old man is unable to fully extend his right 4th finger, the most likely diagnosis is:
Dupuytren's contracture --

32 year old man presents with a painful nodule on the dorsum of the hand, the most likely
diagnosis is:
ganglion-

703.A man presents with a painful encapsulated nodule near the nail, the most likely diagnosis
is:
glomus tumor-

Which of the following is NOT included in the four P's of compartment syndrome?
pallor
paralysis
penetration-
pulselessness

789.26 year old man presents with swelling around the margin of the nail plate of his right index,
the most likely diagnosis is:
paronychia -

754.62 year old man presents with sudden onset of severe pain in his left leg, the leg is pale and
the pulse is absent, the most likely diagnosis is:
arterial embolism -

One-third of individuals with inflammatory involvement of the terminal ileum have:


gallstones -

736.The first and the most common sign/s of obstruction of extrahepatic system is/are:
progressive jaundice -
716.Osteoporosis is characterized by increased:
serum alkaline phosphatase
serum parathyroid hormone
serum calcium and phosphorus
the rate of the bone resorption -

717.Which of the following is NOT a risk factor for osteoporosis?


heparin therapy
copper deficiency
Marfan's syndrome
vitamin A deficiency -

718.Osteomalacia is characterized by:


bone pain
proximal muscle weakness
both -

719.Paget's disease of the bone is characterized by increased serum:


calcium
alkaline phosphatase-

What percentage of gallstones are symptomatic?


30% -

1016. Following are predisposing factors for colorectal cancer, EXCEPT:


50 years of age or older
consumption of vegetables
upper socioeconomic group
consumption of calories and meat protein

1019. Hepatocellular adenomas are benign tumors of the liver found


predominantly in females in their:
3rd and 4th decades

1020. Hepatocellular carcinoma is up to four times more common in men than in


women and with the peak incidence in the:
5th and 6th decades
1021. Patients with severe abdominal pain often obtain relief by sitting with the
trunk flexed and knees drawn up in:
acute pancreatitis
1022. Paget's disease (osteitis deformans) is characterized by increased:
serum calcium
serum phosphate
both
neither
1023. Acute cholecystitis is characterized by the triad of sudden onset of right
upper quadrant tenderness, fever, and:
leukocytosis

1024. Which of the following bone tumors is the most common one in a 16 year
old boy?
osteosarcoma

. Few days after coronary artery bypass surgery, a 74 year old man develops melena,
the most likely diagnosis is:
ischemic colitis

1044. A patient is unable to oppose his thumb to the base of the little finger, which of the
following nerves is most likely to be injured?
median

1069.The most accurate method for the diagnosis of thrombophlebitis of the deep veins is:
ascending contrast venography

1070.The most common cause of septic thrombophlebitis is:


staphylococcus

1071.The most common initial presentation of chronic venous insufficiency is:


pain
edema
itching
hyperpigmentation
1152. Approximately what percentage of acute cholecystitis is NOT associated with
gallstones?
10%

1153. 62 year old man has a metastatic renal cancer, the most likely source is the cancer of:
lung
colon
liver
prostate
1159. Erysipelas is usually caused by:
group A beta-hemolytic streptococci
1173. 28 year old woman presents with breast pain, the most likely diagnosis is:
fibrocystic disease

1174. 46 year old woman presents with bullae and scarring on the dorsum of her hands, the
most likely diagnosis is:
porphyria cutanea tarda
1195. How long after surgery, wound infections classically become apparent?
4 to 7 days

1032. Romberg How ship sign is associated with:


obturator hernia

1033. Double bubble sign on X-ray is associated with:


annular pancreas
duodenal atresia
intestinal malrotation
all of the above
A woman has phlebitis of the superficial veins in the outer quadrants of the breast, the
1219.
likeliest diagnosis is:
Paget's disease
Mondor's disease =
plasma cell mastitis
fibrocystic adenoma
1220. Patients with portal hypertension are LEAST likely to have:
hemorrhoids
facial edema =
caput medusa
Budd-Chiari syndrome
Caput medusae (abnormal wall collaterals increased in size, radiating from the
1221.
umbilicus) is a feature of:
portal hypertension=

Following are important diagnostic aids for peripheral arterial diseases, EXCEPT:
ultrasound
thermography =
plethysmography
plain X-ray studies
The (five P's) of acute arterial occlusion are pain, pallor, and all of the following,
EXCEPT:
pulseless
paralysis
paresthesia
propagation ==
Buerger's disease is an inflammatory disease that involves:
veins
arteries
both =
The (gold standard) of the diagnosis of deep venous thrombosis (DVT) is:
plethysmograph
contrast venogram =
doppler ultrasound
radionuclide venogram
The most reason women bring their varicose veins to the attention of the physician is:
leg pain
leg swelling
purely cosmetic =
statsis dermatitis
What percentage of patients with proved pulmonary emboli will have demonstrable
deep venous thrombosis?
95% =
Dilated veins on chest wall with swelling of neck and upper extremities in a 56 year
old man is most likely due to:
filariasis
lung cancer =
Buerger's disease
abdominal aortic aneurysm

The most decisive way to confirm the diagnosis of splenic rupture is:
plain X-ray
ultrasound
exploratory laparotomy=
CT scan of the abdomen

An abrupt cutoff of colnic gas in the region of the splenic flexure (colon cutoff sing) is
associated with:
diverticulosis
Crohn's disease
chronic pancreatitis
sever hemorrhagic pacreatitis =

The most common cause of a lower abdominal mass in males over 50 years of age
is:
Hodgkin's disease
colon cancer
retroperitoneal cysts
bladder obstructed by benign prostatic hypertrophy =

An elderly male presents with urinary obstruction, the chance of having a prostatic
cancer is:
20% =

Rupture of the posterior prostatic urethra is associated with:


scrotal swelling
fractures of the pelvis =
dripping of blood from urethra
he matoma under Buck's facia

What percentage of polyps and carcinomas of the large intestine arise within reach of
the sigmoidoscope?
60% =

What is the chance for patients with diverticulosis of developing diverticulitis?


1 in 6 =

The most common type of polyp of the large bowel is:


tubular =
villous
juvenile
Peutz-Jeghers

Injury to nerve root C6 (Intervertebral level C5-C6) causes motor deficit in:
Biceps=

Injury to nerve root C7 (Intervertebral level C6-C7) causes motor deficit in:
triceps =

Testicular tumors:
are benign in most cases
constitute 25% of all male malignant tumors
are choriocarcinomas in most cases
should be removed if malignancy suspected =

Complication of acute appendicitis


hemorrhage
intestinal obstruction
malignant transformation
rupture and periappendicual abscess =

Tylosis is associated with carcinoma of:


lung
colon
esophagus =
pancreas

749.Whipple's disease is diagnosed by:


jejunal biopsy -

Management of a cold thyroid nodule does NOT include:


ultrasound
needle biopsy
surgical excision
injection of thyroxine into the nodule –

314. Epulis is a swelling of:


gums-

49. Paralysis of the serratus anterior cause:


winging of the scapula

A femoral hernia with only a portion of bowel passed through hernia ring is called:
incarcerated hernia
Richter's hernia =
incisional hernia
strangulated hernia

Cryptorchidism is almost invariably associated with:


indirect inguinal hernia =

Which of the following pancreatic islet cells secret serotonin?


enterchromaffin cells=

Patients with necrotizing migratory erythema are most likely to have:


glucagonoma =

14 year old boy complains of aching in the front of the knee after exercise,
1242.
the most likely diagnosis is:
Osgood-Schlatter disease (tibial epiphysitis) =
1244. Regarding Ewing's tumor, which of the following in NOT true?
it is radiosensitive
usually occurs in ages 1 to 4 years =
usually affects diaphysis of the long bones
most commonly affects tibia, fibula, humerus and fermur

Pretest Surgery ( 1 )
A pregnant woman in her 32nd wk of gestation is given magnesium sulfate for pre- .1
eclampsia . The earliest clinical indication of hypermagnesemia is
A. Loss of deep tendon reflexes
B. Flaccid paralysis
C. Respiratory arrest
D. Hypotension

The chief surgical risk to which patients with polycythemia vera are exposed is that due .2
to
A. Anemic disturbances
B. Hemorrhage
C. Infection
D. Renal dysfunction

The surgeon should be particularly concerned about which coagulation function in .3


?patients receiving anti-inflammatory or analgesic medications
A. APTT
B. PT
C. Reptilase time
D. Bleeding time

The substrate depleted earliest in the postoperative period is .4


A. Branched-chain amino acids
B. Non-branched-chain amino acids
C. Ketone
D. Glycogen
E. Glucose
Signs and symptoms of hemolytic transfusion reactions include .5
A. Hypothermia
B. Hypertension
C. Polyuria
D. Abnormal bleeding

A patient suspected of having a hemolytic transfusion reaction should be managed with .6


A. Removal of nonessential foreign body irritants, e.g., Foley catheter
B. Fluid restriction
C. 0.1 M HC1 infusion
D. Steroids
E. Fluids and mannitol

In a hemolytic reaction caused by an incompatible blood transfusion, the treatment that .7


is most likely to be helpful is
A. Promoting a diuresis with 250 ml of 50% mannitol
B. Treating anuria with fluid and potassium replacement
C. Acidifying the urine to prevent hemoglobin precipitation in the renal tubules
D. Removing foreign bodies, such as Foley catheters, which may cause hemorrhagic
complications
E. Stopping the transfusion immediately

The enteric fluid with an electrolyte (Na+, K+, C1 ) content similar to that of .8
Ringer’slactate is
A. Saliva
B. Contents of small intestine
C. Contents of right colon
D. Pancreatic secretions
E. Gastric juice

Which of the following medications administered for hyperkalemia counteracts the .9


?myocardial effects of potassium without reducing the serum potassium level
A. Sodium polystyrene sulfonate(Kayexalate)
B. Sodium bicarbonate
C. 50% dextrose
D. Calcium gluconate
E. Insulin
Hypocalcemia is associated with .10
A. Acidosis
B. Shortened QT interval
C. Hypomagnesemia
D. Myocardial irritability

Which of the following characteristics of this patient might increase the risk of a wound .11
?infection
A. History of colon surgery
B. Hypertension
C. Male sex
D. Receipt of chemotherapy
E. Asthma

?A correct statement regarding 5% dextrose in 0.9% saline is which of the following .12
A. It contains the same concentration of sodium ions as does plasma
B. It can be given in large quantities without seriously affecting acid-base balance
C. It is isosmotic with plasma
D. It has a pH of 7.4
E. It may cause a dilutional acidosis

?Correct statements regarding lactated Ringer’s solution include which of the following .13
A. It contains a higher concentration of sodium ions than does plasma
B. It is most appropriate for replacement of nasogastric tube losses
C. It is isosmotic with plasma
D. It has a pH of less than 7.0
E. It may induce a significant metabolic acidosis

Prophylactic regimens of documented benefit in decreasing the risk of postoperative .14


thromboembolism include
A. Early ambulation
B. External pneumatic compression devices placed on the upper extremities
C. Elastic stockings
D. Leg elevation for 24 h postoperatively

Signs and symptoms associated with early sepsis include .15


A. Respiratory acidosis
B. Decreased cardiac output
C. Hypoglycemia
E. Cutaneous vasodilation

The most common physiologic cause of hypoxemia is .16


A. Hypoventilation
B. Incomplete alveolar oxygen diffusion
C. Ventilation-perfusion inequality
D. Pulmonary shunt flow

Generally accepted indications for mechanical ventilatory support include .17


A. PaO2 of less than 70 kPa and Pa greater than 50 kPa while breathing room air
B. Alveolar-arterial oxygen tension difference of 150 kPa while breathing 100% O2
C. Vital capacity of 40–60 mL/kg
D. Respiratory rate greater than 35 breaths/min
E. A dead space:tidal volume ratio (VD /VT) less than 0.6 CO2 of

Which of the following inhalation anesthetics accumulates in air-filled cavities during .18
?general anesthesia
A. Diethyl ether
B. Nitrous oxide
C. Halothane
D. Methoxyflurane
E. Trichloroethylene

Major alterations in pulmonary function associated with adult respiratory distress .19
syndrome (ARDS) include
A. Hypoxemia
B. Increased pulmonary compliance
C. Increased resting lung volume
E. Decreased dead space ventilation

Dopamine is a frequently used drug in critically ill patients because .20


A. At high doses it increases splanchnic flow
B. At high doses it increases coronary flow
C. At low doses it decreases heart rate
D. At low doses it lowers peripheral resistance

If end-diastolic pressure is held constant, increasing which of the following will increase .21
?the cardiac index
A. Peripheral vascular resistance
B. Pulmonary wedge pressure
C. Heart rate
D. Systemic diastolic pressure

The preoperative characteristics of patients likely to experience postoperative ischemia .22


after non cardiac surgery include
A. Angina
B. More than three premature ventricular contractions (PVCs) per minute
C. Dyspnea on exertion
D. Tricuspid regurgitation

Compensatory mechanisms during acute hemorrhage include .23


A. Decreased cerebral and coronary blood flow
B. Decreased myocardial contractility
C. Renal and splanchnic vasodilation
D. Increased respiratory rate
E. Decreased renal sodium resorption

An 18-year-old woman develops urticaria and wheezing after an injection of penicillin. .24
Her blood pressure is 120/60 mm Hg, heart rate is 155 beats/min, and respiratory rate is 30
breaths/min. Immediate therapy should include
A. Intubation
B. Epinephrine
C. Beta blockers
D. Iodine
E. Fluid challenge

During blood transfusion, clotting of transfused blood is associated with .25


A. ABO incompatibility
B. Minor blood group incompatibility
C. Rh incompatibility
D. Transfusion through Ringer’s lactate
When an arterial blood gas determination of PCO2 40 kPa is obtained .26
A. There is probably a paradoxical aciduria
B. Alveolar ventilation is adequate
C. Arterial P will indicate the adequacy of alveolar ventilation
D. Arterial PO2 will indicate the degree of ventilation-perfusion mismatch
E. Arterial PO2 can be safely predicted to exceed 90 kPa on room air

Among patients who require nutritional resuscitation in an intensive care unit, the best .27
evidence that nutritional support is adequate is
A. Urinary nitrogen excretion levels
B. Total serum protein level
C. Serum albumin level
D. Serum transferrin levels

Paradoxical aciduria (the excretion of acid urine in the presence of metabolic alkalosis) .28
may occur in the presence of
A. Release of inappropriate antidiuretic hormone
B. Severe crush injury
C. Acute tubular necrosis
D. Gastric outlet obstruction

If a patient suffered a pulmonary arterial air embolism during an open thoracotomy, .29
the anesthesiologist’s most likely observation would be
A. Unexpected systemic hypertension
B. Rising right atrial filling pressures
C. Reduced systemic arterial oxygen saturation
D. Rising systemic CO partial pressures
E. Falling end-tidal CO2

A 72-year-old man undergoes resection of an abdominal aneurysm. He arrives in the .30


ICU with a core temperature of 33°C (91.4°F) and shivering. The physiologic consequence
of the shivering is
A. Rising mixed venous oxygen saturation
B. Increased production of carbon dioxide
C. Decreased consumption of oxygen
D. Rising base excess
E. Decreased minute ventilation

To prepare for operating on a patient with a bleeding history diagnosed as von .31
Willebrand’s disease (recessive), you would give
A. High-purity factor VIII:C concentrates
B. Low-molecular-weight dextran
C. Fresh frozen plasma (FFP)
D. Cryoprecipitate
E. Whole blood

The accidental aspiration of gastric contents into the tracheobronchial tree should be .32
initially treated by
A. Tracheal intubation and suctioning
B. Steroids
C. Intravenous fluid bolus
D. Cricothyroidotomy

In performing a tracheostomy, authorities agree that .33


A. The strap muscles should be divided
B. The thyroid isthmus should be preserved
C. The trachea should be entered at the second or third cartilaginous ring
D. Only horizontal incisions should be used

If malignant hyperthermia is suspected intraoperatively .34


A. Complete the procedure but pretreat with dantrolene prior to future elective surgery
B. Administer inhalational anesthetic agents
C. Administer succinylcholine
D. Hyperventilate with 100% oxygen
E. Acidify the urine to prevent myoglobin precipitation in the renal tubules

Central venous pressure (CVP) may be decreased by .35


A. Pulmonary embolism
B. Hypervolemia
C. Positive-pressure ventilation
D. Pneumothorax
E. Gram-negative sepsis
Characteristics of continuous arteriovenous hemofiltration (CAVH) in the treatment of .36
surgical patients with acute renal failure include
A. CAVH is useful only in hemodynamically stable patients
B. CAVH requires placement of largebore(8 French) arterial and venous catheters, usually in
the femoral vessels
C. CAVH is not effective in treating hypervolemia
D. Continuous heparinization of the patient who undergoes CAVH is unnecessary
E. During CAVH, blood flow is maintained by a mechanical
extracorporeal pump–oxygenator

Signs and symptoms of unsuspected Addison’s disease include .37


A. Hypothermia
B. Hypokalemia
C. Hyperglycemia
D. Hyponatremia
E. Hypervolemia

The etiologic factor implicated in the development of pulmonary insufficiency following .38
major non thoracic trauma is
A. Aspiration
B. Atelectasis
C. Fat embolism syndrome
D. Fluid overload

For the severely traumatized patient requiring airway management .39


A. Awake endotracheal intubation is indicated in patients with penetrating ocular injury
B. Steroids have been shown to be of value in the treatment of aspiration of acidic gastric
secretions
C. The stomach may be assumed to be empty only if a history is obtained indicating no
ingestion of food or liquid during the prior 8 h
D. Intubation should be performed in the emergency room if the patient is unstable
E. Cricothyroidotomy is contraindicated in the presence of maxillofacial injuries

Treatment for clostridial myonecrosis (gas gangrene) Includes which of the following .40
?measures
A. Administration of an antifungal agent
B. Administration of antitoxin
C. Wide debridement
D. Administration of hyperbaric oxygen

An abnormal ventilatioperfusion ratio (Qs/Qr) in the postoperative patient has been .41
associated with
A. Pulmonary thromboembolism
B. Lower abdominal surgery
C. starvation
D. The upright position
E. Increased cardiac output

Correct statements concerning drowning or near-drowning include which of the .42


?following
A. The prognosis for recovery of cerebral function in affected persons is better if submersion
occurs in warm water rather than extremely cold water
B. A majority of victims will demonstrate a severe metabolic alkalosis
C. Prompt administration of corticosteroids to affected persons has been shown to decrease
the extent of pulmonary membrane damage
D. Renal damage may occur in affected persons as a result of hemoglobinuria
E. The most important initial treatment of drowning victims is emptying the stomach of
swallowed water
Spontaneous retroperitoneal hemorrhage during anticoagulant therapy .43
A. Is best confirmed by bleeding scan
B. Is equally likely with parenteral and oral anticoagulants
C. May mimic an acute surgical abdomen
D. Frequently requires laparotomy for ligation of the bleeding site

Indications for surgical intervention to remove smuggled drug packets that have been .44
ingested include
A. Refusal to take high doses of laxatives
B. Refusal to allow endoscopic retrieval
C. Refusal to allow digital rectal disimpaction
D. Intraintestinal drug packets evident on abdominal x-ray in an asymptomatic smuggler
E. Signs of toxicity from leaking drug packets

Pretest Surgery ( 2 )
1. Wasting of the intrinsic muscles of the hand can be expected to follow injury of the
A. Ulnar nerve
B. Radial nerve
C. Brachial nerve
D. Axillary nerve

2. A 30-year-old man is stabbed in the arm. There is no evidence of vascular injury, but he
cannot flex his three radial digits. He has injured the
C. Median nerve

3. Although wide surgical excision is the traditional treatment for malignant melanoma,
narrow excision of thin (less than 1 mm deep) stage I melanomas has been found to be
equally safe and effective when the margin of resection is as small as
C. 1 cm

4. With regard to wound healing, which one of the following statements is correct?
B. Monocytes are essential for normal wound healing

5. While you are on duty in the emergency room, a 12-year-old boy arrives with pain and
inflammation over the ball of his left foot and red streaks extending up the inner aspect of
his leg. He remembers removing a wood splinter from the sole of his foot on the previous
day. The most likely infecting organism is
E. Streptococcus

6. The appropriate antibiotic to prescribe while awaiting specific culture verification is


A. Penicillin

7. Proper treatment for frostbite consists of


D. Immersion of the affected part in water at 40–44°C (104–111.2°F)

8. The true statement regarding tendon injuries in the hand is


E. The process of healing a tendon injury involves formation of a tenoma

9. Which one of the following cases is considered a clean contaminated wound


A. Open cholecystectomy for cholelithiasis
B. Herniorrhaphy with mesh repair
C. Lumpectomy with axillary node dissection
D. Appendectomy with walled-off abscess

10. True statements regarding squamous cell carcinoma of the lip include
A. The lesion often arises in areas of persistent hyperkeratosis
B. More than 90% of cases occur on the upper lip
C. The lesion constitutes 30% of all cancers of the oral cavity
D. Radiotherapy is considered inappropriate treatment for these lesions

11. Which of the following statements regarding carpal tunnel syndrome is correct?
B. It may be associated with pregnancy

12. Which of the following is true with regard to wound contraction?


D. It is based on specialized fibroblasts that contain actin myofilaments

13. Management of leukoplakia of the oral cavity includes


A. Excisional biopsy of all lesions
B. Application of topical antibiotics
C. Low-dose radiation therapy
D. Ascertaining that dentures fit properly

14. An 8-lb infant, born following uncomplicated labor and delivery, is noted to have a
unilateral cleft lip and palate .The parents should be advised that
A. The child almost certainly has other congenital anomalies
B. Rehabilitation requires adjunctive speech therapy
C. Lip repair is indicated at 1 year of age
D. Palate repair is indicated prior to 6 mo of age

15. A teenage boy falls from his bicycle and is run over by a truck. On arrival in the
emergency room, he is awake and alert and appears frightened but in no distress. The chest
radiograph suggests an airfluid level in the left lower lung field and the nasogastric tube
seems to coil upward into the left chest. The next best step in management is
C. Immediate celiotomy >> “laparotomy”

16. Which of the following conditions is most likely to follow a compression-type abdominal
injury?
A. Renal vascular injury
B. Superior mesenteric thrombosis
C. Mesenteric vascular injury
D. Avulsion of the splenic pedicle
E. Diaphragmatic hernia

17. A 65-year-old man who smokes cigarettes and has chronic obstructive pulmonary
disease falls and fractures the 7th, 8th, and 9th ribs in the left anterolateral chest. Chest x-
ray is otherwise normal. Appropriate treatment might include
D. Peritoneal lavage

18. A 27-year-old man sustains a single gunshot wound to the left thigh. In the emergency
room he is noted to have a large hematoma of his medial thigh. He complains of
paresthesias in his foot. On examination there are weak pulses palpable distal to the injury
and the patient is unable to move his foot. The appropriate initial management of this
patient would be
B. Immediate exploration and repair

Items 19–20 A 25-year-old woman arrives in the emergency room following an


automobile accident. She is acutely dyspneic with a respiratory rate of 60
breaths/min. Breath sounds are markedly diminished on the right side.

19. The first step in managing the patient should be to


C. Decompress the right pleural space

20. A chest x-ray of this woman Before therapy would probably reveal
A. Air in the right pleural space

21. Among the physiologic responses to acute injury is


A. Increased secretion of insulin
B. Increased secretion of thyroxine
C. Decreased secretion of vasopressin (ADH)
D. Decreased secretion of glucagon

22. In a stable patient, the management of a complete transection of the common bile duct
distal to the insertion of the cystic duct would be optimally performed with a
D. Roux-en-Y choledochojejunostomy

23. Non operative management of penetrating neck injuries has been advocated as an
alternative to mandatory exploration in asymptomatic patients. Which of the following
findings would constitute a relative, rather than an absolute, indication for formal neck
exploration?
A. Expanding hematoma
B. Dysphagia
C. Dysphonia
D. Pneumothorax
E. Hemoptysis

24. Following blunt abdominal trauma, a 12-year-old girl develops upper abdominal pain,
nausea, and vomiting. An upper gastrointestinal series reveals a total obstruction of the
duodenum with a “coiled spring” appearance in the second and third portions.
Appropriate management is
B. Nasogastric suction and observation

25. Following traumatic peripheral nerve transection, regrowth usually occurs at which of
the following rates?
A. 0.1 mm per day
B. 1 mm per day
C. 5 mm per day
D. 1 cm per day
Items 26–27 A 28-year-old man is brought to the emergency room for a severe
head injury after a fall. Initially lethargic, he becomes comatose and does not
move his right side .His left pupil is dilated and responds only sluggishly.

26. The most common initial manifestation of increasing intracranial pressure in the victim
of head trauma is
A. Change in level of consciousness

27. Initial emergency reduction of intracranial pressure is most rapidly accomplished by


E. Hyperventilation

28. In the patient described, compression of the affected nerve is produced by


B. Herniation of the uncal process of the temporal lobe

29. Regarding high-voltage electrical burns to an extremity


D. Evaluation for fracture of the other extremities and visceral injury is indicated

30. Which of the following fractures or dislocations of the extremities induced by blunt
trauma is associated with
significant vascular injuries?
A. Knee dislocation

31. Protein metabolism after trauma is characterized by


A. Decreased liver gluconeogenesis
B. Inhibition of skeletal muscle breakdown)
C. Decreased urinary nitrogen loss
D. Hepatic synthesis of acute-phase reactants

32. The response to shock includes which of the following metabolic effects?
A. Increase in sodium and water excretion
B. Increase in renal perfusion
C. Decrease in cortisol levels
D. Hyperkalemia
E. Hypoglycemia

33. Appropriate treatment for an acute stable hematoma of the pinna of the ear includes
which of the following measures?
A. Ice packs and prophylactic antibiotics
B. Excision of the hematoma
C. Needle aspiration
D. Incision, drainage, and pressure bandage

34. Animal and clinical studies have shown that administration of lactated Ringer’s
solution to patients with hypovolemic shock may
A. Increase serum lactate concentration
B. Impair liver function
C. Improve hemodynamics by alleviating the deficit in the interstitial fluid compartment

35. Which of the following situations would be an indication for performance of a


thoracotomy in the emergency room?
a. Massive hemothorax following blunt trauma to the chest
b. Blunt trauma to multiple organ systems with obtainable vital signs in the field but none on
arrival in the emergency room
c. Rapidly deteriorating patient with cardiac tamponade from penetrating thoracic trauma
d. Penetrating thoracic trauma and no signs of life in the field
e. Penetrating abdominal trauma and no signs of life in the field

Items 36–37 An 18-year-old high school football player is kicked in the left
flank. Three hours later he develops hematuria.His vital signs are stable.
36. The diagnostic tests performed reveal extravasation of contrast into the renal
parenchyma .Treatment should consist of
E. Antibiotics and serial monitoring of blood count and vital signs

37. Initial diagnostic tests in the emergency room should include which of the following?
D. Intravenous pyelogram

38. True statements concerning penetrating pancreatic trauma include


E. The major cause of death is exsanguination from associated vascular injuries

39. A 26-year-old man sustains a gunshot wound to the left thigh. Exploration reveals that
a 5-cm portion of superficial femoral artery is destroyed .Appropriate management
includes
A. Debridement and end-to-end anastomosis
B. Debridement and repair with an interposition prosthetic graft
C. Debridement and repair with an interposition arterial graft
D. Debridement and repair with an interposition vein graft

Pretest Surgery ( 3 )
1. Estrogen receptor activity is clinically useful in predicting
A. The presence of ovarian cancer
B. The presence of metastatic disease
C. Response to chemotherapy
D. Response to hormonal manipulation

2. When galactorrhea occurs in a high school student, a diagnostic associated finding would
be
A. Gonadal atrophy
B. Bitemporal hemianopia
C. Exophthalmos and lid lag
E. “Buffalo hump”

3. The diagnosis of primary hyperparathyroidism is most strongly suggested by


A. Serum acid phosphatase above 120 IU/L
B. Serum alkaline phosphatase above 120 IU/L
C. Serum calcium above 11 mg/dL
D. Urinary calcium below 100 mg/day
4. Somatostatin contributes to which of the following processes?
B. Inhibition of pancreatic cells

5. A 35-year-old woman undergoes her first screening mammogram. Which of the


following mammographic findings would require a breast biopsy?
A. Breast calcifications larger than 2 mm in diameter
B. Five or more clustered breast microcalcifications per square centimeter
C. A density that effaces with compression
E. Multiple round well-circumscribed breast densities

6. Which of the following statements concerning Cushing syndrome secondary to adrenal


adenoma is true?
A. Adrenal adenomas cause 40–60% of all cases of Cushing syndrome
B. Biochemical and x-ray procedures are generally unsuccessful in lateralizing the tumors
preoperatively
C. Exploration of both adrenal glands is indicated
D. For uncomplicated tumors, an open transperitoneal surgical approach is usually employed

7. Which statement concerning radiation-induced thyroid cancer is true?


E. The treatment of choice is a neartotal (or total) thyroidectomy

8. Fibrocystic disease of the breast has been associated with elevated blood levels of
A. Testosterone
B. Progesterone
C. Estrogen
D. Luteinizing hormone
9. As an incidental finding during an upper abdominal CT scan, a 3-cm mass in the adrenal
gland is noted. The appropriate next step in analysis and management of this finding would
be
A. Observation
B. CT-guided needle biopsy
C. Excision of the mass
D. Measurement of urine catecholamine excretion
E. Cortisol provocation test

10. The most likely diagnosis in a patient with hypertension, hypokalemia, and a 7-cm
suprarenal mass is
A. Hypernephroma
B. Cushing’s disease
C. Adrenocortical carcinoma
D. Pheochromocytoma

11. Appropriate treatment of this condition would include which of the following?
A. Embolization of the arterial blood supply,
B. Metronidazole
C. Mitotane (toxic for functional adrenocortical cells)
E. Phenoxybenzamine

12. For pregnant women who are found to have breast cancer
E. Administration of adjuvant chemotherapy is safe for the fetus during the second and third
trimesters

13. True statements regarding Paget’s disease of the breast include that it
A. Usually precedes development of Paget’s disease of bone
B. Presents with nipple-areolar eczematous changes
C. Does not involve axillary lymph nodes because it is a manifestation of intraductal
carcinoma only
D. Accounts for 10–15% of all newly diagnosed breast cancers
E. Is adequately treated with wide excision when it presents as a mass

14. Of the common complications of thyroidectomy, the one that may be avoided through
prophylaxis is
A. Injury to the recurrent laryngeal nerve
B. Injury to the superior laryngeal nerve
D. Thyroid storm
E. Postoperative hemorrhage and wound hematoma

15. Following correction of the patient’s hypercalcemia with hydration and gentle diuresis
with furosemide, the most likely therapeutic approach would be
D. Neck exploration and resection of a parathyroid adenoma

16. This 30-year-old woman presented with weakness, bone pain, an elevated
parathormone level, and a serum calcium level of 15.2 mg/dL. Skeletal survey films were
taken, including the hand films and chest x-ray shown. The most likely cause of these
findings is
E. Primary hyperparathyroidism

17. A 36-year-old woman, 20 wk pregnant, presents with a 1.5-cm right thyroid mass. Fine-
needle aspiration is consistent with a papillary neoplasm. The mass is “cold” by scan and
solid by ultrasound. Which method of treatment would be contraindicated?
A. Right thyroid lobectomy
B. Subtotal thyroidectomy
C. Total thyroidectomy
D. Total thyroidectomy with lymph node dissection
E. 131I radioactive ablation of the thyroid gland

18. Incisional biopsy of a breast mass in a 35-year-old woman demonstrates a hypercellular


fibroadenoma (cystosarcoma phylloides) at the time of frozen section. Appropriate
management of this lesion could include
A. Wide local excision with a rim of normal tissue

19. True statements about discharge from the nipple include


A. Intermittent thin or milky discharge can be physiologic
B. Expressible nipple discharge is an indication for open biopsy
C. Bloody discharge is indicative of an underlying malignancy
D. Galactorrhea is indicative of an underlying malignancy

20. The incidence of breast cancer


A. Increases with increasing age
B. Has declined since the 1940s
C. Is related to dietary fat intake
D. Is related to coffee intake
E. Is related to vitamin C intake

21. A cross-match is performed by incubating


B. Donor lymphocytes with recipient serum and complement

22. The primary mechanism of action of cyclosporine A is inhibition of


D. Interleukin 2 production
23. After the first postoperative year of cardiac transplantation, the most common cause of
death is
C. Accelerated graft arteriosclerosis

24. Which of the following precludes cadaveric renal transplantation?


A. Positive cross-match

25. In centers with experienced personnel, 1-year liver transplant survival is now
approximately
B. 80%

26. Graft-versus-host disease has occurred with the transplantation of which of the
following?
D. Bone marrow

27. The most useful serum marker for detecting recurrent disease after treatment of
nonseminomatous testicular cancer is
B. alpha-fetoprotein (AFP)

28. For which of the following malignancies does histologic grade best correlate with
prognosis?
E. Soft tissue sarcoma

29. Interferons are correctly characterized by which of the following statements?


B. They are produced by virus-infected cells

30. Which of the following potentially operable complications is a common occurrence


among patients receiving systemic chemotherapy?
B. Perirectal abscess

31. Human immunodeficiency virus (HIV) has been isolated from many body fluids. Which
of the following is a major source of transmission?
A. Tears
B. Sweat
C. Semen
D. Urine
E. Breast milk

32. Which of the following agents causes hemorrhagic cystitis?


E. Cyclophosphamide

33. What is the most common serious complication of an end colostomy?


A. Bleeding
B. Skin breakdown
C. Parastomal hernia
D. Colonic perforation during irrigation
E. Stomal prolapse

34. Which of the following colonic pathologies is thought to have no malignant potential?
A. Ulcerative colitis
B. Villous adenomas
C. Familial polyposis
D. Peutz-Jeghers syndrome
E. Crohn’s colitis

35. Which of the following hernias follows the path of the spermatic cord within the
cremaster muscle?
A. Femoral
B. Direct inguinal
C. Indirect inguinal
D. Spigelian

36. Spontaneous closure of which of the following congenital abnormalities of the


abdominal wall generally occurs by the age of 4?
A. Umbilical hernia
B. Patent urachus
C. Patent omphalomesenteric duct
D. Omphalocele

37. Laparoscopic cholecystectomy is indicated for symptomatic gallstones in which of the


following conditions?
A. Cirrhosis
B. Prior upper abdominal surgery
C. Suspected carcinoma of the gallbladder
D. Morbid obesity
E. Coagulopathy

38. In determining the proper treatment for a sliding hiatal hernia, the most useful step
would be
A. Barium swallow with cinefluoroscopy during Valsalva maneuver
B. Flexible endoscopy
C. 24-h monitoring of esophageal pH
D. Measuring the size of the hernia

39. A previously healthy 9-year old child comes to the emergency room because of
fulminant upper gastrointestinal bleeding. The hemorrhage is most likely to be the result of
A. Esophageal varices
B. Mallory-Weiss syndrome
C. Gastritis
D. A gastric ulcer

40. Intragastric pressure remains steady near 2–5 mm Hg during slow gastric filling, but
rises rapidly to high levels after reaching a volume of
A. 400–600 mL
B. 700–900 mL
C. 1000–1200 mL
D. 1300–1500 mL

41. Local stimuli that inhibit the release of gastrin from the gastric mucosa include which
of the following?
A. Small proteins
B. 20-proof alcohol
C. Caffeine
D. Acidic antral contents

42. For a symptomatic partial duodenal obstruction secondary to an annular pancreas, the
operative treatment of choice is
A. A Whipple procedure
B. Gastrojejunostomy
D. Partial resection of the annular pancreas
E. Duodenojejunostomy
43. Which of the following would be expected to stimulate intestinal motility?
A. Fear
B. Gastrin
C. Secretin
D. Acetylcholine
E. Cholecystokinin

44. Which of the following statements concerning carcinoma of the esophagus is true?
B. Squamous carcinoma is the most common type at the cardioesophageal junction
C. It has a higher incidence in males
D. It occurs more commonly in patients with corrosive esophagitis
E. Surgical excision is the only effective treatment

45. The most common clinical presentation of idiopathic retroperitoneal fibrosis is


A. Ureteral obstruction
B. Leg edema
D. Jaundice
E. Intestinal obstruction

46. In planning the management of a 2.8-cm epidermoid carcinoma of the anus, the first
therapeutic approach should be
E. Combined radiation therapy and chemotherapy

47. Indications for operation in Crohn’s disease include which of the following?
A. Intestinal obstruction
B. Enterovesical fistula
C. Ileum–ascending colon fistula
E. Free perforation

48. Which of the following is most likely to require surgical correction?


A. Large sliding esophageal hiatal hernia
B. Paraesophageal hiatal hernia
C. Traction diverticulum of esophagus
D. Schatzki’s ring of distal esophagus

49. Which statement regarding adenocarcinoma of the pancreas is true?


A. It occurs most frequently in the body of the gland
B. It carries a 1–2% 5-year survival rate
C. It is nonresectable if it presents as painless jaundice
D. It can usually be resected if it presents in the body or tail of the pancreas and does not
involve the common bile duct

50. Which of the following statements regarding direct inguinal hernias is true?
A. They are the most common inguinal hernias in women
B. They protrude medially to the inferior epigastric vessels
C. They should be opened and ligated at the internal ring
D. They commonly protrude into the scrotal sac in men

51. Which of the following statements regarding stress ulceration is true?


A. It is true ulceration, extending into and through the muscularis mucosa
B. It classically involves the antrum
C. Increased secretion of gastric acid has been shown to play a causative role
D. It frequently involves multiple sites

52. Which statement concerning cholangitis is correct?


A. The most common infecting organism is Staphylococcus aureus
B. The diagnosis is suggested by the Charcot triad (fever, jaundice, pain)
C. The disease occurs primarily in young, immunocompromised patients
D. Cholecystectomy is the procedure of choice in affected patients

53. Indications for surgical removal of polypoid lesions of the gallbladder include
A. Size greater than 0.5 cm
B. Presence of clinical symptoms
C. Patient age of over 25 years
D. Presence of multiple small lesions

54. A patient who has a total pancreatectomy might be expected to develop which of the
following complications?
A. Diabetes mellitus
B. Hypercalcemia
C. Hyperphosphatemia
D. Constipation
E. Weight gain

55. True statements regarding cavernous hemangiomata of the liver in adults include
A. The majority become symptomatic
B. They may undergo malignant transformation
C. They enlarge under hormonal stimulation
D. They should be resected to avoid spontaneous rupture and lifethreatening hemorrhage

Pretest Surgery ( 4 )
1. The superior vena cava syndrome is most frequently seen in association with
A. Histoplasmosis (sclerosing mediastinitis)
C. Thoracic aortic aneurysm
D. Constrictive pericarditis
E. Bronchogenic carcinoma

2. A 3-year-old child with congenital cyanosis is most probably suffering from


A. Tetralogy of Fallot
B. Ventricular septal defect
C. Tricuspid atresia
D. Transposition of the great vessels

3. Superior pulmonary sulcus carcinomas (Pancoast tumors) are bronchogenic carcinomas


that typically produce which of the following clinical features?
A. Atelectasis of the involved apical segment
B. Horner syndrome (miosis, ptosis, anhidrosis)
C. Pain in the T4 and T5 dermatomes
D. Nonproductive cough

4. Which of the following statements is true concerning aortocoronary bypass grafting?


A. It is indicated for crescendo (preinfarction) angina

8. Which of the following statements is true regarding the thoracic outlet syndrome?
C. If conservative measures fail, it is best treated by surgical decompression of the brachial
plexus

6. A 2-year-old asymptomatic child is noted to have a systolic murmur, hypertension, and


diminished femoral pulses. Which of the following is true about this child’s disorder?
C. Rib notching is often seen on x-ray (Coarctation of the aorta)

7. A correct statement concerning bronchial carcinoid tumors is that


C. They rarely produce the carcinoid syndrome
8. Patients with phlebographically confirmed deep vein thrombosis of the calf
E. Are at risk for significant pulmonary embolism

9. Symptoms or signs of atherosclerotic occlusive disease of the bifurcation of the


abdominal aorta (Leriche syndrome) include
A. Claudication of the buttock and thigh
b. Causalgia of the lower leg
c. Retrograde ejaculation
d. Gangrene of the feet
e. Dependent rubor of the feet

10. Initial management of a patient who has a flaccid neurogenic bladder may include
which of the following measures?
A. Surgical bladder augmentation
B. Self-catheterization
C. Supravesical urinary diversion
D. Limiting fluid intake to less than 300 mL/day

11. The recommended treatment for stage A (superficial and sub mucosal)transitional cell
carcinoma of the bladder is
A. Local excision
B. Radical cystectomy
D. Topical (intravesicular) chemotherapy
E. Systemic chemotherapy

12. Seminoma is accurately described by which of the following statements?


A. It is the most common type of testicular cancer
B. Metastases to liver and bone are frequently found
C. It does not respond to radiation
E. Common presentation is that of a painful lump that trans illuminates

13. Meniscal tears usually result from which of the following circumstances?
A. Hyperextension
B. Flexion and rotation
C. Simple hyperflexion
D. Compression

14. In an uncomplicated dislocation of the glenohumeral joint, the humeral head usually
dislocates primarily in which of the following directions?
A. Anteriorly
B. Superiorly
C. Posteriorly
D. Laterally

15. Which of the following fractures is most commonly seen in healthy bones subjected to
violent falls?
A. Colles fracture
B. Femoral neck fracture
C. Intertrochanteric fracture
D. Clavicular fracture

16. Which of the following statements regarding compartment syndromes following


orthopedic injuries is true?
A. The first sign is usually loss of pulse in the extremity
B. Passive flexion of the extremity proximal to the involved compartment will aggravate the
pain
C. Surgical decompression (fasciectomy) is necessary only as a last resort
D. These syndromes are most commonly associated with supracondylar fractures of the
humerus and tibial shaft

17. In contrast to closed reduction, open reduction of a fracture


A. Produces a shorter healing time
B. Decreases trauma to the fracture site
C. Produces a higher incidence of nonunion
D. Reduces the risk of infection

18. In a failed suicide gesture, a depressed student severs her Radial nerve at the wrist. The
expected disability is
A. Loss of ability to extend the wrist
B. Loss of ability to flex the wrist
C. Wasting of the intrinsic muscles of the hand
D. Sensory loss over the thenar pad and the thumb web

19. The most severe epiphyseal growth disturbance is likely to result from which of the
following types of fracture?
E. Crushing injury compressing the growth plate
20. Which of the following statements regarding the Glasgow coma scale is true?
B. A high score correlates with a high mortality
C. It includes measurement of intracranial pressure
D. It includes measurement of pupillary reflexes
E. It includes measurement of verbal response

21. Which of the following statements regarding glioblastoma multiforme is true?


B. It arises from the malignant degeneration of an astrocytoma

22. Which of the following statements regarding skull fractures is true?


C. Any bone fragment displaced more than 1 cm inwardly should be elevated surgically

23. An acute increase in intracranial pressure is characterized by which of the following


clinical findings?
A. Respiratory irregularities
B. Decreased blood pressure
C. Tachycardia
D. Papilledema

24. Which of the following statements about schwannomas is true?


B. Treatment is via excision

25. Which of the following statements regarding cerebral contusions is true?


B. They may occur opposite the point of skull impact

26. True statements regarding meningiomas include that they


C. Are treated primarily by surgical excision

27. Which of the following statements about craniopharyngiomas is true?


D. The tumors may cause compression of the optic tracts and visual symptoms

28. Which of the following statements regarding symptomatic thyroglossal duct cysts is
true?
A. Over 90% manifest themselves before age 12
B. Treatment includes resection of the hyoid bone
C. They usually present as a painful swelling in the lateral neck

29. Pleomorphic adenomas (mixed tumors) of the salivary glands are characterized by
which of the following?
A. They occur most commonly on the lips, tongue, and palate

30. Verrucous carcinoma of the buccal mucosa is identified with which of the following
characteristics?
C. It has a predilection for the gingivobuccal gutter

IV Fluids Management Test


1)Normal blood volume for 3Kg Neonate is approximately :
A ) 180cc B) 260 cc C) 360cc D)480 cc

2) The most appropriate crystalloid used for resuscitation of hypovolemic trauma patient is
A- Ringer lactate B- DW5%
C-0,45% Normal saline D-Gelatines

3)All of the following can be seen in patients with hypovolemia EXCEPT


a) Increased heart rate b) Wide pulse pressure
c) Decreased urine volume d) Flat neck veins

4) A 4-year-old child was struck by a car traveling approximately 30 MPH and was thrown
approximately 15 feet. She is complaining of abdominal pain. Physical exam reveals a blood
pressure of 68/40 mmHg, a heart rate of 200 beats per minute, and a firm, tender abdomen. She
weighs approximately 20 kg. As long as the systolic blood pressure remains below 90 mmHg,
what is the most appropriate fluid management?
(A) 800 cc of normal saline followed by 400 cc of blood.
(B) 400 cc of normal saline followed by 200 cc of blood.
(C) 400 cc of normal saline repeated 3 times, then 100 cc of blood.
(D) 800 cc of normal saline repeated 3 times, then 400 cc of blood.
(E) 400 cc of normal saline repeated 3 times, then 200 cc of blood

5) Lower extremity cannulation ( IV Branula ) is contraindicated in patients with


A- Hypertension B- Respiratory distress
C- Hyperthemia D- Diabetes Mellitus

6) Which fluid is inappropriate for resuscitation a victim with severe burns ?


A) 10% dextrose solution B) Hypertonic saline
C) Plasma protein fraction D) Ringer lactate solution

7) Which statement about iv cannulation is FALSE


A) External jugular IV is indicated for critically ill patients < 12y old in whom an extremity vein
is not obtainable.
B) Intraosseous IV is indicated for critically ill patients < 6y old in whom an extremity vein is
not obtainable
C) Umbilical veins is indicated for new born <1week
D) Upper extremity IV sites are preferable to lower extremity sites .
E) The acellular fluids have the highest flow rates

8) Lactated Ringer contains all of the following EXCEPT


A) Potassium B) Calcium C) Sodium D) Magnesium

9)Total body water for 60 kg male adult is approximately :


A) 30 Liter B) 36liter C) 42 liter D) 48 liter

10)Which of the following contains potassium ?


A) Ringer lactate B)Normal saline C)Pediatric saline D) All of the above

11)Which of the following is contraindicated in head injury


A) Ringer lactate B) Normal saline C) Dextrose water D) All of the above

12)Indications for external jugular vein cannulation is :


A) Baby > 7 days B) Baby < 7 days C) Child >12Y D) Child <12Y

13)------- ml of starch are needed to replace 1ml blood


A) 1 B) 2 C)3 D)4

effective volume expander------------------------- 14)


A) Ringer lactate B)Normal saline C) Starch D) All of the above

15)Laboratory signs of dehydration include all of the following EXCEPT


A) Rising hematocrit B) Rising Urea
C) Hypernatremia D) Respiratory acidosis

16)Which statement about Body weight of fluid is true ?


A)Adult male 50-55% B) Female 55-60% C) Newborn 75-80% D) All of the above

17)Which of the following is present more extracellular ?


A) Na B) K C) PO4 D) All of the above

18)Indications for umbilical vein cannulation is :


A) Baby > 7 days B) Baby < 7 days C) Baby >28 days D) Baby < 7 weeks

19)If the radius of a catheter doubled , the flow rate will :


A) Increase sixteen fold B) Decrease sixteen fold C)Be doubled D) None of the above
20)If the recipient’s blood type is not known , ----blood may be used (packed cells )
A) Type 0 Rh-negative B) Type 0 Rh-positive
C) Type AB Rh-negative D) Type AB Rh- positive

You might also like